SlideShare a Scribd company logo
1 of 150
Download to read offline
Chuyên · 
SÈ HÅC 
Di¹n àn Toán håc
Chuy¶n · 
SÈ HÅC 
Ch¸ b£n 
Tr¦n Quèc Nhªt H¥n [perfectstrong] 
Tr¦n Trung Ki¶n [Ispectorgadget] 
Ph¤m Quang To n [Ph¤m Quang To n] 
L¶ Húu i·n Khu¶ [Nesbit] 
inh Ngåc Th¤ch [T*genie*] 

c 2012 Di¹n  n To¡n håc
Líi giîi thi»u 
B¤n åc th¥n m¸n, 
Sè håc l  mët ph¥n mæn quan trång trong to¡n håc ¢ g­n bâ vîi 
chóng ta xuy¶n suèt qu¡ tr¼nh håc To¡n tø bªc tiºu håc ¸n trung håc 
phê thæng. Chóng ta ÷ñc ti¸p xóc vîi Sè håc b­t ¦u b¬ng nhúng 
kh¡i ni»m ìn gi£n nh÷ t½nh chia h¸t, ÷îc chung lîn nh§t, bëi chung 
nhä nh§t... gióp l m quen d¹ d ng hìn vîi sü k¼ di»u cõa nhúng con 
sè cho ¸n nhúng v§n · ái häi nhi·u t÷ duy hìn nh÷ çng d÷, sè 
nguy¶n tè, c¡c ph÷ìng tr¼nh Diophantine m  nêi ti¸ng nh§t l  ành lþ 
lîn Fermat..., ¥u ¥u tø t¦m vi mæ ¸n v¾ mæ, tø cªu b² lîp mët bi 
bæ 4 chia h¸t cho 2 ¸n Gi¡o s÷ thi¶n t i Andrew Wiles (ng÷íi gi£i 
quy¸t b i to¡n Fermat), chóng ta ·u câ thº th§y ÷ñc hìi thð cõa Sè 
håc trong â. 
Sè håc quan trång nh÷ vªy nh÷ng l¤ thay sè chuy¶n · vi¸t v· nâ l¤i 
khæng nhi·u n¸u em so vîi kho t ng ç së c¡c b i vi¸t v· b§t ¯ng 
thùc tr¶n c¡c di¹n  n m¤ng. Xu§t ph¡t tø sü thi¸u höt â công nh÷ 
º k¿ ni»m trán mët n«m Di¹n  n To¡n håc khai tr÷ìng trang 
chõ mîi (16/01/2012 - 16/01/2013), nhâm bi¶n tªp chóng tæi còng vîi 
nhi·u th nh vi¶n t½ch cüc cõa di¹n  n ¢ chung tay bi¶n so¤n mët 
chuy¶n · gûi ¸n b¤n åc. 
Chuy¶n · l  tªp hñp c¡c b i vi¸t ri¶ng l´ cõa c¡c t¡c gi£ Nguy¹n M¤nh 
Tròng D÷ìng (duongld) , Nguy¹n Tr¦n Huy (yeutoan11), Nguy¹n 
Trung Hi¸u (nguyentrunghieua), Ph¤m Quang To n (Ph¤m Quang 
To n), Tr¦n Nguy¹n Thi¸t Qu¥n (L Lawliet), Tr¦n Trung Ki¶n (Is-pectorgadget), 
Nguy¹n ¼nh Tòng (tungc3sp)... còng sü gâp sùc 
i
ii 
gi¡n ti¸p cõa nhi·u th nh vi¶n t½ch cüc tr¶n Di¹n  n To¡n håc nh÷ 
Nguyen Lam Thinh, nguyenta98, Karl Heinrich Marx, The 
Gunner, perfectstrong... 
Ki¸n thùc · cªp trong chuy¶n · tuy khæng mîi nh÷ng câ thº gióp 
c¡c b¤n ph¦n n o hiºu s¥u hìn mët sè kh¡i ni»m cì b£n trong Sè håc 
công nh÷ trao êi còng c¡c b¤n nhi·u d¤ng b i tªp hay v  khâ tø c§p 
ë d¹ ¸n c¡c b i to¡n trong c¡c k¼ thi Håc sinh giäi quèc gia, quèc t¸. 
Chuy¶n · gçm 7 ch÷ìng. Ch÷ìng 1 · cªp ¸n c¡c kh¡i ni»m v· ×îc 
v  Bëi. Sè nguy¶n tè v  mët sè b i to¡n v· nâ ÷ñc giîi thi»u trong 
ch÷ìng 2. Ch÷ìng 3 nâi s¥u hìn v· C¡c b i to¡n chia h¸t. Ph÷ìng 
tr¼nh nghi»m nguy¶n, Ph÷ìng tr¼nh çng d÷ ÷ñc ph¡c håa trong 
c¡c ch÷ìng 4 v  5. H» th°ng d÷ v  ành lþ Th°ng d÷ Trung Hoa 
s³ ÷ñc gûi ¸n chóng ta qua ch÷ìng 6 tr÷îc khi k¸t thóc chuy¶n · 
b¬ng Mët sè b i to¡n sè håc hay tr¶n VMF ð ch÷ìng 7. 
Do thíi gian chu©n bà g§p rót nëi dung chuy¶n · ch÷a ÷ñc ¦u t÷ 
thªt sü t¿ m¿ công nh÷ câ thº cán nhi·u sai sât trong c¡c b i vi¸t, 
chóng tæi mong b¤n åc thæng c£m. Måi sü õng hë, âng gâp, ph¶ 
b¼nh cõa ëc gi£ s³ l  nguçn ëng vi¶n tinh th¦n to lîn cho ban bi¶n 
tªp công nh÷ cho c¡c t¡c gi£ º nhúng phi¶n b£n cªp nhªt sau cõa 
chuy¶n · ÷ñc tèt hìn, âng gâp nhi·u hìn núa cho kho t ng håc 
thuªt cõa cëng çng to¡n m¤ng. Chóng tæi hi vång qua chuy¶n · n y 
s³ gióp c¡c b¤n t¼m th¶m ÷ñc c£m hùng trong sè håc v  th¶m y¶u v´ 
µp cõa nhúng con sè. Måi trao êi gây þ xin gûi v· àa ch¿ email : 
contact@diendantoanhoc.net. 
Tr¥n trång, 
Nhâm bi¶n tªp Chuy¶n · Sè håc. 
Di¹n  n To¡n håc Chuy¶n · Sè håc
Möc löc 
i Líi giîi thi»u 
1 
Ch֓ng 1 
×îc v  Bëi 
1.1 ×îc sè, ÷îc sè chung, ÷îc sè chung lîn nh§t 1 
1.2 Bëi sè, bëi sè chung, bëi sè chung nhä nh§t 4 
1.3 B i tªp · nghà 6 
9 
Ch֓ng 2 
Sè Nguy¶n Tè 
2.1 Mët sè ki¸n thùc cì b£n v· sè nguy¶n tè 9 
2.2 Mët sè b i to¡n cì b£n v· sè nguy¶n tè 13 
2.3 B i tªp 19 
2.4 Phö löc: B¤n n¶n bi¸t 24 
29 Ch֓ng 3 
B i to¡n chia h¸t 
3.1 Lþ thuy¸t cì b£n 29 
3.2 Ph÷ìng ph¡p gi£i c¡c b i to¡n chia h¸t 31 
57 
Ch֓ng 4 
Ph÷ìng tr¼nh nghi»m nguy¶n 
iii
iv Möc löc 
4.1 X²t t½nh chia h¸t 57 
4.2 Sû döng b§t ¯ng thùc 74 
4.3 Nguy¶n t­c cüc h¤n, lòi væ h¤n 86 
89 
Ch֓ng 5 
Ph÷ìng tr¼nh çng d÷ 
5.1 Ph÷ìng tr¼nh çng d÷ tuy¸n t½nh 89 
5.2 Ph÷ìng tr¼nh çng d÷ bªc cao 90 
5.3 H» ph÷ìng tr¼nh çng d÷ bªc nh§t mët ©n 90 
5.4 Bªc cõa ph÷ìng tr¼nh çng d÷ 95 
5.5 B i tªp 95 
5.6 Ùng döng ành lþ Euler º gi£i ph÷ìng tr¼nh 
çng d÷ 96 
5.7 B i tªp 101 
103 
Ch֓ng 6 
H» th°ng d÷ v  ành lþ Th°ng d÷ Trung Hoa 
6.1 Mët sè k½ hi»u sû döng trong b i vi¸t 103 
6.2 H» th°ng d÷ 104 
6.3 ành l½ th°ng d÷ Trung Hoa 117 
6.4 B i tªp · nghà  gñi þ  ¡p sè 125 
129 
Ch֓ng 7 
Mët sè b i to¡n sè håc hay tr¶n VMF 
7.1 m3 + 17 
... 
3n 129 
7.2 c(ac + 1)2 = (5c + 2)(2c + b) 136 
141 T i li»u tham kh£o 
Di¹n  n To¡n håc Chuy¶n · Sè håc
Ch֓ng 
1 
×îc v  Bëi 
1.1 ×îc sè, ÷îc sè chung, ÷îc sè chung 
lîn nh§t 1 
1.2 Bëi sè, bëi sè chung, bëi sè chung 
nhä nh§t 4 
1.3 B i tªp · nghà 6 
Nguy¹n M¤nh Tròng D÷ìng (duongld) 
Nguy¹n Tr¦n Huy (yeutoan11) 
×îc v  bëi l  2 kh¡i ni»m quan trång trong ch÷ìng tr¼nh sè håc THCS. 
Chuy¶n · n y s³ giîi thi»u nhúng kh¡i ni»m v  t½nh ch§t cì b£n v· 
÷îc, ÷îc sè chung, ÷îc chung lîn nh§t, bëi, bëi sè chung, bëi chung 
nhä nh§t. Mët sè b i tªp · nghà v· c¡c v§n · n y công s³ ÷ñc · 
cªp ¸n ð cuèi b i vi¸t. 
1.1 ×îc sè, ÷îc sè chung, ÷îc sè chung lîn nh§t 
Trong ph¦n n y, chóng tæi s³ tr¼nh b y mët sè kh¡i ni»m v· ÷îc sè, 
÷îc sè chung v  ÷îc sè chung lîn nh§t k±m theo mët v i t½nh ch§t cõa 
chóng. Mët sè b i tªp v½ dö cho b¤n åc tham kh£o công s³ ÷ñc ÷a 
ra. 
1.1.1 ành ngh¾a 
ành ngh¾a 1.1 Sè tü nhi¶n d6= 0 ÷ñc gåi l  mët ÷îc sè cõa sè tü 
nhi¶n a khi v  ch¿ khi a chia h¸t cho d. Ta nâi d chia h¸t a, k½ hi»u dja. 
Tªp hñp c¡c ÷îc cõa a l : U(a) = fd 2 N : djag. 4 
1
2 1.1. ×îc sè, ÷îc sè chung, ÷îc sè chung lîn nh§t 
T½nh ch§t 1.1 N¸u U(a) = f1; ag th¼ a l  sè nguy¶n tè.  
ành ngh¾a 1.2 N¸u U(a) v  U(b) câ nhúng ph¦n tû chung th¼ nhúng 
ph¦n tû â gåi l  ÷îc sè chung cõa a v  b. Ta k½ hi»u: 
USC(a; b) = fd 2 N : (dja) ^ (djb)g 
= fd 2 N : (d 2 U(a)) ^ (d 2 U(b))g: 
T½nh ch§t 1.2 N¸u USC(a; b) = f1g th¼ a v  b nguy¶n tè còng nhau. 
ành ngh¾a 1.3 Sè d 2 N ÷ñc gåi l  ÷îc sè chung lîn nh§t cõa a v  b 
(a; b 2 Z) khi d l  ph¦n tû lîn nh§t trong tªp USC(a; b). Kþ hi»u ÷îc 
chung lîn nh§t cõa a v  b l  UCLN(a; b), (a; b) hay gcd(a; b). 4 
1.1.2 T½nh ch§t 
Sau ¥y l  mët sè t½nh ch§t cõa ÷îc chung lîn nh§t: 
 N¸u (a1; a2; : : : :; an) = 1 th¼ ta nâi c¡c sè a1; a2; : : : ; an nguy¶n 
tè còng nhau. 
 N¸u (am; ak) = 1; 8m6= k; fm; kg 2 f1; 2; : : : ; ng th¼ ta nâi c¡c 
a1; a2; : : : ; an æi mët nguy¶n tè còng nhau. 
 c 2 USC(a; b) th¼ 
 
a 
c 
; 
b 
c 
 
= 
(a; b) 
c 
. 
 d = (a; b) , 
 
a 
d 
; 
b 
d 
 
= 1. 
 (ca; cb) = c(a; b). 
 (a; b) = 1 v  bjac th¼ bjc. 
 (a; b) = 1 v  (a; c) = 1 th¼ (a; bc) = 1. 
 (a; b; c) = ((a; b); c). 
 Cho a  b  0 
 N¸u a = b:q th¼ (a; b) = b. 
 N¸u a = bq + r(r6= 0) th¼ (a; b) = (b; r). 
Di¹n  n To¡n håc Chuy¶n · Sè håc
1.1. ×îc sè, ÷îc sè chung, ÷îc sè chung lîn nh§t 3 
1.1.3 C¡ch t¼m ÷îc chung lîn nh§t b¬ng thuªt to¡n Euclide 
º t¼m (a; b) khi a khæng chia h¸t cho b ta dòng thuªt to¡n Euclide 
sau: 
 a = b:q + r1 th¼ (a; b) = (b; r1). 
 b = r1:q1 + r2 th¼ (b; r1) = (r1; r2). 
    
 rn2 = rn1:qn1 + rn th¼ (rn2; rn1) = (rn1; rn). 
 rn1 = rn:qn th¼ (rn1; rn) = rn. 
 (a; b) = rn. 
 (a; b) l  sè d÷ cuèi còng kh¡c 0 trong thuªt to¡n Euclide. 
1.1.4 B i tªp v½ dö 
V½ dö 1.1. T¼m (2k  1; 9k + 4); k 2 N. 4 
Líi gi£i. Ta °t d = (2k  1; 9k + 4). Theo t½nh ch§t v· ÷îc sè chung 
ta câ dj2k1 v  dj9k+4. Ti¸p töc ¡p döng t½nh ch§t v· chia h¸t ta l¤i 
câ dj9(2k  1) v  dj2(9k + 4). Suy ra dj2(9k + 4)  9(2k  1) hay dj17. 
Vªy (2k  1; 9k + 4) = 1.  
V½ dö 1.2. T¼m (123456789; 987654321). 4 
Líi gi£i. °t b = 123456789; a = 987654321. Ta nhªn th§y a v  b ·u 
chia h¸t cho 9. 
Ta l¤i câ : 
a + b = 1111111110 
= 
1010  10 
9 
: 
, 9a + 9b = 1010  10 
(1.1) 
M°t kh¡c : 
10b + a = 9999999999 
= 1010  1: 
(1.2) 
Chuy¶n · Sè håc Di¹n  n To¡n håc
4 1.2. Bëi sè, bëi sè chung, bëi sè chung nhä nh§t 
Trø (1.2) v  (1.1) v¸ theo v¸ ta ÷ñc b8a = 9. Do â n¸u °t d = (a; b) 
th¼ 9 
.. .d. 
M  a v  b ·u chia h¸t cho 9, suy ra d = 9.  
Düa v o thuªt to¡n Euclide, ta câ líi gi£i kh¡c cho V½ dö 1.2 nh÷ sau : 
Líi gi£i.  987654321 = 123456789:8+9 th¼ (987654321; 123456789) = 
(123456789; 9). 
 123456789 = 9:1371421. 
 (123456789; 987654321) = 9.  
V½ dö 1.3. Chùng minh r¬ng d¢y sè An = 
1 
2 
n(n + 1); n 2 N chùa 
nhúng d¢y sè væ h¤n nhúng sè æi mët nguy¶n tè còng nhau. 4 
Líi gi£i. Gi£ sû trong d¢y ang x²t câ k sè æi mët nguy¶n tè còng 
nhau l  t1 = 1; t2 = 3; : : : ; tk = m(m 2 N). °t a = t1t2: : : tk. X²t sè 
h¤ng t2a+1 trong d¢y An: 
t2a+1 = 
1 
2 
(2a + 1)(2a + 2) 
= (a + 1)(2a + 1) 
 tk 
M°t kh¡c ta câ (a + 1; a) = 1 v  (2a + 1; a) = 1 n¶n (t2a+1; a) = 1. 
Do â t2a+1 nguy¶n tè còng nhau vîi t§t c£ k sè ft1; t2; : : : tkg. Suy ra 
d¢y sè An chùa væ h¤n nhúng sè æi mët nguy¶n tè còng nhau.  
1.2 Bëi sè, bëi sè chung, bëi sè chung nhä nh§t 
T÷ìng tü nh÷ c§u tróc ¢ tr¼nh b y ð ph¦n tr÷îc, trong ph¦n n y 
chóng tæi công s³ ÷a ra nhúng ành ngh¾a, t½nh ch§t cì b£n cõa bëi 
sè, bëi sè chung, bëi sè chung nhä nh§t v  mët sè b i tªp v½ dö minh 
håa. 
Di¹n  n To¡n håc Chuy¶n · Sè håc
1.2. Bëi sè, bëi sè chung, bëi sè chung nhä nh§t 5 
1.2.1 ành ngh¾a 
ành ngh¾a 1.4 Sè tü nhi¶n m ÷ñc gåi l  mët bëi sè cõa a6= 0 khi 
v  ch¿ khi m chia h¸t cho a hay a l  mët ÷îc sè cõa m. 4 
Nhªn x²t. Tªp hñp c¡c bëi sè cõa a6= 0 l : B(a) = f0; a; 2a; : : : ; kag; k 2 
Z. 
ành ngh¾a 1.5 Sè tü nhi¶n m ÷ñc gåi l  mët bëi sè cõa a6= 0 khi 
v  ch¿ khi m chia h¸t cho a hay a l  mët ÷îc sè cõa m 4 
ành ngh¾a 1.6 N¸u 2 tªp B(a) v  B(b) câ ph¦n tû chung th¼ c¡c ph¦n 
tû chung â gåi l  bëi sè chung cõa a v  b. Ta kþ hi»u bëi sè chung 
cõa a v  b: BSC(a; b). 
ành ngh¾a 1.7 Sè m6= 0 ÷ñc gåi l  bëi chung nhä nh§t cõa a v  
b khi m l  ph¦n tû d÷ìng nhä nh§t trong tªp BSC(a; b). Kþ hi»u : 
BCNN(a; b), [a; b] hay lcm(a; b). 4 
1.2.2 T½nh ch§t 
Mët sè t½nh ch§t cõa bëi chung lîn nh§t: 
 N¸u [a; b] = M th¼ 
 
M 
a 
; 
M 
b 
 
= 1. 
 [a; b; c] = [[a; b]; c]. 
 [a; b]:(a; b) = a:b. 
1.2.3 B i tªp v½ dö 
V½ dö 1.4. T¼m [n; n + 1; n + 2]. 4 
Líi gi£i. °t A = [n; n + 1] v  B = [A; n + 2]. p döng t½nh ch§t 
[a; b; c] = [[a; b]; c], ta câ: B = [n; n + 1; n + 2]. 
D¹ th§y (n; n + 1) = 1, suy ra [n; n + 1] = n(n + 1). 
Chuy¶n · Sè håc Di¹n  n To¡n håc
6 1.3. B i tªp · nghà 
L¤i ¡p döng t½nh ch§t [a; b] = 
a:b 
(a; b) 
th¸ th¼ 
[n; n + 1; n + 2] = 
n(n + 1)(n + 2) 
(n(n + 1); n + 2) 
. 
Gåi d = (n(n + 1); n + 2). Do (n + 1; n + 2) = 1 n¶n 
d = (n; n + 2) 
= (n; 2): 
X²t hai tr÷íng hñp: 
 N¸u n ch®n th¼ d = 2, suy ra [n; n + 1; n + 2] = 
n(n + 1)(n + 2) 
2 
. 
 N¸u n l´ th¼ d = 1, suy ra [n; n + 1; n + 2] = n(n + 1)(n + 2) .  
V½ dö 1.5. Chùng minh r¬ng [1; 2; : : : 2n] = [n + 1; n + 2; : : : ; 2n]. 4 
Líi gi£i. Ta th§y ÷ñc trong k sè nguy¶n li¶n ti¸p câ mët v  ch¿ mët sè 
chia h¸t cho k. Do â b§t trong c¡c sè f1; 2; : : : ; 2ng ·u l  ÷îc cõa mët 
sè n o â trong c¡c sè fn + 1; n + 2; : : : ; 2ng. Do â [1; 2; : : : n; 2n] = 
[n + 1; n + 2; : : : ; 2n].  
1.3 B i tªp · nghà 
Thay cho líi k¸t, chóng tæi xin gûi ¸n b¤n åc mët sè b i tªp · nghà 
º luy»n tªp nh¬m gióp c¡c b¤n quen hìn vîi c¡c kh¡i ni»m v  c¡c 
t½nh ch§t tr¼nh b y trong chuy¶n ·. 
B i 1. a: Cho A = 5a + 3b;B = 13a + 8b(a; b 2 N) chùng minh 
(A;B) = (a; b). 
b: Têng qu¡t A = ma+nb;B = pa+qb thäa m¢n jmqnpj = 
1 vîi a; b; m; n; p; q 2 N. Chùng minh (A;B) = (a; b). 
B i 2. T¼m (6k + 5; 8k + 3)(k 2 N). 
Di¹n  n To¡n håc Chuy¶n · Sè håc
1.3. B i tªp · nghà 7 
B i 3. Tø c¡c chú sè 1; 2; 3; 4; 5; 6 th nh lªp t§t c£ sè câ s¡u chú sè 
(méi sè ch¿ vi¸t mët l¦n). T¼m UCLN cõa t§t c£ c¡c sè â. 
B i 4. Cho A = 2n + 1;B = 
n(n + 1) 
2 
(n 2 N). T¼m (A;B). 
B i 5. a: Chùng minh r¬ng trong 5 sè tü nguy¶n li¶n ti¸p bao gií 
công chån ÷ñc mët sè nguy¶n tè còng nhau vîi c¡c sè 
cán l¤i. 
b: Chùng minh r¬ng trong 16 sè nguy¶n li¶n ti¸p bao gií công 
chån ÷ñc mët sè nguy¶n tè còng nhau vîi c¡c sè cán l¤i. 
B i 6. Cho 1  m  n(m; n 2 N). 
a: Chùng minh r¬ng (22n 
 1; 22n 
+ 1) = 1. 
b: T¼m (2m  1; 2n  1). 
B i 7. Cho m; n 2 N vîi (m; n) = 1. T¼m (m2 + n2;m + n). 
B i 8. Cho A = 2n+3;B = 2n+1+3n+1(n 2 N);C = 2n+2+3n+2(n 2 
N). T¼m (A;B) v  (A;C). 
B i 9. Cho s¡u sè nguy¶n d÷ìng a; b; a0; b0; d; d0 sao cho (a; b) = d; (a0; b0) = 
d0. Chùng minh r¬ng (aa0; bb0; ab0; a0b) = dd0. 
B i 10. Chùng minh r¬ng d¢y sè Bn = 
1 
6 
n(n + 1)(n + 2)(n 2 N) chùa 
væ h¤n nhúng sè nguy¶n tè còng nhau. 
B i 11. Chùng minh r¬ng d¢y sè 2n  3 vîi måi n 2 N v  n  2 chùa 
d¢y sè væ h¤n nhúng sè nguy¶n tè còng nhau. 
B i 12. Chùng minh d¢y Mersen Mn = 2n  1(n 2 N) chùa d¢y sè væ 
h¤n nhúng sè nguy¶n tè còng nhau. 
B i 13. Chùng minh r¬ng d¢y Fermat Fn = 22n 
+ 1(n 2 N) l  d¢y sè 
nguy¶n tè còng nhau. 
B i 14. Cho n 2 N; n  1 v  2n  2 chia h¸t cho n. T¼m (22n 
; 2n  1). 
Chuy¶n · Sè håc Di¹n  n To¡n håc
8 1.3. B i tªp · nghà 
B i 15. Chùng minh r¬ng vîi måi n 2 N, ph¥n sè 21n + 1 
14n + 3 
tèi gi£n. 
B i 16. Cho ba sè tü nhi¶n a; b; c æi mët nguy¶n tè còng nhau. Chùng 
minh r¬ng (ab + bc + ca; abc) = 1. 
B i 17. Cho a; b 2 N. Chùng minh r¬ng tçn t¤i væ sè n 2 N sao cho 
(a + n; b + n) = 1. 
B i 18. Gi£ sû m; n 2 N(m  n) thäa m¢n (199k1;m) = (19931; n). 
Chùng minh r¬ng tçn t¤i t(t 2 N) sao cho m = 1993t:n. 
B i 19. Chùng minh r¬ng n¸u a;m 2 N; a  1 th¼ 
 
am  1 
a  1 
; a  1 
 
= 
(m; a  1). 
B i 20. T¼m sè nguy¶n d÷ìng n nhä nh§t º c¡c ph¥n sè sau tèi gi£n: 
a: 
1 
n1996 + 1995n + 2 
, 
b: 
2 
n1996 + 1995n + 3 
, 
c: 
1994 
n1996 + 1995n + 1995 
, 
d: 
1995 
n1996 + 1995n + 1996 
. 
B i 21. Cho 20 sè tü nhi¶n kh¡c 0 l  a1; a2; : : : an câ têng b¬ng S 
v  UCLN b¬ng d. Chùng minh r¬ng UCLN cõa S  a1; S  
a2; : : : ; S  an b¬ng t½ch cõa d vîi mët ÷îc n o â cõa n  1. 
Di¹n  n To¡n håc Chuy¶n · Sè håc
Ch֓ng 
2 
Sè Nguy¶n Tè 
2.1 Mët sè ki¸n thùc cì b£n v· sè 
nguy¶n tè 9 
2.2 Mët sè b i to¡n cì b£n v· sè nguy¶n 
tè 13 
2.3 B i tªp 19 
2.4 Phö löc: B¤n n¶n bi¸t 24 
Nguy¹n Trung Hi¸u (nguyentrunghieua) 
Ph¤m Quang To n (Ph¤m Quang To n) 
2.1 Mët sè ki¸n thùc cì b£n v· sè nguy¶n tè 
2.1.1 ành ngh¾a, ành lþ cì b£n 
ành ngh¾a 2.1 Sè nguy¶n tè l  nhúng sè tü nhi¶n lîn hìn 1, ch¿ câ 2 
÷îc sè l  1 v  ch½nh nâ. 4 
ành ngh¾a 2.2 Hñp sè l  sè tü nhi¶n lîn hìn 1 v  câ nhi·u hìn 2 
֔c. 4 
Nhªn x²t. C¡c sè 0 v  1 khæng ph£i l  sè nguy¶n tè công khæng ph£i 
l  hñp sè. B§t ký sè tü nhi¶n lîn hìn 1 n o công câ ½t nh§t mët ÷îc 
sè nguy¶n tè. 
ành lþ 2.1 D¢y sè nguy¶n tè l  d¢y sè væ h¤n.  
9
10 2.1. Mët sè ki¸n thùc cì b£n v· sè nguy¶n tè 
Chùng minh. Gi£ sû ch¿ câ húu h¤n sè nguy¶n tè l  p1; p2; p3; :::; pn; 
trong â pn l  sè lîn nh§t trong c¡c nguy¶n tè. 
X²t sè N = p1p2:::pn + 1 th¼ N chia cho méi sè nguy¶n tè pi(i = 1; n) 
·u d÷ 1 (*) 
M°t kh¡c N l  mët hñp sè (v¼ nâ lîn hìn sè nguy¶n tè lîn nh§t l  pn) 
do â N ph£i câ mët ÷îc nguy¶n tè n o â, tùc l  N chia h¸t cho mët 
trong c¡c sè pi (**). 
Ta th§y (**) m¥u thu¨n (*). Vªy khæng thº câ húu h¤n sè nguy¶n tè. 
ành lþ 2.2 Måi sè tü nhi¶n lîn hìn 1 ·u ph¥n t½ch ÷ñc ra thøa 
sè nguy¶n tè mët c¡ch duy nh§t (khæng kº thù tü c¡c thøa sè).  
Chùng minh. * Måi sè tü nhi¶n lîn hìn 1 ·u ph¥n t½ch ÷ñc ra thøa 
sè nguy¶n tè: 
Thªt vªy: gi£ sû i·u kh¯ng ành tr¶n l  óng vîi måi sè m tho£ m¢n: 
1  m  n ta chùng minh i·u â óng ¸n n. 
N¸u n l  nguy¶n tè, ta câ i·u ph£i chùng minh. 
N¸u n l  hñp sè, theo ành ngh¾a hñp sè, ta câ: n = a:b (vîi a; b  n) 
Theo gi£ thi¸t quy n¤p: a v  b l  t½ch c¡c thøa sè nhä hìn n n¶n n l  
t½ch cu£ c¡c thøa sè nguy¶n tè. 
* Sü ph¥n t½ch l  duy nh§t: 
Gi£ sû måi sè m  n ·u ph¥n t½ch ÷ñc ra thøa sè nguy¶n tè mët 
c¡ch duy nh§t, ta chùng minh i·u â óng ¸n n: 
N¸u n l  sè nguy¶n tè th¼ ta ÷ñc i·u ph£i chùng minh. N¸u n l  hñp 
sè: Gi£ sû câ 2 c¡ch ph¥n t½ch n ra thøa sè nguy¶n tè kh¡c nhau: 
n = p:q:r:::: 
n = p0:q0:r0:::: 
Trong â p; q; r::::: v  p0; q0; r0:::: l  c¡c sè nguy¶n tè v  khæng câ sè 
nguy¶n tè n o công câ m°t trong c£ hai ph¥n t½ch â (v¼ n¸u câ sè 
tho£ m¢n i·u ki»n nh÷ tr¶n, ta câ thº chia n cho sè â lóc â th÷íng 
s³ nhä hìn n, th÷ìng n y câ hai c¡ch ph¥n t½ch ra thøa sè nguy¶n tè 
kh¡c nhau, tr¡i vîi gi£ thi¸t cõa quy n¤p). 
Khæng m§t t½nh têng qu¡t, ta câ thº gi£ thi¸t p v  p0 l¦n l÷ñt l  c¡c sè 
nguy¶n tè nhä nh§t trong ph¥n t½ch thù nh§t v  thù hai. 
V¼ n l  hñp sè n¶n n  p2 v  n  p02. Do p6= p ) n  p:p0 
Di¹n  n To¡n håc Chuy¶n · Sè håc
2.1. Mët sè ki¸n thùc cì b£n v· sè nguy¶n tè 11 
X²t m = n  pp0  n ÷ñc ph¥n t½ch ra thøa sè nguy¶n tè mët c¡ch 
duy nh§t ta th§y: 
pjn ) pjn  pp0 hay pjm 
Khi ph¥n t½ch ra thøa sè nguy¶n tè ta câ: m = npp0 = p0p:P:Q::: vîi 
P;Q 2 P ( P l  tªp c¡c sè nguy¶n tè). 
) pp0jn ) pp0jp:q:r::: ) pjq:r::: ) p l  ÷îc nguy¶n tè cõa q:r::: 
M  p khæng tròng vîi mët thøa sè n o trong q; r::: (i·u n y tr¡i vîi 
g¿a thi¸t quy n¤p l  måi sè nhä hìn n ·u ph¥n t½ch ÷ñc ra thøa sè 
nguy¶n tè mët c¡ch duy nh§t). 
Vªy, i·u gi£ sû khæng óng. ành lþ ÷ñc chùng minh.  
2.1.2 C¡ch nhªn bi¸t mët sè nguy¶n tè 
C¡ch 1 
Chia sè â l¦n l÷ñt cho c¡c nguy¶n tè tø nhä ¸n lîn: 2; 3; 5; 7::: 
N¸u câ mët ph²p chia h¸t th¼ sè â khæng nguy¶n tè. 
N¸u thüc hi»n ph²p chia cho ¸n lóc th÷ìng sè nhä hìn sè chia m  c¡c 
ph²p chia v¨n câ sè d÷ th¼ sè â l  nguy¶n tè. 
C¡ch 2 
Mët sè câ hai ÷îc sè lîn hìn 1 th¼ sè â khæng ph£i l  sè nguy¶n tè. 
Cho håc sinh lîp 6 håc c¡ch nhªn bi¸t 1 sè nguy¶n tè b¬ng ph÷ìng 
ph¡p thù nh§t (n¶u ð tr¶n), l  düa v o ành lþ cì b£n: 
×îc sè nguy¶n tè nhä nh§t cõa mët hñp sè A l  mët sè khæng v÷ñt 
p 
qu¡ 
A. 
Vîi quy t­c tr¶n trong mët kho£n thíi gian ng­n, vîi c¡c d§u hi»u chia 
h¸t th¼ ta nhanh châng tr£ líi ÷ñc mët sè câ hai chú sè n o â l  
Chuy¶n · Sè håc Di¹n  n To¡n håc
12 2.1. Mët sè ki¸n thùc cì b£n v· sè nguy¶n tè 
nguy¶n tè hay khæng. 
H» qu£ 2.1 N¸u câ sè A  1 khæng câ mët ÷îc sè nguy¶n tè n o tø 
2 ¸n 
p 
A th¼ A l  mët nguy¶n tè.  
2.1.3 Sè c¡c ÷îc sè v  têng c¡c ÷îc sè cõa 1 sè 
Gi£ sû: A = px1 
1 :px2 
2 ::::::pnxn; trong â: pi 2 P; xi 2 N; i = 1; n 
T½nh ch§t 2.1 Sè c¡c ÷îc sè cõa A t½nh b¬ng cæng thùc: 
T(A) = (x1 + 1)(x2 + 1):::::(xn + 1) 
V½ dö 2.1. 30 = 2:3:5 th¼ T(A) = (1 + 1)(1 + 1)(1 + 1) = 8. Kiºm tra: 
(30) = f1; 2; 3; 5; 6; 10; 15; 30g n¶n (30) câ 8 ph¥n tû. 4 
T½nh ch§t 2.2 Têng c¡c ÷îc mët sè cõa A t½nh b¬ng cæng thùc: 
 (A) = 
Yn 
i=1 
pxi+1 
i  1 
pi  1 
2.1.4 Hai sè nguy¶n tè còng nhau 
ành ngh¾a 2.3 Hai sè tü nhi¶n ÷ñc gåi l  nguy¶n tè còng nhau khi 
v  ch¿ khi chóng câ ÷îc chung lîn nh§t (×CLN) b¬ng 1. 4 
T½nh ch§t 2.3 Hai sè tü nhi¶n li¶n ti¸p luæn nguy¶n tè còng nhau.  
T½nh ch§t 2.4 Hai sè nguy¶n tè kh¡c nhau luæn nguy¶n tè còng nhau. 
T½nh ch§t 2.5 C¡c sè a; b; c nguy¶n tè còng nhau khi v  ch¿ khi (a; b; c) 
= 1.  
ành ngh¾a 2.4 Nhi·u sè tü nhi¶n ÷ñc gåi l  nguy¶n tè s¡nh æi khi 
chóng æi mët nguy¶n tè còng nhau. 4 
Di¹n  n To¡n håc Chuy¶n · Sè håc
2.2. Mët sè b i to¡n cì b£n v· sè nguy¶n tè 13 
2.1.5 Mët sè ành lþ °c bi»t 
ành lþ 2.3 (Dirichlet) Tçn t¤i væ sè sè nguy¶n tè p câ d¤ng: 
p = ax + b (x; a; b 2 N, a; b l  2 sè nguy¶n tè còng nhau).  
Vi»c chùng minh ành lþ n y kh¡ phùc t¤p, trø mët sè tr÷íng hñp °c 
bi»t, ch¯ng h¤n câ væ sè sè nguy¶n tè d¤ng: 2x1; 3x1; 4x+3; 6x+ 
5; : : : 
ành lþ 2.4 (Tchebycheff-Betrand) Trong kho£ng tø sè tü nhi¶n 
n ¸n sè tü nhi¶n 2n câ ½t nh§t mët sè nguy¶n tè (n  2).  
ành lþ 2.5 (Vinogradow) Måi sè l´ lîn hìn 33 l  têng cõa 3 sè 
nguy¶n tè.  
2.2 Mët sè b i to¡n cì b£n v· sè nguy¶n tè 
2.2.1 Câ bao nhi¶u sè nguy¶n tè d¤ng ax + b 
V½ dö 2.2. Chùng minh r¬ng: câ væ sè sè nguy¶n tè câ d¤ng 3x  1.4 
Líi gi£i. Måi sè tü nhi¶n khæng nhä hìn 2 câ 1 trong 3 d¤ng: 3x; 3x+1 
ho°c 3x  1 
 Nhúng sè câ d¤ng 3x (vîi x  1) l  hñp sè 
 X²t 2 sè câ d¤ng 3x + 1: â l  sè 3m + 1 v  sè 3n + 1. 
X²t t½ch (3m + 1)(3n + 1) = 9mn + 3m + 3n + 1. T½ch n y câ 
d¤ng: 3x + 1 
 L§y mët sè nguy¶n tè p b§t câ d¤ng 3x  1, ta lªp t½ch cõa p 
vîi t§t c£ c¡c sè nguy¶n tè nhä hìn p rçi trø i 1 ta câ: M = 
2:3:5:7::::p  1 = 3(2:5:7::::p)  1 th¼ M câ d¤ng 3x  1. 
Câ 2 kh£ n«ng x£y ra: 
1. Kh£ n«ng 1: M l  sè nguy¶n tè, â l  sè nguy¶n tè câ d¤ng 
3x  1  p, b i to¡n ÷ñc chùng minh. 
Chuy¶n · Sè håc Di¹n  n To¡n håc
14 2.2. Mët sè b i to¡n cì b£n v· sè nguy¶n tè 
2. Kh£ n«ng 2: M l  hñp sè: Ta chia M cho 2; 3; 5; ::::; p ·u tçn 
t¤i mët sè d÷ kh¡c 0 n¶n c¡c ÷îc nguy¶n tè cõa M ·u lîn 
hìn p, trong c¡c ÷îc n y khæng câ sè n o câ d¤ng 3x+1 (¢ 
chùng minh tr¶n). Do â ½t nh§t mët trong c¡c ÷îc nguy¶n 
tè cõa M ph£i câ d¤ng 3x (hñp sè) ho°c 3x + 1 
V¼ n¸u t§t c£ câ d¤ng 3x+1 th¼ M ph£i câ d¤ng 3x+1 (¢ chùng 
minh tr¶n). Do â, ½t nh§t mët trong c¡c ÷îc nguy¶n tè cõa M 
ph£i câ d¤ng 3x  1, ÷îc n y luæn lîn hìn p. 
Vªy: Câ væ sè sè nguy¶n tè d¤ng 3x  1.  
V½ dö 2.3. Chùng minh r¬ng: Câ væ sè sè nguy¶n tè câ d¤ng 4x+3.4 
Líi gi£i. Nhªn x²t. C¡c sè nguy¶n tè l´ khæng thº câ d¤ng 4x ho°c 
4x + 2. Vªy chóng ch¿ câ thº tçn t¤i d÷îi 1 trong 2 d¤ng 4x + 1 ho°c 
4x + 3. 
Ta s³ chùng minh câ væ sè sè nguy¶n tè câ d¤ng 4x + 3. 
 X²t t½ch 2 sè câ d¤ng 4x + 1 l : 4m + 1 v  4n + 1. 
Ta câ: (4m+1)(4n+1) = 16mn+4m+4n+1 = 4(4mn+m+n)+1. 
Vªy t½ch cõa 2 sè câ d¤ng 4x + 1 l  mët sè công câ d¤ng 4x + 1. 
 L§y mët sè nguy¶n tè p b§t ký câ d¤ng 4x+3, ta lªp t½ch cõa 4p 
vîi t§t c£ c¡c sè nguy¶n tè nhä hìn p rçi trø i 1 khi â ta câ: 
N = 4(2:3:5:7:::::p)  1. Câ 2 kh£ n«ng x£y ra 
1. N l  sè nguy¶n tè ) N = 4(2:3:5:7::::p)1 câ d¤ng 4x1. 
Nhúng sè nguy¶n tè câ d¤ng 4x  1 công ch½nh l  nhúng sè 
câ d¤ng 4x + 3 v  b i to¡n ÷ñc chùng minh. 
2. N l  hñp sè. Chia N cho 2; 3; 5; ::::; p ·u ÷ñc c¡c sè d÷ 
kh¡c 0. Suy ra c¡c ÷îc nguy¶n tè cõa N ·u lîn hìn p. 
C¡c ÷îc n y khæng thº câ d¤ng 4x ho°c 4x+2 (v¼ â l  hñp sè). 
Công khæng thº to n c¡c ÷îc câ d¤ng 4x + 1 v¼ nh÷ th¸ N ph£i 
câ d¤ng 4x + 1. Nh÷ vªy trong c¡c ÷îc nguy¶n tè cõa N câ ½t 
nh§t 1 ÷îc câ d¤ng 4x  1 m  ÷îc n y hiºn nhi¶n lîn hìn p. 
Di¹n  n To¡n håc Chuy¶n · Sè håc
2.2. Mët sè b i to¡n cì b£n v· sè nguy¶n tè 15 
Vªy: Câ væ sè sè nguy¶n tè câ d¤ng 4x  1 (hay câ d¤ng 4x + 3).  
Tr¶n ¥y l  mët sè b i to¡n chùng minh ìn gi£n cõa ành lþ Dirichlet: 
Câ væ sè sè nguy¶n tè d¤ng ax + b trong â a; b; x 2 N; (a; b) = 1. 
2.2.2 Chùng minh sè nguy¶n tè 
V½ dö 2.4. Chùng minh r¬ng: (p  1)! chia h¸t cho p n¸u p l  hñp sè, 
khæng chia h¸t cho p n¸u p l  sè nguy¶n tè. 4 
Líi gi£i.  X²t tr÷íng hñp p l  hñp sè: N¸u p l  hñp sè th¼ p l  t½ch 
cõa c¡c thøa sè nguy¶n tè nhä hìn p v  sè mô c¡c luÿ thøa n y 
khæng thº lîn hìn sè mô cõa ch½nh c¡c luÿ thøa §y chùa trong 
(p  1)!. Vªy: (p  1)! 
... 
p (pcm). 
 X²t tr÷íng hñp p l  sè nguy¶n tè: V¼ p 2 P ) p nguy¶n tè còng 
nhau vîi måi thøa sè cõa (p  1)! (pcm).  
V½ dö 2.5. Cho 2m  1 l  sè nguy¶n tè. Chùng minh r¬ng m công l  
sè nguy¶n tè. 4 
Líi gi£i. Gi£ sû m l  hñp sè ) m = p:q (p; q 2 N; p; q  1) 
Khi â: 2m1 = 2pq1 = (2p)q1 = (2p1)((2p)q1+(2p)q2+:::::+1) 
v¼ p  1 ) 2p  1  1 v  (2p)q1 + (2p)q2 + ::::: + 1  1 
D¨n ¸n 2m  1 l  hñp sè :tr¡i vîi gi£ thi¸t 2m1 l  sè nguy¶n tè. 
Vªy m ph£i l  sè nguy¶n tè (pcm)  
V½ dö 2.6. Chùng minh r¬ng: måi ÷îc nguy¶n tè cõa 1994!1 ·u lîn 
hìn 1994. 4 
Líi gi£i. Gåi p l  ÷îc sè nguy¶n tè cõa 1994!  1 
Gi£ sû p  1994 ) 1994:1993:::::3:2:1 
.. .p ) 1994! 
.. .p. 
M  1994!  1 
.. .p ) 1 
.. .p (væ lþ) 
Vªy: p  1994 (pcm).  
V½ dö 2.7. Chùng minh r¬ng: n 2 th¼ giúa n v  n! câ ½t nh§t 1 sè 
nguy¶n tè (tø â suy ra câ væ sè sè nguy¶n tè). 4 
Chuy¶n · Sè håc Di¹n  n To¡n håc
16 2.2. Mët sè b i to¡n cì b£n v· sè nguy¶n tè 
Líi gi£i. V¼ n  2 n¶n k = n!  1  1, do â k câ ½t nh§t mët ÷îc sè 
nguy¶n tè p. T÷ìng tü b i tªp 3, ta chùng minh ÷ñc måi ÷îc nguy¶n 
tè p cõa k ·u lîn hìn k. 
Vªy: p  n ) n  p  n!  1  n! (pcm)  
2.2.3 T¼m sè nguy¶n tè thäa m¢n i·u ki»n cho tr÷îc 
V½ dö 2.8. T¼m t§t c£ c¡c gi¡ trà cõa sè nguy¶n tè p º: p + 10 v  
p + 14 công l  sè nguy¶n tè. 4 
Líi gi£i. N¸u p = 3 th¼ p + 10 = 3 + 10 = 13 v  p + 14 = 3 + 14 = 17 
·u l  c¡c sè nguy¶n tè n¶n p = 3 l  gi¡ trà c¦n t¼m. 
N¸u p  3 ) p câ d¤ng 3k + 1 ho°c d¤ng 3k  1 
 N¸u p = 3k + 1 th¼ p + 14 = 3k + 15 = 3(k + 5) 
.. .3 
 N¸u p = 3k  1 th¼ p + 10 = 3k + 9 = 3(k + 3) 
... 
3 
Vªy n¸u p  3 th¼ ho°c p + 10 ho°c p + 14 l  hñp sè : khæng thäa m¢n 
b i. Vªy p = 3.  
V½ dö 2.9. T¼m k 2 N º trong 10 sè tü nhi¶n li¶n ti¸p: 
k + 1; k + 2; k + 3; ::::k + 10 
câ nhi·u sè nguy¶n tè nh§t. 4 
Líi gi£i. N¸u k = 0: tø 1 ¸n 10 câ 4 sè nguy¶n tè: 2; 3; 5; 7. 
N¸u k = 1: tø 2 ¸n 11 câ 5 sè nguy¶n tè: 2; 3; 5; 7; 11. 
N¸u k  1: tø 3 trð i khæng câ sè ch®n n o l  sè nguy¶n tè. Trong 5 
sè l´ li¶n ti¸p, ½t nh§t câ 1 sè l  bëi sè cõa 3 do â, d¢y s³ câ ½t hìn 5 
sè nguy¶n tè. 
Vªy vîi k = 1, d¢y t÷ìng ùng: k + 1; k + 2; :::::k + 10 câ chùa nhi·u sè 
nguy¶n tè nh§t (5 sè nguy¶n tè).  
V½ dö 2.10. T¼m t§t c£ c¡c sè nguy¶n tè p º: 2p+p2 công l  sè nguy¶n 
tè. 4 
Líi gi£i. X²t 3 tr÷íng hñp: 
Di¹n  n To¡n håc Chuy¶n · Sè håc
2.2. Mët sè b i to¡n cì b£n v· sè nguy¶n tè 17 
 p = 2 ) 2p + p2 = 22 + 22 = 862 P 
 p = 3 ) 2p + p2 = 23 + 32 = 17 2 P 
 p  3 ) p6 
.. .3. Ta câ 2p + p2 = (p2  1) + (2p + 1). 
V¼ p l´ ) 2p + 1 
... 
... 
3 v  p2  1 = (p + 1)(p  1) 
3 ) 2p + p262 P 
Vªy câ duy nh§t 1 gi¡ trà p = 3 tho£ m¢n.  
V½ dö 2.11. T¼m t§t c£ c¡c sè nguy¶n tè p sao cho: pj2p + 1. 4 
Líi gi£i. V¼ p 2 P : pj2p + 1 ) p  2 ) (2; p) = 1 
Theo ành lþ Fermat, ta câ: pj2p1  1. M  
pj2p + 1 ) pj2(2p1  1) + 3 ) pj3 ) p = 3 
Vªy: p = 3.  
2.2.4 Nhªn bi¸t sè nguy¶n tè 
V½ dö 2.12. N¸u p l  sè nguy¶n tè v  1 trong 2 sè 8p + 1 v  8p  1 l  
sè nguy¶n tè th¼ sè cán l¤i l  sè nguy¶n tè hay hñp sè? 4 
Líi gi£i.  N¸u p = 2 ) 8p + 1 = 17 2 P; 8p  1 = 1562 P 
 N¸u p = 3 ) 8p  1 = 23 2 P; 8p  1 = 2562 P 
 N¸u p  3, x²t 3 sè tü nhi¶n li¶n ti¸p: 8p1; 8p v  8p+1. Trong 
3 sè n y ­t câ 1 sè chia h¸t cho 3. N¶n mët trong hai sè 8p + 1 
v  8p  1 chia h¸t cho 3. 
K¸t luªn: N¸u p 2 P v  1 trong 2 sè 8p + 1 v  8p  1 l  sè nguy¶n tè 
th¼ sè cán l¤i ph£i l  hñp sè.  
V½ dö 2.13. N¸u p  5 v  2p + 1 l  c¡c sè nguy¶n tè th¼ 4p + 1 l  
nguy¶n tè hay hñp sè? 4 
Líi gi£i. X²t 3 sè tü nhi¶n li¶n ti¸p: 4p; 4p + 1; 4p + 2. Trong 3 sè ­t 
câ mët sè l  bëi cõa 3. 
M  p  5; p 2 P n¶n p câ d¤ng 3k + 1 ho°c 3k + 2 
 N¸u p = 3k + 1 th¼ 2p + 1 = 6k + 3 
.. .3: (tr¡i vîi gi£ thi¸t) 
Chuy¶n · Sè håc Di¹n  n To¡n håc
18 2.2. Mët sè b i to¡n cì b£n v· sè nguy¶n tè 
 N¸u p = 3k+2. Khi â 4p+1 = 4(3k+2)+1 = 12k+9 
.. .3 ) 4p+1 
l  hñp sè  
V½ dö 2.14. Trong d¢y sè tü nhi¶n câ thº t¼m ÷ñc 1997 sè li¶n ti¸p 
nhau m  khæng câ sè nguy¶n tè n o hay khæng ? 4 
Líi gi£i. Chån d¢y sè: (ai) : ai = 1998! + i + 1 (i = 1; 1997) ) ai 
.. .i + 
1 8i = 1; 1997 
Nh÷ vªy: D¢y sè a1; a2; a3; :::::a1997 gçm câ 1997 sè tü nhi¶n li¶n ti¸p 
khæng câ sè n o l  sè nguy¶n tè.  
V½ dö 2.15 (Têng qu¡t b i tªp 2.14). Chùng minh r¬ng câ thº t¼m 
÷ñc 1 d¢y sè gçm n sè tü nhi¶n li¶n ti¸p (n  1) khæng câ sè n o 
l  sè nguy¶n tè ? 4 
Líi gi£i. Ta chån d¢y sè sau: (ai) : ai = (n+1)!+i+1 ) ai 
.. .i+1 8i = 
1; n. 
B¤n åc h¢y tü chùng minh d¢y (ai) ð tr¶n s³ gçm câ n sè tü nhi¶n 
li¶n ti¸p trong â khæng câ sè n o l  sè nguy¶n tè c£.  
2.2.5 C¡c d¤ng kh¡c 
V½ dö 2.16. T¼m 3 sè nguy¶n tè sao cho t½ch cõa chóng g§p 5 l¦n têng 
cõa chóng. 4 
Líi gi£i. Gåi 3 sè nguy¶n tè ph£i t¼m l  a; b; c. Ta câ: abc = 5(a + b + 
c) ) abc 
... 
5 
V¼ a; b; c câ vai trá b¼nh ¯ng n¶n khæng m§t t½nh têng qu¡t, gi£ sû: 
a 
... 
5 ) a = 5 
Khi â: 5bc = 5(5 + b + c) , 5 + b + c = bc , (c  1)(b  1) = 6 
Do vªy: 
2 
664 
 
b  1 = 1 
c  1 = 6 
, 
 
b = 2 
c = 7 
chån 
 
b  1 = 2 
c  1 = 3 
, 
 
b = 3 
c = 4 
lo¤i 
Vªy bë sè (a; b; c) c¦n t¼m l  ho¡n và cõa (2; 5; 7).  
V½ dö 2.17. T¼m p; q 2 P sao cho p2 = 8q + 1. 4 
Di¹n  n To¡n håc Chuy¶n · Sè håc
2.3. B i tªp 19 
Líi gi£i. Ta câ: 
p2 = 8q + 1 ) 8q = p2  1 = (p + 1)(p  1) (2.1) 
Do p2 = 8q + 1 : l´ ) p2 : l´ ) p : l´. °t p = 2k + 1. 
Thay v o (2.1) ta câ: 
8q = 2k(2k + 2) ) 2q = k(k + 1) (2.2) 
N¸u q = 2 ) 4 = k(k + 1) ) khæng t¼m ÷ñc k 2 N 
Vªy q  2. V¼ q 2 P ) (2; q) = 1. 
Tø (2.2) ta câ: 
a) k = 2 v  q = k + 1 ) k = 2; q = 3. Thay k¸t qu£ tr¶n v o (2.2) 
ta câ: p = 2:2 + 1 = 5 
b) q = k v  2 = k + 1 ) q = 1 :lo¤i. 
Vªy (q; p) = (5; 3).  
2.3 B i tªp 
2.3.1 B i tªp câ h÷îng d¨n 
B i 1. Ta bi¸t r¬ng câ 25 sè nguy¶n tè nhä hìn 100. Têng cõa 25 sè 
nguy¶n tè nhä hìn 100 l  sè ch®n hay sè l´? 
HD :Trong 25 sè nguy¶n tè nhä hìn 100 câ chùa mët sè nguy¶n 
tè ch®n duy nh§t l  2, cán 24 sè nguy¶n tè cán l¤i l  sè l´. Do 
â têng cõa 25 sè nguy¶n tè l  sè ch®n. 
B i 2. Têng cõa 3 sè nguy¶n tè b¬ng 1012. T¼m sè nguy¶n tè nhä nh§t 
trong ba sè nguy¶n tè â. 
HD: V¼ têng cõa 3 sè nguy¶n tè b¬ng 1012, n¶n trong 3 sè 
nguy¶n tè â tçn t¤i ½t nh§t mët sè nguy¶n tè ch®n. M  sè 
nguy¶n tè ch®n duy nh§t l  2 v  l  sè nguy¶n tè nhä nh§t. Vªy 
sè nguy¶n tè nhä nh§t trong 3 sè nguy¶n tè â l  2. 
Chuy¶n · Sè håc Di¹n  n To¡n håc
20 2.3. B i tªp 
B i 3. Têng cõa 2 sè nguy¶n tè câ thº b¬ng 2003 hay khæng? V¼ sao? 
HD: V¼ têng cõa 2 sè nguy¶n tè b¬ng 2003, n¶n trong 2 sè 
nguy¶n tè â tçn t¤i 1 sè nguy¶n tè ch®n. M  sè nguy¶n tè 
ch®n duy nh§t l  2. Do â sè nguy¶n tè cán l¤i l  2001. Do 
2001 chia h¸t cho 3 v  2001  3. Suy ra 2001 khæng ph£i l  sè 
nguy¶n tè. 
B i 4. T¼m sè nguy¶n tè p, sao cho p + 2; p + 4 công l  c¡c sè nguy¶n 
tè. 
B i 5. Cho p v  p + 4 l  c¡c sè nguy¶n tè (p  3). Chùng minh r¬ng 
p + 8 l  hñp sè. 
HD: V¼ p l  sè nguy¶n tè v  p  3, n¶n sè nguy¶n tè p câ 1 
trong 2 d¤ng: 
... 
 N¸u p = 3k +2 th¼ p+4 = 3k +6 = 3(k +2) ) p+4 
3 v  
p + 4  3. Do â p + 4 l  hñp sè: tr¡i · b i. 
... 
 N¸u p = 3k +1 th¼ p+8 = 3k +9 = 3(k +3) ) p+8 
3 v  
p + 8  3. Do â p + 8 l  hñp sè. 
B i 6. Chùng minh r¬ng måi sè nguy¶n tè lîn hìn 2 ·u câ d¤ng 4n+1 
ho°c 4n  1. 
B i 7. T¼m sè nguy¶n tè, bi¸t r¬ng sè â b¬ng têng cõa hai sè nguy¶n 
tè v  b¬ng hi»u cõa hai sè nguy¶n tè. 
HD: Gi£ sû a; b; c; d; e l  c¡c sè nguy¶n tè v  d  e. Theo · 
b i: 
a = b + c = d  e () 
Tø (*) ) a  2 n¶n a l  sè nguy¶n tè l´ ) b+c; de l  sè l´. 
Do b; d l  c¡c sè nguy¶n tè ) b; d l  sè l´ ) c; e l  sè ch®n. 
) c = e = 2 (do c; el  sè nguy¶n tè) ) a = b + 2 = d  2 ) 
d = b + 4. 
Vªy ta c¦n t¼m sè nguy¶n tè b sao cho b + 2 v  b + 4 công l  
c¡c sè nguy¶n tè. 
Di¹n  n To¡n håc Chuy¶n · Sè håc
2.3. B i tªp 21 
B i 8. T¼m t§t c£ c¡c sè nguy¶n tè x, y sao cho: x2  6y2 = 1. 
B i 9. Cho p v  p + 2 l  c¡c sè nguy¶n tè (p  3). Chùng minh r¬ng 
p + 1 
... 
6. 
2.3.2 B i tªp khæng câ h÷îng d¨n 
B i 1. T¼m sè nguy¶n tè p sao cho c¡c sè sau công l  sè nguy¶n tè: 
a) p + 2 v  p + 10. 
b) p + 10 v  p + 20. 
c) p + 10 v  p + 14. 
d) p + 14 v  p + 20. 
e) p + 2 v  p + 8. 
f) p + 2 v  p + 14. 
g) p + 4 v  p + 10. 
h) p + 8 v  p + 10. 
B i 2. T¼m sè nguy¶n tè p sao cho c¡c sè sau công l  sè nguy¶n tè: 
a) p + 2; p + 8; p + 12; p + 14 
b) p + 2; p + 6; p + 8; p + 14 
c) p + 6; p + 8; p + 12; p + 14 
d) p + 2; p + 6; p + 8; p + 12; p + 14 
e) p + 6; p + 12; p + 18; p + 24 
f) p + 18; p + 24; p + 26; p + 32 
g) p + 4; p + 6; p + 10; p + 12; p + 16 
B i 3. Cho tr÷îc sè nguy¶n tè p  3 thäa 
a) p + 4 2 P. Chùng minh r¬ng: p + 8 l  hñp sè. 
b) 2p + 1 2 P. Chùng minh r¬ng: 4p + 1 l  hñp sè. 
c) 10p + 1 2 P. Chùng minh r¬ng: 5p + 1 l  hñp sè. 
Chuy¶n · Sè håc Di¹n  n To¡n håc
22 2.3. B i tªp 
d) p + 8 2 P. Chùng minh r¬ng: p + 4 l  hñp sè. 
e) 4p + 1 2 P. Chùng minh r¬ng: 2p + 1 l  hñp sè. 
f) 5p + 1 2 P. Chùng minh r¬ng: 10p + 1 l  hñp sè. 
g) 8p + 1 2 P. Chùng minh r¬ng: 8p  1 l  hñp sè. 
h) 8p  1 2 P. Chùng minh r¬ng: 8p + 1 l  hñp sè. 
i) 8p2  1 2 P. Chùng minh r¬ng: 8p2 + 1 l  hñp sè. 
j) 8p2 + 1 2 P. Chùng minh r¬ng: 8p2  1 l  hñp sè. 
B i 4. Chùng minh r¬ng: 
a) N¸u p v  q l  hai sè nguy¶n tè lîn hìn 3 th¼ p2  q2.. .24. 
b) N¸u a; a+k; a+2k(a; k 2 N) l  c¡c sè nguy¶n tè lîn hìn 
3 th¼ k 
... 
6. 
B i 5. a) Mët sè nguy¶n tè chia cho 42 câ sè d÷ r l  hñp sè. T¼m sè 
d÷ r. 
b) Mët sè nguy¶n tè chia cho 30 câ sè d÷ r. T¼m sè d÷ r bi¸t 
r¬ng r khæng l  sè nguy¶n tè. 
B i 6. T¼m sè nguy¶n tè câ ba chú sè, bi¸t r¬ng n¸u vi¸t sè â theo 
thù tü ng÷ñc l¤i th¼ ta ÷ñc mët sè l  lªp ph÷ìng cõa mët sè 
tü nhi¶n. 
B i 7. T¼m sè tü nhi¶n câ 4 chú sè, chú sè h ng ngh¼n b¬ng chú sè 
h ng ìn và, chú sè h ng tr«m b¬ng chú sè h ng chöc v  sè â 
vi¸t ÷ñc d÷îi d¤ng t½ch cõa 3 sè nguy¶n tè li¶n ti¸p. 
B i 8. T¼m 3 sè nguy¶n tè l  c¡c sè l´ li¶n ti¸p. 
B i 9. T¼m 3 sè nguy¶n tè li¶n ti¸p p; q; r sao cho p2 + q2 + r2 2 P. 
B i 10. T¼m t§t c£ c¡c bë ba sè nguy¶n tè a; b; c sao cho abc  ab + 
bc + ca. 
B i 11. T¼m 3 sè nguy¶n tè p; q; r sao cho pq + qp = r. 
Di¹n  n To¡n håc Chuy¶n · Sè håc
2.3. B i tªp 23 
B i 12. T¼m c¡c sè nguy¶n tè x; y; z tho£ m¢n xy + 1 = z. 
B i 13. T¼m sè nguy¶n tè abcd thäa ab; ac l  c¡c sè nguy¶n tè v  b2 = 
cd + b  c. 
B i 14. Cho c¡c sè p = bc + a; q = ab + c; r = ca + b(a; b; c 2 N) l  
c¡c sè nguy¶n tè. Chùng minh r¬ng 3 sè p; q; r câ ½t nh§t hai sè 
b¬ng nhau. 
B i 15. T¼m t§t c£ c¡c sè nguy¶n tè x; y sao cho: 
a) x2  12y2 = 1 
b) 3x2 + 1 = 19y2 
c) 5x2  11y2 = 1 
d) 7x2  3y2 = 1 
e) 13x2  y2 = 3 
f) x2 = 8y + 1 
B i 16. Chùng minh r¬ng i·u ki»n c¦n v  õ º p v  8p2 +1 l  c¡c sè 
nguy¶n tè l  p = 3. 
B i 17. Chùng minh r¬ng: N¸u a2b2 l  mët sè nguy¶n tè th¼ a2b2 = 
a + b. 
B i 18. Chùng minh r¬ng måi sè nguy¶n tè lîn hìn 3 ·u câ d¤ng 6n+1 
ho°c 6n  1. 
B i 19. Chùng minh r¬ng têng b¼nh ph÷ìng cõa 3 sè nguy¶n tè lîn hìn 
3 khæng thº l  mët sè nguy¶n tè. 
B i 20. Cho sè tü nhi¶n n  2. Gåi p1; p2; :::; pn l  nhúng sè nguy¶n tè 
sao cho pn  n+1. °t A = p1:p2:::pn. Chùng minh r¬ng trong 
d¢y sè c¡c sè tü nhi¶n li¶n ti¸p: A + 2;A + 3; :::;A + (n + 1), 
khæng chùa mët sè nguy¶n tè n o. 
B i 21. Chùng minh r¬ng: N¸u p l  sè nguy¶n tè th¼ 2:3:4:::(p3)(p 
2)  1 
.. .p. 
Chuy¶n · Sè håc Di¹n  n To¡n håc
24 2.4. Phö löc: B¤n n¶n bi¸t 
B i 22. Chùng minh r¬ng: N¸u p l  sè nguy¶n tè th¼ 2:3:4:::(p2)(p 
1) + 1 
.. .p. 
2.4 Phö löc: B¤n n¶n bi¸t 
M÷íi sè nguy¶n tè câ 93 chú sè lªp th nh c§p sè cëng 
Sau ¥y l  mët sè nguy¶n tè gçm 93 chú sè: 
100996972469714247637786655587969840329509324689190041 
803603417758904341703348882159067229719 
K löc n y do 70 nh  to¡n håc lªp ÷ñc n«m 1998 thªt khâ m  ¡nh 
b¤i ÷ñc. Hå m§t nhi·u th¡ng t½nh to¡n mîi t¼m ÷ñc m÷íi sè nguy¶n 
tè t¤o th nh mët c§p sè cëng. 
Tø möc trá chìi trong 1 t¤p ch½ khoa håc, hai nh  nghi¶n cùu ð tr÷íng 
¤i håc Lyonl (Ph¡p) ¢  o s¥u þ t÷ðng: T¼m 6 sè nguy¶n tè sao cho 
hi»u 2 sè li¶n ti¸p luæn luæn nh÷ nhau. i·u â l  d¹ èi vîi c¡c chuy¶n 
gia nh÷ng hå muèn i xa hìn. Công khæng câ v§n · g¼ khâ kh«n èi 
vîi mët d¢y 7 sè. Hå c¦n sü hé trñ mët chót º ¤t ÷ñc 8 sè, mët sü 
hé trñ hìn núa º ¤t tîi 9 sè. Cuèi còng th¡ng 3 n«m 1998 câ 70 nh  
to¡n håc tø kh­p tr¶n th¸ giîi còng vîi 200 m¡y i»n to¡n ho¤t ëng 
li¶n töc ¢ t¼m ra 10 sè, méi sè câ 93 chú sè, m  hi»u sè cõa 2 sè li¶n 
ti¸p luæn luæn l  210. Tø sè nguy¶n tè ð tr¶n ch¿ c¦n th¶m v o 210 l  
÷ñc sè nguy¶n tè thù 2.... 
K löc câ l³ døng ð â: Theo ÷îc t½nh cõa c¡c nh  khoa håc muèn t¼m 
÷ñc 1 d¢y 11 sè nguy¶n tè th¼ ph£i m§t hìn 10 t¿ n«m. 
Sinh ba r§t ½t, ph£i ch«ng sinh æi l¤i r§t nhi·u 
Ta bi¸t r¬ng c¡c sè nguy¶n tè câ thº xa nhau tuý þ i·u n y thº hi»n 
ð b i tªp: 
Di¹n  n To¡n håc Chuy¶n · Sè håc
2.4. Phö löc: B¤n n¶n bi¸t 25 
B i to¡n 2.1. Cho tr÷îc sè nguy¶n d÷ìng n tuý þ. Chùng minh r¬ng 
tçn t¤i n sè tü nhi¶n li¶n ti¸p m  méi sè trong chóng ·u l  hñp sè.4 
Vªy nh÷ng, c¡c sè nguy¶n tè công câ thº r§t g¦n nhau. C°p sè (2; 3) 
l  c°p sè tü nhi¶n li¶n ti¸p duy nh§t m  c£ hai b¶n ·u l  sè nguy¶n 
tè. C°p sè (p; q)÷ñc gåi l  c°p sè sinh æi, n¸u c£ 2 ·u l  sè nguy¶n 
tè v  q = p + 2. Bë 3 sè (p; q; r) gåi l  bë sè nguy¶n tè sinh ba n¸u 
c£ 3 sè p,q,r ·u l  c¡c sè nguy¶n tè v  q = p + 2; r = q + 2. 
B i to¡n 2.2. T¼m t§t c£ c¡c bë sè nguy¶n tè sinh ba? 4 
¥y l  mët b i to¡n d¹, dòng ph÷ìng ph¡p chùng minh duy nh§t ta 
t¼m ra bë (3; 5; 7) l  bë ba sè nguy¶n tè sinh ba duy nh§t, c¡c bë 3 sè 
l´ lîn hìn 3 luæn câ 1 sè l  hñp sè v¼ nâ chia h¸t cho 3. 
Tø b i to¡n 2.2 th¼ b i to¡n sau trð th nh mët gi£ thuy¸t lîn ang chí 
c¥u tr£ líi. 
Dü o¡n 2.1 Tçn t¤i væ h¤n c°p sè sinh æi.  
Sè ho n h£o (ho n to n) cõa nhúng ng÷íi Hy L¤p cê ¤i 
Ng÷íi Hy L¤p cê ¤i câ quan ni»m th¦n b½ v· c¡c sè. Hå r§t thó và 
ph¡t hi»n ra c¡c sè ho n h£o, ngh¾a l  c¡c sè tü nhi¶n m  têng c¡c ÷îc 
sè tü nhi¶n thüc sü cõa nâ (c¡c ÷îc sè nhä hìn sè â) b¬ng ch½nh nâ. 
Ch¯ng h¤n: 
6 = 1 + 2 + 3 28 = 1 + 2 + 4 + 7 + 14 
Ng÷íi Hy L¤p cê ¤i ¢ bi¸t t¼m t§t c£ c¡c sè ho n h£o ch®n ngh¾a l  
hå ¢ l m ÷ñc b i to¡n sau ¥y: 
B i to¡n 2.3. Mët sè tü nhi¶n ch®n n6= 0 l  sè ho n h£o n¸u v  ch¿ 
n¸u: n = 2m+1(2m  1). Trong â m l  sè tü nhi¶n kh¡c 0 sao cho 
2m  1 l  sè nguy¶n tè. 4 
Tø â ta câ gi£ thuy¸t 
Chuy¶n · Sè håc Di¹n  n To¡n håc
26 2.4. Phö löc: B¤n n¶n bi¸t 
Dü o¡n 2.2 Khæng tçn t¤i sè ho n h£o l´.  
Ð b i to¡n 2.3 tr¶n, sè nguy¶n tè d¤ng 2m  1 gåi l  sè nguy¶n tè 
Merseme. C¡c sè nguy¶n tè Merseme câ vai trá r§t quan trång. Cho 
¸n nay ng÷íi ta v¨n ch÷a bi¸t câ húu h¤n hay væ h¤n sè nguy¶n tè 
Merseme. 
Dü o¡n 2.3 Tçn t¤i væ h¤n sè nguy¶n tè Merseme.  
N«m 1985 sè nguy¶n tè lîn nh§t m  ng÷íi ta bi¸t l  sè 21320491 gçm 
39751 chú sè ghi trong h» thªp ph¥n. G¦n ¥y 2 sinh vi¶n Mÿ ¢ t¼m 
ra mët sè nguy¶n tè lîn hìn núa â l  sè 22160911 gçm 65050 chú sè. 
Ta bi¸t r¬ng vîi håc sinh lîp 6 º thû xem sè A câ ½t hìn 20 chú sè 
câ l  sè nguy¶n tè khæng b¬ng c¡ch thû xem A câ chia h¸t cho sè n o 
nhä hìn A hay khæng, th¼ º t¼m h¸t c¡c sè nguy¶n tè vîi chi¸c m¡y 
si¶u i»n to¡n c¦n h ng th¸ k !!! 
David SlowinSky ¢ so¤n mët ph¦n m·m, l m vi»c tr¶n m¡y si¶u i»n 
to¡n Gray-2 , sau 19 gií æng ¢ t¼m ra sè nguy¶n tè 2756839 1. Sè n y 
vi¸t trong h» thªp ph¥n s³ câ 227832 chú sè- vi¸t h¸t sè n y c¦n 110 
trang v«n b£n b¼nh th÷íng. Ho°c n¸u vi¸t h ng ngang nhúng sè tr¶n 
phæng chú .VnTime Size 14 th¼ ta c¦n kho£ng 570 m. 
Líi K¸t 
Thæng qua · t i n y, chóng ta câ thº kh¯ng ành r¬ng: To¡n håc câ 
m°t trong måi cæng vi»c, måi l¾nh vüc cõa cuëc sèng quanh ta, nâ 
khæng thº t¡ch ríi v  l¢ng qu¶n ÷ñc, n¶n chóng ta ph£i hiºu bi¸t v  
n­m b­t ÷ñc nâ mët c¡ch tü gi¡c v  hi»u qu£. 
Möc ½ch cõa · t i n y l  trang bà nhúng ki¸n thùc cì b£n câ  o 
s¥u câ n¥ng cao v  r±n luy»n t÷ duy to¡n håc cho håc sinh, t¤o ra n·n 
t£ng tin cªy º c¡c em câ vèn ki¸n thùc nh§t ành l m h nh trang cho 
Di¹n  n To¡n håc Chuy¶n · Sè håc
2.4. Phö löc: B¤n n¶n bi¸t 27 
nhúng n«m håc ti¸p theo. 
Vîi i·u ki»n câ nhi·u h¤n ch¸ v· thíi gian, v· n«ng lüc tr¼nh ë n¶n 
trong khuæn khê · t i n y ph¥n chia d¤ng to¡n, lo¤i to¡n ch¿ câ t½nh 
t÷ìng èi. çng thíi công mîi ch¿ ÷a ra líi gi£i chù ch÷a câ ph÷ìng 
ph¡p, thuªt l m rã r ng. Tuy ¢ câ cè g­ng nhi·u nh÷ng chnsg tæi tü 
th§y trong · t i n y cán nhi·u h¤n ch¸. Chóng tæi r§t mong nhªn 
÷ñc nhúng þ ki¸n âng gâp cõa c¡c th¦y cæ gi¡o còng b¤n åc º to¡n 
håc thªt sü câ þ ngh¾a cao µp nh÷ c¥u ng¤n ngú Ph¡p ¢ vi¸t: 
To¡n håc l  Vua cõa c¡c khoa håc 
Sè håc l  Nú ho ng 
Chuy¶n · Sè håc Di¹n  n To¡n håc
Ch֓ng 
3 
B i to¡n chia h¸t 
3.1 Lþ thuy¸t cì b£n 29 
3.2 Ph÷ìng ph¡p gi£i c¡c b i to¡n chia 
h¸t 31 
Ph¤m Quang To n (Ph¤m Quang To n) 
Chia h¸t l  mët · t i quan trång trong ch÷ìng tr¼nh Sè håc cõa bªc 
THCS. i k±m theo â l  c¡c b i to¡n khâ v  hay. B i vi¸t n y xin 
giîi thi»u vîi b¤n åc nhúng ph÷ìng ph¡p gi£i c¡c b i to¡n chia h¸t: 
ph÷ìng ph¡p x²t sè d÷, ph÷ìng ph¡p quy n¤p, ph÷ìng ph¡p çng d÷, 
v.v... 
3.1 Lþ thuy¸t cì b£n 
3.1.1 ành ngh¾a v· chia h¸t 
ành ngh¾a 3.1 Cho hai sè nguy¶n a v  b trong â b6= 0, ta luæn t¼m 
÷ñc hai sè nguy¶n q v  r duy nh§t sao cho 
a = bq + r 
vîi 0  r  b. 
Trong â, ta nâi a l  sè bà chia, b l  sè chia, q l  th÷ìng, r l  sè d÷.4 
Nh÷ vªy, khi a chia cho b th¼ câ thº ÷a ra c¡c sè d÷ r 2 f0; 1; 2;    ; jbjg. 
°c bi»t, vîi r = 0 th¼ a = bq, khi â ta nâi a chia h¸t cho b (ho°c a l  
bëi cõa b, ho°c b l  ÷îc cõa a). Ta k½ hi»u b j a. Cán khi a khæng chia 
29
30 3.1. Lþ thuy¸t cì b£n 
h¸t cho b, ta k½ hi»u b - a. 
Sau ¥y l  mët sè t½nh ch§t th÷íng dòng, chùng minh ÷ñc suy ra trüc 
ti¸p tø ành ngh¾a. 
3.1.2 T½nh ch§t 
Sau ¥y xin giîi thi»u mët sè t½nh ch§t v· chia h¸t, vi»c chùng minh 
kh¡ l  d¹ d ng n¶n s³ d nh cho b¤n åc. Ta câ vîi a; b; c; d l  c¡c sè 
nguy¶n th¼: 
T½nh ch§t 3.1 N¸u a6= 0 th¼ a j a, 0 j a.  
T½nh ch§t 3.2 N¸u b j a th¼ b j ac.  
T½nh ch§t 3.3 N¸u b j a v  c j b th¼ c j a.  
T½nh ch§t 3.4 N¸u c j a v  c j b th¼ c j (ax  by) vîi x; y nguy¶n. 
T½nh ch§t 3.5 N¸u b j a v  a j b th¼ a = b ho°c a = b. 
T½nh ch§t 3.6 N¸u c j a v  d j b th¼ cd j ab. 
T½nh ch§t 3.7 N¸u b j a; c j a th¼ BCNN(b; c) j a. 
T½nh ch§t 3.8 N¸u c j ab v  UCLN(b; c) = 1 th¼ c j a. 
T½nh ch§t 3.9 N¸u p j ab, p l  sè nguy¶n tè th¼ p j a ho°c p j b.  
Tø t½nh ch§t tr¶n ta suy ra h» qu£ 
H» qu£ 3.1 N¸u p j an vîi p l  sè nguy¶n tè, n nguy¶n d÷ìng th¼ 
pn j an.  
Di¹n  n To¡n håc Chuy¶n · Sè håc
3.2. Ph÷ìng ph¡p gi£i c¡c b i to¡n chia h¸t 31 
3.1.3 Mët sè d§u hi»u chia h¸t 
Ta °t N = anan1 : : : a1a0 
D§u hi»u chia h¸t cho 2; 5; 4; 25; 8; 125 
2 j N , 2 j a0 , a0 2 f0; 2; 4; 6; 8g 
5 j N , 5 j a0 , a0 2 f0; 5g 
4; 25 j N , 4; 25 j a1a0 
8; 125 j N , 8; 125 j a2a1a0 
D§u hi»u chia h¸t cho 3 v  9 
3; 9 j N , 3; 9 j (a0 + a1 +    + an1 + an) 
Mët sè d§u hi»u chia h¸t kh¡c 
11 j N , 11 j [(a0 + a2 +    )  (a1 + a3 +    )] 
101 j N , 101 j [(a1a0 + a5a4 +    )  (a3a2 + a7a6 +    )] 
7; 13 j N , 7; 37 j [(a2a1a0 + a8a7a6 +    )  (a5a4a3 + a11a10a9 +    )] 
37 j N , 37 j (a2a1a0 + a5a4a3 +    + anan1an2) 
19 j N , 19 j 
 
an + 2an1 + 22an2 +    + 2na0 
 
3.2 Ph÷ìng ph¡p gi£i c¡c b i to¡n chia h¸t 
3.2.1 p döng ành lþ Fermat nhä v  c¡c t½nh ch§t cõa chia 
h¸t 
ành lþ Fermat nhä 
ành lþ 3.1 (ành lþ Fermat nhä) Vîi måi sè nguy¶n a v  sè 
nguy¶n tè p th¼ ap  p (mod p).  
Chùng minh. 1. N¸u p j a th¼ p j (a5  a). 
2. N¸u p - a th¼ 2a; 3a; 4a;    ; (p  1)a công khæng chia h¸t cho p. 
Gåi r1; r2;    ; rp1 l¦n l÷ñt l  sè d÷ khi chia a; 2a; 3a;    ; (p1)a 
cho p. th¼ chóng s³ thuëc tªp f1; 2; 3;    ; p1g v  æi mët kh¡c 
nhau (v¼ ch¯ng h¤n n¸u r1 = r3 th¼ p j (3a  a) hay p j 2a, 
Chuy¶n · Sè håc Di¹n  n To¡n håc
32 3.2. Ph÷ìng ph¡p gi£i c¡c b i to¡n chia h¸t 
ch¿ câ thº l  p = 2, m  p = 2 th¼ b i to¡n khæng óng). Do â 
r1r2  rp1 = 1  2  3    (p  1). Ta câ 
a  r1 (mod p) 
2a  r2 (mod p) 
   
(p  1)a  rp1 (mod p) 
Nh¥n v¸ theo v¸ ta suy ra 
123    (p1)ap1  r1r2    rp1 (mod p) ) ap1  1 (mod p) 
V¼ UCLN(a; p) = 1 n¶n ap  a (mod p). 
Nh÷ vªy vîi måi sè nguy¶n a v  sè nguy¶n tè p th¼ ap  a (mod p). 
Nhªn x²t. Ta câ thº chùng minh ành lþ b¬ng quy n¤p. Ngo i ra, ành 
lþ cán ÷ñc ph¡t biºu d÷îi d¤ng sau: 
ành lþ 3.2 Vîi måi sè nguy¶n a, p l  sè nguy¶n tè, UCLN(a; p) = 
1 th¼ ap1  1 (mod p).  
Ph÷ìng ph¡p sû döng t½nh ch§t chia h¸t v  ¡p döng ành lþ 
Fermat nhä 
Cì sð: Sû döng c¡c t½nh ch§t chia h¸t v  ành lþ Fermat nhä º gi£i 
to¡n. 
V½ dö 3.1. Cho a v  b l  hai sè tü nhi¶n. Chùng minh r¬ng 5a2+15ab 
b2 chia h¸t cho 49 khi v  ch¿ khi 3a + b chia h¸t cho 7. 4 
Líi gi£i. )) Gi£ sû 49 j 5a2 + 15ab  b2 ) 7 j 5a2 + 15ab  b2 ) 7 j 
(14a2 + 21ab)  (5a2 + 15ab  b2) ) 7 j (9a2 + 6ab + b2) ) 7 j 
(3a + b)2 ) 7 j 3a + b. 
() Gi£ sû 7 j 3a+b. °t 3a+b = 7c (c 2 Z. Khi â b = 7c3a. Nh÷ 
vªy 
) 5a2 + 15ab  b2 = 5a2 + 15a(7c  3a)  (7c  3a)2 
= 49(c2 + 3ac  a2) 
Di¹n  n To¡n håc Chuy¶n · Sè håc
3.2. Ph÷ìng ph¡p gi£i c¡c b i to¡n chia h¸t 33 
chia h¸t cho 49. 
Vªy 5a2 + 15ab  b2 chia h¸t cho 49 khi v  ch¿ khi 3a + b chia h¸t cho 
7.  
V½ dö 3.2. Cho 11 j (16a + 17b)(17a + 16b) vîi a; b l  hai sè nguy¶n. 
Chùng minh r¬ng 121 j (16a + 17b)(17a + 16b). 4 
Líi gi£i. Ta câ theo ¦u b i, v¼ 11 nguy¶n tè n¶n ½t nh§t mët trong 
hai sè 16a + 17b v  17a + 16b chia h¸t cho 11. Ta l¤i câ (16a + 17b) + 
(17a + 16b) = 33(a + b) chia h¸t cho 11. Do â n¸u mët trong hai sè 
16a + 17b v  17a + 16b chia h¸t cho 11 th¼ sè cán l¤i công chia h¸t cho 
11. Cho n¶n 121 j (16a + 17b)(17a + 16b).  
V½ dö 3.3. Chùng minh r¬ng A = 130 + 230 +  + 1130 khæng chia h¸t 
cho 11. 4 
Líi gi£i. Vîi måi a = 1; 2;    ; 10 th¼ (a; 10) = 1. Do â theo ành lþ 
Fermat b² th¼ a10  1 (mod 11) ) a30  1 (mod 11) vîi måi a = 
1; 2;    ; 10 v  1130  0 (mod 11). Nh÷ vªy 
A  |1 + 1 +{z   + 1} 
10 sè 1 
+0 (mod 11) 
 10 (mod 11) ) 11 - A 
V½ dö 3.4. Cho p v  q l  hai sè nguy¶n tè ph¥n bi»t. Chùng minh r¬ng 
pq1 + qp1  1 chia h¸t cho pq. 4 
Líi gi£i. V¼ q nguy¶n tè n¶n theo ành lþ Fermat nhä th¼ 
pq1  1 (mod q) 
Do â 
pq1 + qp1  1 (mod q) 
V¼ q v  p câ vai trá b¼nh ¯ng n¶n ta công d¹ d ng suy ra 
qp1 + pq1  1 (mod p): 
Cuèi còng v¼ UCLN(q; p) = 1 n¶n pq1 + qp1  1 (mod pq) hay 
pq1 + qp1  1 chia h¸t cho pq.  
Chuy¶n · Sè håc Di¹n  n To¡n håc
34 3.2. Ph÷ìng ph¡p gi£i c¡c b i to¡n chia h¸t 
B i tªp · nghà 
B i 1. Chùng minh r¬ng 11a+2b chia h¸t cho 19 khi v  ch¿ khi 18a+5b 
chia h¸t cho 19 vîi a; b l  c¡c sè nguy¶n. 
B i 2. Chùng minh r¬ng 2a + 7 chia h¸t cho 7 khi v  ch¿ khi 3a2 + 
10ab  8b2. 
B i 3. Cho p l  sè nguy¶n tè lîn hìn 5. Chùng minh r¬ng n¸u n l  sè 
tü nhi¶n câ p1 chú sè v  c¡c chú sè â ·u b¬ng 1 th¼ n chia 
h¸t cho p. 
B i 4. Gi£ sû n 2 N; n  2. X²t c¡c sè tü nhi¶n an = 11  1 ÷ñc vi¸t 
bði n chú sè 1. Chùng minh r¬ng n¸u an l  mët sè nguy¶n tè 
th¼ n l  ÷îc cõa an  1. 
B i 5. Gi£ sû a v  b l  c¡c sè nguy¶n d÷ìng sao cho 2a  1; 2b  1 v  
a + b ·u l  sè nguy¶n tè. Chùng minh r¬ng ab + ba v  aa + bb 
·u khæng chia h¸t cho a + b. 
B i 6. Chùng minh r¬ng vîi måi sè nguy¶n tè p th¼ tçn t¤i sè nguy¶n 
n sao cho 2n + 3n + 6n  1 chia h¸t cho p. 
3.2.2 X²t sè d÷ 
Cì sð: º chùng minh A(n) chia h¸t cho p, ta x²t c¡c sè n d¤ng 
n = kp + r vîi r 2 f0; 1; 2;    ; p  1g. 
Ch¯ng h¤n, vîi p = 5 th¼ sè nguy¶n n câ thº vi¸t l¤i th nh 5k; 5k + 
1; 5k + 2; 5k + 3; 5k + 4. Ta th¸ méi d¤ng n y v o c¡c và tr½ cõa n rçi 
lþ luªn ra ¡p sè. Sau ¥y l  mët sè v½ dö 
V½ dö 3.5. T¼m k 2 N º tçn t¤i n 2 N sao cho 
4 j n2  k 
vîi k 2 f0; 1; 2; 3g. 4 
Líi gi£i. Gi£ sû tçn t¤i k 2 N º tçn t¤i n 2 N thäa m¢n 4 j n2  k. 
Ta x²t c¡c Tr÷íng hñp: (m 2 N) 
Di¹n  n To¡n håc Chuy¶n · Sè håc
3.2. Ph÷ìng ph¡p gi£i c¡c b i to¡n chia h¸t 35 
1. N¸u n = 4m th¼ n2  k = 16m2  k chia h¸t cho 4 khi v  ch¿ khi 
4 j k n¶n k = 0: 
2. N¸u n = 4m  1 th¼ n2  k = 16m2  8m + 1  k chia h¸t cho 4 
khi v  ch¿ khi 4 j 1  k n¶n k = 1. 
3. N¸u n = 4m 2 th¼ n2  k = 16m2  16m+ 4  k chia h¸t cho 4 
khi v  ch¿ khi 4 j k n¶n k = 0. 
Vªy k = 0 ho°c k = 1.  
V½ dö 3.6. Chùng minh r¬ng vîi måi n 2 N th¼ 6 j n(2n+7)(7n+1).4 
Líi gi£i. Ta th§y mët trong hai sè n v  7n + 1 l  sè ch®n 8n 2 N. Do 
â 2 j n(2n+7)(7n+1). Ta s³ chùng minh 3 j n(2n+7)(7n+1). Thªt 
vªy, x²t 
1. Vîi n = 3k th¼ 3 j n(2n + 7)(7n + 1). 
2. Vîi n = 3k + 1 th¼ 2n + 7 = 6k + 9 chia h¸t cho 3 n¶n 3 j 
n(2n + 7)(7n + 1). 
3. Vîi n = 3k + 2 th¼ 7n + 1 = 21k + 15 chia h¸t cho 3 n¶n 3 j 
n(2n + 7)(7n + 1). 
Do â 3 j n(2n+7)(7n+1) m  (2; 3) = 1 n¶n 6 j n(2n+7)(7n+1) 8n 2 
N.  
V½ dö 3.7. (HSG 9, Tp Hç Ch½ Minh, váng 2, 1995) Cho x; y; z l  c¡c 
sè nguy¶n thäa m¢n 
(x  y)(y  z)(z  x) = x + y + z (3.1) 
Chùng minh r¬ng 27 j (x + y + z). 4 
Líi gi£i. X²t hai tr÷íng hñp sau 
Chuy¶n · Sè håc Di¹n  n To¡n håc
36 3.2. Ph÷ìng ph¡p gi£i c¡c b i to¡n chia h¸t 
1. N¸u ba sè x; y; z chia h¸t cho 3 câ c¡c sè d÷ kh¡c nhau th¼ c¡c 
hi»u xy; yz; zx còng khæng chia h¸t cho 3. M  3 j (x+y+z) 
n¶n tø (3.1) suy ra væ l½ . 
2. N¸u ba sè x; y; z ch¿ câ hai sè chia cho 3 câ còng sè d÷ th¼ trong ba 
hi»u xy; yz; zx câ mët hi»u chia h¸t cho 3. M  3 - (x+y+z) 
n¶n tø (3.1) suy ra væ l½. 
Vªy x; y; z chia cho 3 câ còng sè d÷, khi â x  y; y  z; z  x ·u chia 
h¸t cho 3. Tø (3.1) ta suy ra 27 j (x + y + z), ta câ pcm.  
B i tªp · nghà 
B i 1. i) T¼m sè tü nhi¶n n º 7 j (2n  1). 
ii) Chùng minh r¬ng 7 - (2n + 1) 8n 2 N. 
B i 2. Chùng minh r¬ng vîi måi sè nguy¶n a th¼ a(a6  1) chia h¸t 
cho 7. 
B i 3. T¼m n º 13 j 32n + 3n + 1. 
B i 4. Chùng minh r¬ng vîi måi a; b 2 N th¼ ab(a2b2)(4a2b2) luæn 
luæn chia h¸t cho 5. 
B i 5. Chùng minh r¬ng 24 j (p  1)(p + 1) vîi p l  sè nguy¶n tè lîn 
hìn 3. 
B i 6. Chùng minh r¬ng khæng tçn t¤i sè nguy¶n a º a2 +1 chia h¸t 
cho 12. 
B i 7. Chùng minh r¬ng vîi måi sè nguy¶n x; y; z n¸u 6 j x+y+z th¼ 
6 j x3 + y3 + z3. 
B i 8. Cho ab = 20112012, vîi a; b 2 N. Häi têng a + b câ chia h¸t cho 
2012 hay khæng ? 
B i 9. Sè 3n+2003 trong â n l  sè nguy¶n d÷ìng câ chia h¸t cho 184 
khæng ? 
Di¹n  n To¡n håc Chuy¶n · Sè håc
3.2. Ph÷ìng ph¡p gi£i c¡c b i to¡n chia h¸t 37 
B i 10. Cho c¡c sè nguy¶n d÷ìng x; y; z thäa m¢n x2 +y2 = z2. Chùng 
minh r¬ng xyz chia h¸t cho 60. 
B i 11. Cho c¡c sè nguy¶n d÷ìng x; y; z thäa m¢n x2+y2 = 2z2. Chùng 
minh r¬ng x2  y2 chia h¸t cho 84: 
B i 12. Cho n  3; (n 2 N). Chùng minh r¬ng n¸u 2n = 10a+b; (0  
b  9) th¼ 6 j ab. 
3.2.3 Ph¥n t½ch 
Ph¥n t½ch th nh t½ch 
Cì sð: º chùng minh A(n) chia h¸t cho p, ta ph¥n t½ch A(n) = D(n)p, 
cán n¸u trong ta khæng thº ÷a ra c¡ch ph¥n t½ch nh÷ vªy, ta câ thº 
vi¸t p = kq. 
 N¸u (k; q) = 1 th¼ ta chùng minh A(n) còng chia h¸t cho k v  q. 
 N¸u (k; q)6= 1 th¼ ta vi¸t A(n) = B(n)C(n) v  chùng minh B(n) 
chia h¸t cho k, C(n) chia h¸t cho q. 
V½ dö 3.8. Cho n l  mët sè nguy¶n d÷ìng. Chùng minh r¬ng 
2n j (n + 1) (n + 2)    (2n) : 
Líi gi£i. Ta câ 
(n + 1) (n + 2)    (2n) = 
(2n)! 
n! 
= 
(1:3:5:::(2n  1)) (2:4:6:::2n) 
n! 
= 1:3:5:::(2n  1):2n: 
n! 
n! 
= 1:3:5:::(2n  1):2n: 
Do â 2n j (n + 1) (n + 2)    (2n) :  
Chuy¶n · Sè håc Di¹n  n To¡n håc
38 3.2. Ph÷ìng ph¡p gi£i c¡c b i to¡n chia h¸t 
V½ dö 3.9. Chùng minh r¬ng vîi måi sè nguy¶n n th¼ 6 j n3  n. 4 
Líi gi£i. Ph¥n t½ch 
n3  n = n(n2  1) = n(n  1)(n + 1) 
Biºu thùc l  t½ch ba sè nguy¶n li¶n ti¸p n¶n tçn t¤i ½t nh§t mët trong 
ba sè mët sè chia h¸t cho 2 v  mët sè chia h¸t cho 3. M  (2; 3) = 1 n¶n 
6 j n3  n.  
V½ dö 3.10. Chùng minh r¬ng n6  n4  n2 + 1 chia h¸t cho 128 vîi n 
l´. 4 
Líi gi£i. Ta câ 
n6  n4  n2 + 1 = (n2  1)2(n + 1) = (n  1)2(n + 1)2 
V¼ n l´ n¶n °t n = 2k; k 2 N, suy ra 
(n2  1)2 = 
 
(2k + 1)2  1 
 
= (4k2 + 4k)2 = [4k(k + 1)]2 
Vªy 64 j (n2  1)2. V¼ n l´ n¶n 2 j n + 1, suy ra pcm.  
V½ dö 3.11. Cho ba sè nguy¶n d÷ìng kh¡c nhau x; y; z. Chùng minh 
r¬ng (xy)5+(yz)5+(xz)5 chia h¸t cho 5(xy)(yz)(xz).4 
Líi gi£i. Ta câ 
(x  y)5 + (y  z)5 + (x  z)5 
= (x  z + z  y)5 + (y  z)5 + (z  x)5 
= (x  z)5 + 5(x  z)4(z  y) + 10(x  z)3(z  y)2 
+10(x  z)4(z  y) + 10(x  z)3(z  y)2 
+10(x  z)2(z  y)3 + 5(x  z)(z  y)4 
= 5(x  z)(z  y) 
 
 
(x  z)3 + 2(x  z)2(z  y) + 2(x  z)(z  y)2 + (z  y)3 
 
: 
Di¹n  n To¡n håc Chuy¶n · Sè håc
3.2. Ph÷ìng ph¡p gi£i c¡c b i to¡n chia h¸t 39 
Nh÷ng ta công câ: 
(x  z)3 + 2(x  z)2(z  y) + 2(x  z)(z  y)2 + (z  y)3 
= (x  y + y  z)3 + 2(x  y + y  z)2(z  y) 
+2(x  y + y  z)(z  y)2 + (z  y)3 
= (x  y)3 + 2(x  y)2(y  z) + 3(x  y)(y  z)2 
+(y  z)3 + 2(x  y)2(z  y) 
+4(x  y)(y  z)(z  y) + 2(y  z)2(z  y) 
+2(x  y)(z  y)2 + 2(y  z)(z  y)2 + (z  y)3 
= (x  y)3 + 3(x  y)2(y  z) + 3(x  y)(y  z)2 
+2(x  y)2(z  y) + 4(x  y)(y  z)(z  y) + 2(x  y)(z  y)2; 
Biºu thùc cuèi còng câ nh¥n tû chung (x  y): Ta suy ra i·u ph£i 
chùng minh.  
B i tªp · nghà 
B i 1. Chùng minh r¬ng n¸u a; k l  c¡c sè nguy¶n, a l´ th¼ 2k+1 j 
(a2k 
 1). 
B i 2. Chùng minh r¬ng n5  n chia h¸t cho 30 vîi måi n 2 Z. 
B i 3. Chùng minh r¬ng 3n4  14n3 + 21n2  10n chia h¸t cho 24 vîi 
måi n 2 Z. 
B i 4. Chùng minh r¬ng n55n3+4n chia h¸t cho 120 vîi måi n 2 Z. 
B i 5. Chùng minh r¬ng n3  3n2  n + 3 chia h¸t cho 48 vîi måi n 
l´, n 2 Z. 
B i 6. Chùng minh r¬ng n8  n6  n4 + n2 chia h¸t cho 1152 vîi måi 
sè nguy¶n n l´. 
B i 7. Chùng minh r¬ng n44n34n2+16n chia h¸t cho 348 vîi måi 
n l  sè nguy¶n ch®n. 
B i 8. Chùng minh r¬ng n4  14n3 + 71n2  154n + 120 chia h¸t cho 
24 vîi måi sè tü nhi¶n n. 
Chuy¶n · Sè håc Di¹n  n To¡n håc
40 3.2. Ph÷ìng ph¡p gi£i c¡c b i to¡n chia h¸t 
B i 9. Cho x; y; z l  c¡c sè nguy¶n kh¡c 0. Chùng minh r¬ng n¸u 
x2  yz = a; y2  zx = b; z2  xy = c th¼ têng (ax + by + cz) 
chia h¸t cho têng (a + b + c). 
B i 10. Cho m; n l  hai sè ch½nh ph÷ìng l´ li¶n ti¸p. Chùng minh r¬ng 
mn  m  n + 1 chia h¸t cho 192. 
B i 11. (HSG 9 TQ 1970) Chùng minh r¬ng n12 n8 n4 +1 chia h¸t 
cho 512 vîi måi sè tü nhi¶n n l´. 
B i 12. (HSG 9 TQ 1975) Chùng minh r¬ng n4 +6n3 +11n2 +6n chia 
h¸t cho 24 vîi måi sè nguy¶n d÷ìng n. 
T¡ch têng 
Cì sð: º chùng minh A(n) chia h¸t cho p, ta bi¸n êi A(n) th nh 
têng nhi·u h¤ng tû rçi chùng minh méi h¤ng tû ·u chia h¸t cho p. 
Ta câ thº sû döng mët sè h¬ng ¯ng thùc ¡p döng v o chia h¸t, v½ dö 
nh÷: 
Cho a; b l  c¡c sè thüc v  n l  sè nguy¶n d÷ìng. Khi â ta 
câ 
an  bn = (a  b)(an1 + an2b +    + abn2 + bn1) 
Ta s³ câ h» qu£ l : 
H» qu£ 3.2 N¸u a  b6= 0 th¼ an  bn chia h¸t cho a  b.  
H» qu£ 3.3 N¸u a + b6= 0 v  n l´ th¼ an + bn chia h¸t cho a + b.  
H» qu£ 3.4 N¸u a+b6= 0 v  n ch®n th¼ an bn chia h¸t cho a+b 
V½ dö 3.12. Chùng minh r¬ng ax2 +bx+c 2 Z; 8x 2 Z khi v  ch¿ khi 
2a; a + b; c 2 Z 4 
Di¹n  n To¡n håc Chuy¶n · Sè håc
3.2. Ph÷ìng ph¡p gi£i c¡c b i to¡n chia h¸t 41 
Líi gi£i. Ph¥n t½ch 
ax2 + bx + c = ax2  ax + (a + b)x + c 
= 2a: 
x(x  1) 
2 
+ (a + b)x + c 2 Z; 8x 2 Z: 
V½ dö 3.13. Chùng minh r¬ng 6 j (a3 + 5a) 8a 2 N. 4 
Líi gi£i. Ph¥n t½ch a3+5a = (a3a)+6a. Hiºn nhi¶n óng v¼ 6 j n3n 
(chùng minh ð v½ dö Equation 4.27).  
Nhªn x²t. Tø v½ dö Equation 4.27 ta công câ thº ÷a ra c¡c b i to¡n 
sau, chùng minh công b¬ng c¡ch vªn döng ph÷ìng ph¡p t¡ch têng: 
B i to¡n 3.1. Cho m; n 2 Z. Chùng minh r¬ng 6 j m2n2(m  n). 4 
B i to¡n 3.2. Cho a; b; c 2 Z. Chùng minh r¬ng 6 j (a3 + b3 + c3) khi 
v  ch¿ khi 6 j (a + b + c) 4 
B i to¡n 3.3. Cho a 2 Z. Chùng minh r¬ng a 
3 
+ 
a2 
2 
+ 
a3 
6 
2 Z 4 
B i to¡n 3.4. Vi¸t sè 20112012 th nh têng c¡c sè nguy¶n d÷ìng. em 
têng lªp ph÷ìng t§t c£ c¡c sè h¤ng â chia cho 3 th¼ ÷ñc d÷ l  bao 
nhi¶u ? 4 
V½ dö 3.14. Cho m; n l  c¡c sè nguy¶n thäa m¢n: 
m 
n 
= 1  
1 
2 
+ 
1 
3 
 
1 
4 
+     
1 
1334 
+ 
1 
1335 
Chùng minh r¬ng 2003 j m. 4 
Chuy¶n · Sè håc Di¹n  n To¡n håc
42 3.2. Ph÷ìng ph¡p gi£i c¡c b i to¡n chia h¸t 
Líi gi£i. º þ r¬ng 2003 l  sè nguy¶n tè. Ta câ 
m 
1 
1 
1 
1 
1 
= 1  
+ 
 
+     
+ 
n 
2 
3 
4 
1334 
1335 
= 
 
1 + 
1 
2 
+ 
1 
3 
+    + 
1 
1335 
 
 2 
 
1 
2 
+ 
1 
4 
+ 
1 
6 
+    + 
1 
1334 
 
= 
 
1 + 
1 
2 
+ 
1 
3 
+    + 
1 
1335 
 
 
 
1 + 
1 
2 
+ 
1 
3 
+    + 
1 
667 
 
= 
1 
668 
+ 
1 
669 
+    + 
1 
1335 
= 
 
1 
668 
+ 
1 
1335 
 
+ 
 
1 
669 
+ 
1 
1334 
 
+    + 
 
1 
1001 
+ 
1 
1002 
 
= 2003 
 
1 
668:1335 
+ 
1 
669:1334 
+    + 
1 
1001:1002 
 
= 2003: 
p 
q 
Ð ¥y p l  sè nguy¶n cán q = 668  669    1335. V¼ 2003 nguy¶n tè n¶n 
(q; 2003) = 1. 
Do â tø () suy ra 2003pn = mq: 
V¼ p; n nguy¶n n¶n suy ra 2003jmq m  (q; 2003) = 1 n¶n 2003jm.  
V½ dö 3.15. Chùng minh r¬ng vîi måi sè tü nhi¶n n th¼ A = 2005n + 
60n  1897n  168n chia h¸t cho 2004. 4 
Líi gi£i. Ta câ 2004 = 12  167. V¼ (12; 167) = 1 n¶n º chùng minh 
A chia h¸t cho 2004 ta chùng minh A chia h¸t cho 12 v  167. 
p döng t½nh ch§t an  bn chia h¸t cho a  b vîi måi n tü nhi¶n v  
ab6= 0 suy ra 2005n1897n chia h¸t cho 20051897 = 108 = 129, 
hay 2005n  1897n chia h¸t cho 12. T÷ìng tü th¼ 168n  60n chia h¸t 
cho 12. Vªy A chia h¸t cho 12. 
Ti¸p töc ph¥n t½ch 
A = (2005n  168n)  (1897n  60n): 
Lªp luªn t÷ìng tü nh÷ tr¶n th¼ 2005n  168n v  1897n  60n chia h¸t 
cho 167, tùc A chia h¸t cho 167. Vªy ta câ i·u ph£i chùng minh.  
Di¹n  n To¡n håc Chuy¶n · Sè håc
3.2. Ph÷ìng ph¡p gi£i c¡c b i to¡n chia h¸t 43 
V½ dö 3.16. (· thi tuyºn sinh HKHTN-HQG H  Nëi, váng 1, n«m 
2007-2008) Cho a; b l  hai sè nguy¶n d÷ìng v  a+1; b+2007 ·u chia 
h¸t cho 6. Chùng minh r¬ng 4a + a + b chia h¸t cho 6. 4 
Líi gi£i. Ph¥n t½ch 
4a + a + b = (4a + 2) + (a + 1) + (b + 2007)  2010 
4a + 2 = 4a  1 + 3 = (4  1)(4a1 +    1) + 3 
Nh÷ vªy 3 j 4a + 2. Do â 4a + a + b l  têng cõa c¡c sè nguy¶n d÷ìng 
chia h¸t cho 6 n¶n 4a + a + b chia h¸t cho 6.  
B i tªp · nghà 
B i 1. ÷a ra c¡c mð rëng tø b i tªp · nghà cõa ph÷ìng ph¡p ph¥n 
t½ch th nh t½ch th nh c¡c b i to¡n vªn döng ph÷ìng ph¡p t¡ch 
têng (gièng nh÷ c¡ch mð rëng cõa v½ dö 1.9). 
B i 2. (Hungary MO 1947) Chùng minh r¬ng 46n + 296:13n chia h¸t 
cho 1947 vîi måi sè tü nhi¶n n l´. 
B i 3. Chùng minh r¬ng 20n + 16n  3n  1 chia h¸t cho 323 vîi måi 
sè tü nhi¶n n ch®n. 
B i 4. Chùng minh r¬ng 2903n803n464n+261n chia h¸t cho 1897 
vîi måi sè tü nhi¶n n. 
B i 5. Chùng minh r¬ng vîi måi sè nguy¶n n  1 ta câ nn + 5n2  
11n + 5 chia h¸t cho (n  1)2. 
B i 6. (HSG 9 Tp H  Nëi, váng 2, 1998) Chùng minh r¬ng 1997 j m 
vîi m; n 2 N thäa m¢n 
m 
n 
= 1  
1 
2 
+ 
1 
3 
 
1 
4 
+    + 
1 
1329 
 
1 
1330 
+ 
1 
1331 
: 
B i 7. Chùng minh r¬ng 32n+1 + 2n+2 chia h¸t cho 7 vîi måi n 2 N: 
Chuy¶n · Sè håc Di¹n  n To¡n håc
44 3.2. Ph÷ìng ph¡p gi£i c¡c b i to¡n chia h¸t 
B i 8. Chùng minh r¬ng 20032005 + 20172015 chia h¸t cho 12. 
B i 9. Cho p l  sè tü nhi¶n l´ v  c¡c sè nguy¶n a; b; c; d; e thäa m¢n 
a + b + c + d + e v  a2 + b2 + c2 + d2 + e2 ·u chia h¸t cho p. 
Chùng minh r¬ng sè a5 + b5 + c5 + d5 + e5  5abcde công chia 
h¸t cho p. 
B i 10. (Canada Training for IMO 1987) 
K½ hi»u: 
1  3  5    (2n  1) = (2n  1)!! 
2  4  6    (2n) = (2n)!!: 
Chùng minh r¬ng (1985)!! + (1986)!! chia h¸t cho 1987: 
B i 11. Chùng minh r¬ng sè 22225555 + 55552222 chia h¸t cho 7. 
B i 12. Cho k l  sè nguy¶n d÷ìng sao cho sè p = 3k + 1 l  sè nguy¶n 
tè v  
1 
1  2 
+ 
1 
3  4 
+    + 
1 
(2k  1)2k 
= 
m 
n 
vîi hai sè nguy¶n d÷ìng nguy¶n tè còng nhau m v  n.Chùng 
minh m chia h¸t cho p. 
(T¤p ch½ Mathematics Reflections, «ng bði T.Andreescu) 
3.2.4 X²t çng d÷ 
ành ngh¾a v  mët sè t½nh ch§t 
ành ngh¾a 3.2 Cho a; b l  c¡c sè nguy¶n v  n l  sè nguy¶n d÷ìng. Ta 
nâi, a çng d÷ vîi b theo modun n v  k½ hi»u a  b (mod n) n¸u a v  
b câ còng sè d÷ khi chia cho n. 4 
Nh÷ vªy a  n (mod n) () n j (a  b). V½ dö: 2012  2 (mod 5). 
T½nh ch§t (b¤n åc tü chùng minh) 
Cho a; b; c; d; n l  c¡c sè nguy¶n. 
T½nh ch§t 3.10 
a  a (mod n); 
a  b (mod n) , b  a (mod n); 
a  b (mod n); b  c (mod n) ) a  c (mod n): 
 
Di¹n  n To¡n håc Chuy¶n · Sè håc
3.2. Ph÷ìng ph¡p gi£i c¡c b i to¡n chia h¸t 45 
T½nh ch§t 3.11 
( 
a  b (mod n) 
c  d (mod n) 
) 
( 
a  c  b  d (mod n) 
ac  bd (mod n) 
 
T½nh ch§t 3.12 a  b (mod n) ) ak  bk (mod n); 8k  1:  
T½nh ch§t 3.13 a  b (mod n) ) ac  bc (mod mc); c  0  
T½nh ch§t 3.14 (a + b)n  bn (mod a); (a  0):  
T½nh ch§t 3.15 N¸u d l  ÷îc chung d÷ìng cõa a; b v  m th¼ a  b 
(mod m) th¼ 
a 
d 
 
b 
d 
(mod 
m 
d 
). 
T½nh ch§t 3.16 a  b (mod m), c l  ÷îc chung cõa a v  b, (c;m) = 1 
th¼ 
a 
c 
 
b 
c 
(mod m). 
Ph÷ìng ph¡p çng d÷ thùc º gi£i c¡c b i to¡n chia h¸t 
Cì sð: Sû döng c¡c t½nh ch§t v  ành ngh¾a tr¶n º gi£i c¡c b i to¡n 
chia h¸t. 
V½ dö 3.17. Chùng minh r¬ng vîi måi sè tü nhi¶n n th¼ 7 j 8n + 6. 4 
Líi gi£i. Ta câ 8n  1 (mod 7) =) 8n + 6  7  0 (mod 7):  
V½ dö 3.18. Chùng minh r¬ng 19 j 7  52n + 12  6n: vîi måi sè nguy¶n 
d֓ng n. 4 
Líi gi£i. Ta câ 52 = 25  6 (mod 19) =) (52)n  6n (mod 19) =) 
7  52n  7  6n (mod 19) =) 7  52n +12  6n  19  6n  0 (mod 19): 
V½ dö 3.19. Vi¸t li¶n ti¸p c¡c sè 111; 112;    ; 888 º ÷ñc sè A = 
111112    888. Chùng minh r¬ng 1998 j A. 4 
Chuy¶n · Sè håc Di¹n  n To¡n håc
46 3.2. Ph÷ìng ph¡p gi£i c¡c b i to¡n chia h¸t 
Líi gi£i. Ta th§y A ch®n n¶n 2 j A. M°t kh¡c 
A = 111  1000777 + 112  1000776 +    + 888: 
Do 1000k  1 (mod 999); 8k 2 N n¶n 
A  111 + 112 +    + 888  0 (mod 999): 
Suy ra 999 j A, v  (999; 2) = 1 n¶n 1998 j A:  
V½ dö 3.20. Chùng minh r¬ng 7 j 55552222 + 22225555. 4 
Líi gi£i. Ta câ 
2222  4 (mod 7) =) 22225555  (4)5555 (mod 7) 
5555  4 (mod 7) =) 55552222  4 (mod 7) 
=) 55552222 + 22225555  45555 + 42222 (mod 7) 
L¤i câ 
45555 + 42222 = 42222  
43333  1 
 
= 42222  
641111  1 
 
V  64  1 (mod 7) =) 641111  1  0 (mod 7). 
Do â 7 j 55552222 + 22225555  
B i tªp · nghà 
B i 1. Mët sè b i tªp ð ph÷ìng ph¡p ph¥n t½ch câ thº gi£i b¬ng ph÷ìng 
ph¡p çng d÷ thùc. 
B i 2. Chùng minh r¬ng 333555777 
+ 777555333 chia h¸t cho 10. 
B i 3. Chùng minh r¬ng sè 11101967 
 1 chia h¸t cho 101968. 
B i 4. Cho 9 j a3 +b3 +c3; 8a; b; c 2 Z. Chùng minh r¬ng 3 j a  b  c. 
B i 5. Chùng minh r¬ng 222333 + 333222 chia h¸t cho 13. 
Di¹n  n To¡n håc Chuy¶n · Sè håc
3.2. Ph÷ìng ph¡p gi£i c¡c b i to¡n chia h¸t 47 
B i 6. Chùng minh r¬ng 9n + 1 khæng chia h¸t cho 100; 8n 2 N: 
B i 7. Chùng minh r¬ng vîi måi sè nguy¶n khæng ¥m n th¼ 25n+3 + 
5n  3n+1 chia h¸t cho 17. 
B i 8. T¼m n 2 N sao cho 2n3 + 3n = 19851986. 
B i 9. Vi¸t li¶n ti¸p 2000 sè 1999 ta ÷ñc sè X = 19991999    1999: 
T¼m sè d÷ trong ph²p chia X cho 10001. 
B i 10. Chùng minh r¬ng 100 j 77777 
 777 . 
B i 11. Cho b2  4ac v  b2 + 4ac l  hai sè ch½nh ph÷ìng vîi a; b; c 2 N. 
Chùng minh r¬ng 30 j abc. 
3.2.5 Quy n¤p 
Cì sð : º chùng minh m»nh · óng vîi måi sè tü nhi¶n n  p, ta 
l m nh÷ sau: 
 Kiºm tra m»nh · óng vîi n = p. 
 Gi£ sû m»nh · óng vîi n = k. Ta i chùng minh m»nh · công 
óng vîi n = k + 1. 
V½ dö 3.21. Chùng minh r¬ng A = 4n + 15  1 chia h¸t cho 9 vîi måi 
n 2 N. 4 
Líi gi£i. Vîi n = 1 =) A = 18 chia h¸t cho 9. 
Gi£ sû b i to¡n óng vîi n = k. Khi â 9 j 4k+15k1, hay 4k+15k1 = 
9q vîi q 2 N. Suy ra 4k = 9q  15k + 1. 
Ta i chùng minh b i to¡n óng vîi n = k+1, tùc 9 j 4k+1+15(k+1)1. 
Thªt vªy: 
4k+1 + 15(k + 1)  1 = 4  4k + 15k + 14 
= 4 (9q  15k + 1) + 15k + 14 
= 36q  45k + 18 
chia h¸t cho 9. Ta câ pcm.  
Chuy¶n · Sè håc Di¹n  n To¡n håc
48 3.2. Ph÷ìng ph¡p gi£i c¡c b i to¡n chia h¸t 
V½ dö 3.22. (HSG 9 TQ 1978)Chùng minh r¬ng sè ÷ñc t¤o bði 3n chú 
sè gièng nhau th¼ chia h¸t cho 3n vîi 1  n; n 2 N: 4 
Líi gi£i. Vîi n = 1, b i to¡n hiºn nhi¶n óng. 
Gi£ sû b i to¡n óng vîi n = k, tùc 3k j |aa{z  a} 
3n sè a 
: 
Vîi n = k + 1 ta câ: 
|aa{z  a} 
3k+1 
= |aa{z  a} 
3k 
|aa{z  a} 
3k 
|aa{z  a} 
3k 
= |aa{z  a} 
3k 
 1 |00{z  0} 
3k1 
|00{z  0} 
3k1 
1 
chia h¸t cho 3k+1. Ta câ pcm.  
V½ dö 3.23. Chùng minh r¬ng vîi måi n 2 N; k l  sè tü nhi¶n l´ th¼ 
2n+2 j k2n 
 1 
Líi gi£i. Vîi n = 1 th¼ k2n 
 1 = k2  1 = (k + 1)(k  1). Do k l´,n¶n 
°t k = 2m+ 1 vîi m 2 N, th¼ khi â (k + 1)(k  1) = 4k(k + 1) chia 
h¸t cho 23 = 8. 
Gi£ sû b i to¡n óng vîi n = p, tùc 2p+2 j k2p 
1 hay k2p 
= q  2p+2+1 
vîi q 2 N. 
Ta chùng minh b i to¡n óng vîi n = p + 1. Thªt vªy 
A = k2p+1 
 1 = k22p 
 1 = 
 
k2p2 
 1 
= 
 
k2p 
 1 
  
k2p 
+ 1 
 
= q  2p+2  
 
2 + q  2p+2 
= q  2p+3  
 
1 + q  2p+1 
chia h¸t cho 2p+3. Ta câ pcm.  
Di¹n  n To¡n håc Chuy¶n · Sè håc
3.2. Ph÷ìng ph¡p gi£i c¡c b i to¡n chia h¸t 49 
B i tªp · nghà 
B i 1. Mët sè b i to¡n ð c¡c ph÷ìng ph¡p n¶u tr¶n câ thº gi£i b¬ng 
ph÷ìng ph¡p quy n¤p. 
B i 2. Chùng minh r¬ng 255 j 16n  15n  1 vîi n 2 N. 
B i 3. Chùng minh r¬ng 64 j 32n+3 + 40n  27 vîi n 2 N. 
B i 4. Chùng minh r¬ng 16 j 32n+2 + 8n  9 vîi n 2 N. 
B i 5. Chùng minh r¬ng 676 j 33n+3  16n  27 vîi n 2 N; n  1. 
B i 6. Chùng minh r¬ng 700 j 292n  140n  1 vîi n 2 N. 
B i 7. Chùng minh r¬ng 270 j 2002n  138n  1 vîi n 2 N. 
B i 8. Chùng minh r¬ng 22 j 324n+1 
+ 234n+1 
+ 5 vîi n 2 N. 
B i 9. Chùng minh r¬ng sè 23n 
+ 1 chia h¸t cho 3n nh÷ng khæng chia 
h¸t cho 3n+1 vîi n 2 N. 
B i 10. Chùng minh r¬ng sè 20012n 
1 chia h¸t cho 2n+4 nh÷ng khæng 
chia h¸t cho 2n+5 vîi n 2 N. 
B i 11. Chùng minh r¬ng vîi måi sè tü nhi¶n n  2, tçn t¤i mët sè tü 
nhi¶n m sao cho 3n j (m3 + 17), nh÷ng 3n+1 - (m3 + 17). 
B i 12. Câ tçn t¤i hay khæng mët sè nguy¶n d÷ìng l  bëi cõa 2007 v  
câ bèn chú sè tªn còng l  2008. 
B i 13. Chùng minh r¬ng tçn t¤i mët sè câ 2011 chú sè gçm to n chú 
sè 1 v  2 sao cho sè â chia h¸t cho 22011. 
B i 14. T¼m ph¦n d÷ khi chia 32n cho 2n+3, trong â n l  sè nguy¶n 
d֓ng. 
B i 15. Cho n 2 N; n  2. °t A = 77::: 
(lôy thøa n l¦n). Chùng minh 
r¬ng An + 17 chia h¸t cho 20. 
Chuy¶n · Sè håc Di¹n  n To¡n håc
50 3.2. Ph÷ìng ph¡p gi£i c¡c b i to¡n chia h¸t 
3.2.6 Sû döng nguy¶n l½ Dirichlet 
Nëi dung: Nhèt 5 con thä v o 3 chuçng th¼ tçn t¤i chuçng chùa ½t nh§t 
2 con. 
ành lþ 3.3 Nhèt m = nk + 1 con thä v o k chuçng (k  n) th¼ tçn 
t¤i chuçng chùa ½t nh§t n + 1 con thä.  
Chùng minh. Gi£ sû khæng câ chuçng n o chùa ½t nh§t n+1 con thä, 
khi â méi chuçng chùa nhi·u nh§t n con thä, n¶n k chuçng chùa nhi·u 
nh§t kn con thä, m¥u thu¨n vîi sè thä l  nk + 1.  
ành lþ 3.4 (p döng v o sè håc) Trong m = nk + 1 sè câ ½t 
nh§t n + 1 sè chia cho k câ còng sè d÷.  
Tuy nguy¶n lþ ÷ñc ph¡t biºu kh¡ ìn gi£n nh÷ng l¤i câ nhúng ùng 
döng h¸t sùc b§t ngí, thó và. B i vi¸t n y ch¿ xin n¶u mët sè ùng döng 
cõa nguy¶n l½ trong vi»c gi£i c¡c b i to¡n v· chia h¸t. 
V½ dö 3.24. Chùng minh r¬ng luæn tçn t¤i sè câ d¤ng 
20112011    201100    0 
chia h¸t cho 2012. 4 
Líi gi£i. L§y 2013 sè câ d¤ng 
2011; 20112011;    ; |2011201{1z   2011} 
2012 sè 2011 
: 
L§y 2013 sè n y chia cho 2012. Theo nguy¶n l½ Dirichlet th¼ tçn t¤i hai 
sè câ còng sè d÷ khi chia cho 2012. 
Gi£ sû hai sè â l  |2011201{1z   2011} 
m sè 2011 
v  2|011201{1z   2011} 
n sè 2011 
(m  n  
0). 
=) 2012 j |2011201{1z   2011} 
m sè 2011 
2|011201{1z   2011} 
n sè 2011 
Di¹n  n To¡n håc Chuy¶n · Sè håc
3.2. Ph÷ìng ph¡p gi£i c¡c b i to¡n chia h¸t 51 
=) 2012 j |2011201{1z   2011} 
mn sè 2011 
|00 {z 00} 
n sè 2011 
Vªy tçn t¤i sè thäa m¢n · b i.  
V½ dö 3.25. Chùng minh r¬ng trong 101 sè nguy¶n b§t k¼ câ thº t¼m 
÷ñc hai sè câ 2 chú sè tªn còng gièng nhau. 4 
Líi gi£i. L§y 101 sè nguy¶n ¢ cho chia cho 100 th¼ theo nguy¶n l½ 
Dirichlet tçn t¤i hai sè câ còng sè d÷ khi chia cho 100. Suy ra trong 
101 sè nguy¶n ¢ cho tçn t¤i hai sè câ chú sè tªn còng gièng nhau.  
V½ dö 3.26 (Tuyºn sinh 10 chuy¶n HSPHN, 1993). Cho 5 sè nguy¶n 
ph¥n bi»t tòy þ a1; a2; a3; a4; a5. Chùng minh r¬ng t½ch 
P = (a1  a2)(a1  a3)(a1  a4)(a1  a5)(a2  a3) 
 (a2  a4)(a2  a5)(a3  a4)(a3  a5)(a4  a5) 
chia h¸t cho 288. 4 
Líi gi£i. Ph¥n t½ch 288 = 25  32. 
1. Chùng minh 9 j P: Theo nguy¶n l½ Dirichlet th¼ trong 4 sè 
a1; a2; a3 câ hai sè câ hi»u chia h¸t cho 3. Khæng m§t t½nh têng 
qu¡t, gi£ sû: 3 j a1  a2. X²t 4 sè a2; a3; a4; a5 công câ hai sè câ 
hi»u chia h¸t cho 3. Nh÷ vªy P câ ½t nh§t hai hi»u kh¡c nhau 
chia h¸t cho 3, tùc 9 j p. 
2. Chùng minh 32 j P: Theo nguy¶n l½ Dirichlet th¼ täng 5 sè ¢ cho 
tçn t¤i ½t nh§t 3 sè câ còng t½nh ch®n l´. Ch¿ câ thº câ hai kh£ 
n«ng sau x£y ra: 
 N¸u câ ½t nh§t 4 sè câ còng t½nh ch®n l´, th¼ tø bèn sè câ thº 
lªp th nh s¡u hi»u kh¡c nhau chia h¸t cho 2. Do â 32 j P. 
Chuy¶n · Sè håc Di¹n  n To¡n håc
52 3.2. Ph÷ìng ph¡p gi£i c¡c b i to¡n chia h¸t 
 N¸u câ 3 sè câ còng t½nh ch®n l´. Khæng m§t t½nh têng qu¡t, 
gi£ sû ba sè â l  a1; a2; a3. Khi â a4; a5 công còng t½nh 
ch®n l´ nh÷ng l¤i kh¡c t½nh ch®n l´ cõa a1; a2; a3. Khi â 
c¡c hi»u sau chia h¸t cho 2: a1 a2; a1 a3; a2 a3; a4 a5. 
M°t kh¡c, trong 5 sè ¢ cho câ ½t nh§t hai hi»u chia h¸t cho 
4, cho n¶n trong 4 hi»u a1  a2; a1  a3; a2  a3; a4  a5 câ 
½t nh§t mët hi»u chia h¸t cho 4. Vªy 32 j P. 
Ta câ pcm.  
V½ dö 3.27. Cho 2012 sè tü nhi¶n b§t k¼ a1; a2;    ; a2012. Chùng minh 
r¬ng tçn t¤i mët sè chia h¸t cho 2012 ho°c têng mët sè sè chia h¸t cho 
2012. 4 
Líi gi£i. X²t 2012 sè 
S1 = a2 
S2 = a1 + a2 
   
S2012 = a1 + a2 +    + a2012 
Tr÷íng hñp 1: N¸u tçn t¤i sè Si (i = 1; 2;    ; 2012) chia h¸t cho 
2012 th¼ b i to¡n chùng minh xong. 
Tr÷íng hñp 2: N¸u 2012 - Si vîi måi i = 1; 2;    ; 2012. em 2012 
sè n y chia cho 2012 nhªn ÷ñc 2012 sè d÷. C¡c sè d÷ nhªn gi¡ 
trà thuëc tªp f1; 2;    ; 2011g. V¼ câ 2012 sè d÷ m  ch¿ câ 2011 
gi¡ trà n¶n theo nguy¶n l½ Dirichlet ch­c ch­n câ hai sè d÷ b¬ng 
nhau. G¿a sû gåi hai sè â l  Sm v  Sn câ còng sè d÷ khi chia 
cho 2012 (m; n 2 N; 1  n  m  2012) th¼ hi»u 
Sm  Sn = an+1 + an+2 +    + am 
chia h¸t cho 2012.  
Di¹n  n To¡n håc Chuy¶n · Sè håc
3.2. Ph÷ìng ph¡p gi£i c¡c b i to¡n chia h¸t 53 
Nhªn x²t. Ta câ thº rót ra b i to¡n têng qu¡t v  b i to¡n mð rëng 
sau: 
B i to¡n 3.5 (B i to¡n têng qu¡t). Cho n sè a1; a2;    ; an. Chùng 
minh r¬ng trong n sè tr¶n tçn t¤i mët sè chia h¸t cho n ho°c têng mët 
sè sè chia h¸t cho n. 4 
B i to¡n 3.6 (B i to¡n mð rëng). (T¤p ch½ To¡n Tuêi Thì sè 115) 
Cho n l  mët sè chuy¶n d÷ìng v  n sè nguy¶n d÷ìng a1; a2;    ; an câ 
têng b¬ng 2n  1. Chùng minh r¬ng tçn t¤i mët sè sè trong n sè ¢ 
cho câ têng b¬ng n. 4 
B i tªp · nghà 
B i 1. Chùng minh r¬ng câ væ sè sè chia h¸t cho 201311356 m  trong 
biºu di¹n thªp ph¥n cõa c¡c sè â khæng câ c¡c chú sè 0; 1; 2; 3. 
B i 2. (HSG 9 H  Nëi, 2006) Chùng minh r¬ng tçn t¤i sè tü nhi¶n 
n6= 0 thäa m¢n 313579 j (13579n  1). 
B i 3. Chùng minh r¬ng trong 52 sè nguy¶n d÷ìng b§t k¼ luæn luæn 
t¼m ÷ñc hai sè câ têng ho°c hi»u chia h¸t cho 100. 
B i 4. Cho 10 sè nguy¶n d÷ìng a1; a2;    ; a10. Chùng minh r¬ng tçn 
t¤i c¡c sè ci 2 f0;1; 1g; (i = 1;    10) khæng çng thíi b¬ng 
0 sao cho 
A = c1a1 + c2a2 +    + c10a10 
chia h¸t cho 1032: 
B i 5. Chùng minh r¬ng tçn t¤i sè tü nhi¶n k sao cho 2002k  1 chia 
h¸t cho 200310. 
B i 6. Bi¸t r¬ng ba sè a; a+k; a+2k ·u l  c¡c sè nguy¶n tè lîn hìn 
3. Chùng minh r¬ng khi â k chia h¸t cho 6. 
Chuy¶n · Sè håc Di¹n  n To¡n håc
54 3.2. Ph÷ìng ph¡p gi£i c¡c b i to¡n chia h¸t 
3.2.7 Ph£n chùng 
Cì sð: º chùng minh p - A(n), ta l m nh÷ sau: 
 Gi£ sû ng÷ñc l¤i p j A(n). 
 Chùng minh i·u ng÷ñc l¤i sai. 
V½ dö 3.28. Chùng minh r¬ng vîi måi sè nguy¶n n th¼ n2+n+1 khæng 
chia h¸t cho 9. 4 
Líi gi£i. Gi£ sû 9 j (n2 +n+1). Khi â n2 +n+1 = (n+2)(n1)+3 
chia h¸t cho 3. Suy ra 3 j n + 2 v  3 j n  1. Nh÷ vªy (n + 2)(n  1) 
chia h¸t cho 9, tùc n2 + n + 1 chia 9 d÷ 3, m¥u thu¨n. Ta câ pcm.  
Nhªn x²t. B i to¡n n y v¨n câ thº gi£i theo ph÷ìng ph¡p x²t sè d÷. 
V½ dö 3.29. Gi£ sû p = k:2t + 1 l  sè nguy¶n tè l´, t l  sè nguy¶n 
d֓ 
ng v  k l   
sè tü nhi¶n l´. Gi£ thi¸t x v  y l  c¡c sè tü nhi¶n m  
p j 
x2t 
+ y2t 
. Chùng minh r¬ng khi â x v  y çng thíi chia h¸t cho 
p. 4 
Líi gi£i. Gi£ sû tr¡i l¤i p - x, suy ra p - y. 
Do p l  sè nguy¶n tè n¶n theo ành lþ Fermat nhä ta câ 
 
xp1  1 (mod p) 
yp1  1 (mod p) 
Theo gi£ thi¸t th¼ p  1 = k:2t, do â 
 
xk:2t 
 1 (mod p) 
yk:2t 
 1 (mod p) 
Tø â ta câ 
xk:2t 
+ yk:2t 
 2 (mod p): (i) 
Theo gi£ thi¸t th¼ 
x2t 
+ y2t 
 0 (mod p): 
Di¹n  n To¡n håc Chuy¶n · Sè håc
3.2. Ph÷ìng ph¡p gi£i c¡c b i to¡n chia h¸t 55 
Do k l´ n¶n 
xk:2t 
+ yk:2t 
= 
 
x2t 
k 
+ 
 
y2t 
k ... 
 
x2t 
+ y2t 
 
) 
 
xk:2t 
+ yk:2t 
 
 0 (mod p) (ii) 
Tø (i) v  (ii) suy ra i·u m¥u thu¨n. Vªy gi£ thi¸t ph£n chùng sai. Do 
â x; y çng thíi chia h¸t cho p.  
B i tªp · nghà 
B i 1. Chùng minh n2 + n + 2 khæng chia h¸t cho 15 vîi måi n 2 Z. 
B i 2. Chùng minh n2 +3n+5 khæng chia h¸t cho 121 vîi måi n 2 N. 
B i 3. Chùng minh 9n3 + 9n2 + 3n  16 khæng chia h¸t cho 343 vîi 
måi n 2 N. 
B i 4. Chùng minh 4n3  6n2 + 3n + 37 khæng chia h¸t cho 125 vîi 
måi n 2 N. 
B i 5. Chùng minh n3 +3n38 khæng chia h¸t cho 49 vîi måi n 2 N. 
Chuy¶n · Sè håc Di¹n  n To¡n håc
Ch֓ng 
4 
Ph÷ìng tr¼nh nghi»m 
nguy¶n 
4.1 X²t t½nh chia h¸t 57 
4.2 Sû döng b§t ¯ng thùc 74 
4.3 Nguy¶n t­c cüc h¤n, lòi væ h¤n 86 
Tr¦n Nguy¹n Thi¸t Qu¥n (L Lawliet) 
Ph¤m Quang To n (Ph¤m Quang To n) 
Trong ch÷ìng tr¼nh THCS v  THPT th¼ ph÷ìng tr¼nh nghi»m nguy¶n 
v¨n luæn l  mët · t i hay v  khâ èi vîi håc sinh. C¡c b i to¡n nghi»m 
nguy¶n th÷íng xuy¶n xu§t hi»n t¤i c¡c k¼ thi lîn, nhä, trong v  ngo i 
n÷îc. Trong b i vi¸t n y tæi ch¿ muèn · cªp ¸n c¡c v§n · cì b£n cõa 
nghi»m nguy¶n (c¡c d¤ng, c¡c ph÷ìng ph¡p gi£i) chù khæng i nghi¶n 
cùu s¥u s­c v· nâ. Tæi công khæng · cªp tîi ph÷ìng tr¼nh Pell, ph÷ìng 
tr¼nh Pythagore, ph÷ìng tr¼nh Fermat v¼ nâ câ nhi·u trong c¡c s¡ch, 
c¡c chuy¶n · kh¡c. 
4.1 X²t t½nh chia h¸t 
4.1.1 Ph¡t hi»n t½nh chia h¸t cõa 1 ©n 
V½ dö 4.1. Gi£i ph÷ìng tr¼nh nghi»m nguy¶n 
13x + 5y = 175 (4.1) 
57
58 4.1. X²t t½nh chia h¸t 
Líi gi£i. Gi£ sû x; y l  c¡c sè nguy¶n thäa m¢n ph÷ìng tr¼nh (4.1). Ta 
th§y 175 v  5y ·u chia h¸t cho 5 n¶n 13x 
.. .5 ) x 
... 
5 (do GCD(13; 5) = 1). 
°t x = 5t (t 2 Z). Thay v o ph÷ìng tr¼nh (4.1), ta ÷ñc 
13:5t + 5y = 175 , 13t + y = 35 , y = 35  13t 
Do â, ph÷ìng tr¼nh (4.1) câ væ sè nghi»m nguy¶n biºu di¹n d÷îi d¤ng 
(x; y) = (5t; 35  13t); (t 2 Z) 
B i tªp · nghà 
B i 1. Gi£i ph÷ìng tr¼nh nghi»m nguy¶n 12x  19y = 285 
B i 2. Gi£i ph÷ìng tr¼nh nghi»m nguy¶n 7x + 13y = 65 
B i 3. Gi£i ph÷ìng tr¼nh nghi»m nguy¶n 5x + 7y = 112 
4.1.2 ÷a v· ph÷ìng tr¼nh ÷îc sè 
V½ dö 4.2. T¼m nghi»m nguy¶n cõa ph÷ìng tr¼nh 
3xy + 6x + y  52 = 0 (4.2) 
Líi gi£i. Nhªn x²t. èi vîi ph÷ìng tr¼nh n y, ta khæng thº ¡p döng 
ph÷ìng ph¡p tr¶n l  ph¡t hi»n t½nh chia h¸t, vªy ta ph£i gi£i nh÷ th¸ 
n o? 
Ta gi£i nh÷ sau: 
(4.2) , 3xy + y + 6x + 2  54 = 0 
, y (3x + 1) + 2 (3x + 1)  54 = 0 
, (3x + 1) (y + 2) = 54 
Nh÷ vªy, ¸n ¥y ta câ x v  y nguy¶n n¶n 3x + 1 v  y + 2 ph£i l  ÷îc 
cõa 54. Nh÷ng n¸u nh÷ vªy th¼ ta ph£i x²t ¸n hìn 10 tr÷íng hñp sao? 
V¼: 
4 = 1:54 = 2:27 = 3:18 = 6:9 
= (1):(54) = (2):(27) = (3):(18) = (6):(9) 
Di¹n  n To¡n håc Chuy¶n · Sè håc
4.1. X²t t½nh chia h¸t 59 
Câ c¡ch n o kh¡c khæng? C¥u tr£ líi l  câ! N¸u ta º þ mët chót ¸n 
thøa sè 3x + 1, biºu thùc n y chia cho 3 luæn d÷ 1 vîi måi x nguy¶n. 
Vîi lªp luªn tr¶n, ta ÷ñc: 
2 
664 
 
3x + 1 = 1 
y + 2 = 54 
, 
 
x = 0 
 y = 52 
3x + 1 = 2 
y + 2 = 54 
, 
 
x = 1 
y = 56 
V½ dö 4.3. Gi£i ph÷ìng tr¼nh nghi»m nguy¶n sau: 
2x + 5y + 3xy = 8 (4.3) 
Líi gi£i. Ta câ 
(4:3) , x(2 + 3y) + 5y = 8 
, 3x(2 + 3y) + 15y = 24 
, 3x(2 + 3y) + 5(2 + 3y) = 34 
, (3x + 5)(3y + 3) = 34 
¸n ¥y ph¥n t½ch 34 = 1  34 = 2  17 rçi x²t c¡c tr÷íng hñp. Chó þ 
r¬ng 3x + 5; 3y + 2 l  hai sè nguy¶n chia 3 d÷ 2, vªn döng i·u n y ta 
câ thº gi£m bît sè tr÷íng hñp c¦n x²t.  
V½ dö 4.4. Gi£i ph÷ìng tr¼nh nghi»m nguy¶n 
x2  y2 = 2011 (4.4) 
Líi gi£i. (4:4) , (x  y)(x + y) = 2011. V¼ 2011 l  sè nguy¶n tè n¶n 
÷îc nguy¶n cõa 2011 ch¿ câ thº l  1;2011. Tø â suy ra nghi»m 
(x; y) l  (1006; 1005); (1006;1005); (1006;1005); (1006; 1005).  
V½ dö 4.5. T¼m c¡c sè nguy¶n x; y tho£ m¢n i·u ki»n 
x2 + y2 = (x  y)(xy + 2) + 9 (4.5) 
Chuy¶n · Sè håc Di¹n  n To¡n håc
60 4.1. X²t t½nh chia h¸t 
Líi gi£i. °t a = x  y; b = xy. Khi â (4.5) trð th nh 
a2 + 2b = a(b + 2) + 9 , (a  2)(a  b) = 9 (4.6) 
V¼ x; y 2 Z n¶n a; ; a2; ab ·u l  c¡c sè nguy¶n. Tø (4.6) ta câ c¡c 
tr÷íng hñp sau: 
 
( 
a  2 = 9 
a  b = 1 
, 
( 
a = 11 
b = 10 
, 
( 
x  y = 11 
xy = 10 
(4.7) 
 
( 
a  2 = 3 
a  b = 3 
, 
( 
a = 5 
b = 2 
, 
( 
x  y = 5 
xy = 2 
(4.8) 
 
( 
a  2 = 1 
a  b = 9 
, 
( 
a = 3 
b = 6 
, 
( 
x  y = 3 
xy = 6 
(4.9) 
 
( 
a  2 = 1 
a  b = 9 
, 
( 
a = 1 
b = 10 
, 
( 
x  y = 1 
xy = 10 
(4.10) 
 
( 
a  2 = 3 
a  b = 3 
, 
( 
a = 1 
b = 2 
, 
( 
x  y = 1 
xy = 2 
(4.11) 
 
( 
a  2 = 3 
a  b = 3 
, 
( 
a = 1 
b = 2 
, 
( 
x  y = 1 
xy = 2 
(4.12) 
D¹ th§y c¡c h» (4.7),(4.8),(4.10) khæng câ nghi»m nguy¶n, h» (4.9) væ 
nghi»m, h» (4.11) câ hai nghi»m nguy¶n (1; 2) v  (2;1), h» (4.12) 
câ hai nghi»m nguy¶n (1; 6) v  (6; 1). 
Tâm l¤i ph÷ìng tr¼nh (4.5) câ c¡c c°p nghi»m nguy¶n (x; y) l  (1; 2); 
(2;1); (1; 6); (6; 1).  
V½ dö 4.6. T¼m nghi»m nguy¶n cõa ph÷ìng tr¼nh: 
 
x2 + 1 
  
y2 + 1 
 
+ 2 (x  y) (1  xy) = 4 (1 + xy) (4.13) 
Di¹n  n To¡n håc Chuy¶n · Sè håc
4.1. X²t t½nh chia h¸t 61 
Líi gi£i. Ph÷ìng tr¼nh (4.13) t÷ìng ÷ìng vîi: 
x2y2 + x2 + y2 + 1 + 2x  2x2y  2y + 2xy2 = 4 + 4xy 
, (x2 + 2x + 1)y2  2(x2 + 2x + 1)y + (x2 + 2x + 1) = 4 
, (x + 1)2(y  1)2 = 4 
, 
 
(x + 1)(y  1) = 2 
(x + 1)(y  1) = 2 
Vîi (x + 1)(y  1) = 2 m  x; y 2 Z n¶n ta câ c¡c tr÷íng hñp sau: 
 
 
x + 1 = 1 
y  1 = 2 
, 
 
x = 0 
y = 3 
 
 
x + 1 = 2 
y  1 = 1 
, 
 
x = 1 
y = 2 
 
 
x + 1 = 2 
y  1 = 1 
, 
 
x = 3 
y = 0 
 
 
x + 1 = 1 
y  1 = 2 
, 
 
x = 2 
y = 1 
Vîi (x + 1)(y  1) = 2 , t÷ìng tü ta công suy ra ÷ñc: 
 
 
x + 1 = 1 
y  1 = 2 
, 
 
x = 2 
y = 3 
 
 
x + 1 = 1 
y  1 = 2 
, 
 
x = 0 
y = 1 
 
 
x + 1 = 2 
y  1 = 1 
, 
 
x = 1 
y = 0 
 
 
x + 1 = 2 
y  1 = 1 
, 
 
x = 3 
y = 2 
Vªy ph÷ìng tr¼nh ¢ cho câ c¡c c°p nghi»m nguy¶n: 
(x; y) = f(0; 3); (1; 2); (3; 0); (2;1); (2; 3); (0;1); (1; 0); (3; 2)g 
V½ dö 4.7. T¼m nghi»m nguy¶n cõa ph÷ìng tr¼nh 
x6 + 3x3 + 1 = y4 (4.14) 
Chuy¶n · Sè håc Di¹n  n To¡n håc
62 4.1. X²t t½nh chia h¸t 
Líi gi£i. Nh¥n hai v¸ cõa ph÷ìng tr¼nh (4.14) cho 4, ta ÷ñc: 
4x6 + 12x3 + 4 = 4y4 
, (4x6 + 12x3 + 9)  4y4 = 5 
, (2x3 + 3)2  4y4 = 5 
, (2x3  2y2 + 3)(2x3 + 2y2 + 3) = 5: 
Vîi l÷u þ r¬ng 5 = 1:5 = 5:1 = (1):(5) = (5):(1) v  x; y 2 Z n¶n 
ta suy ra ÷ñc c¡c tr÷íng hñp sau: 
 
 
2x3  2y2 + 3 = 1 
2x3 + 2y2 + 3 = 5 
, 
 
x3  y2 = 1 
x3 + y2 = 1 
, 
 
x3 = 0 
y2 = 1 
, 
8 
: 
 
x = 0 
 y = 1 
x = 0 
y = 1 
 
 
2x3  2y2 + 3 = 1 
2x3 + 2y2 + 3 = 5 
, 
 
x3  y2 = 2 
x3 + y2 = 4 
, 
 
x3 = 3 
y2 = 1 
(lo¤i) 
 
 
2x3  2y2 + 3 = 5 
2x3 + 2y2 + 3 = 1 
, 
 
x3  y2 = 1 
x3 + y2 = 1 
, 
 
x3 = 0 
y2 = 1 
(lo¤i) 
 
 
2x3  2y2 + 3 = 5 
2x3 + 2y2 + 3 = 1 
, 
 
x3  y2 = 4 
x3 + y2 = 2 
, 
 
x3 = 3 
y2 = 1 
(lo¤i) 
Vªy ph÷ìng tr¼nh ¢ cho câ c¡c c°p nghi»m nguy¶n: 
(x; y) = f(0; 1); (0;1)g 
Nhªn x²t. B i to¡n n y công câ thº gi£i b¬ng ph÷ìng ph¡p kµp. 
V½ dö 4.8. Gi£i ph÷ìng tr¼nh nghi»m nguy¶n d÷ìng: 
1 
x 
+ 
1 
y 
= 
1 
p 
(4.15) 
trong â p l  sè nguy¶n tè. 4 
Di¹n  n To¡n håc Chuy¶n · Sè håc
4.1. X²t t½nh chia h¸t 63 
Líi gi£i. 
(4:15) , xy = px + py ) (x  y)(y  p) = p2: 
V¼ p l  sè nguy¶n tè n¶n ÷îc sè nguy¶n cõa p2 ch¿ câ thº l  1;p;p2. 
Thû l¦n l÷ñt vîi c¡c ÷îc tr¶n ta d¹ t¼m ÷ñc k¸t qu£. Ph¦n tr¼nh b y 
xin d nh cho b¤n åc.  
Nhªn x²t. Ph÷ìng ph¡p n y c¦n hai b÷îc ch½nh: Ph¥n t½ch th nh ÷îc 
sè v  x²t tr÷íng hñp º t¼m k¸t qu£. Hai b÷îc n y câ thº nâi l  khæng 
qu¡ khâ èi vîi b¤n åc, nh÷ng xin nâi mët sè l÷u þ th¶m v· b÷îc x²t 
tr÷íng hñp. Trong mët sè b i to¡n, h¬ng sè nguy¶n ð v¸ ph£i sau khi 
ph¥n t½ch l  mët sè câ nhi·u ÷îc, nh÷ vªy ái häi x²t tr÷íng hñp v  
t½nh to¡n r§t nhi·u. Mët c¥u häi °t ra l : L m th¸ n o º gi£m sè 
tr÷íng hñp bà x²t ¥y? V  º tr£ líi ÷ñc c¥u häi â, ta s³ tham kh£o 
v½ dö d÷îi ¥y. 
V½ dö 4.9. T¼m nghi»m nguy¶n cõa ph÷ìng tr¼nh: 
x2 + 12x = y2: (4.16) 
Líi gi£i. (thæng th÷íng) Ph÷ìng tr¼nh (4.16) ¢ cho t÷ìng ÷ìng vîi: 
(x + 6)2  y2 = 36 , (x + 6 + y)(x + 6  y) = 36 
Suy ra x + y + 6; x + 6  y l  ÷îc cõa 36. M  sè 36 câ t§t c£ 18 ÷îc 
n¶n ta ph£i x²t 18 tr÷íng hñp t÷ìng ùng vîi 
x + 6 + y 2 f1;2;3;4;6;9;12;18;36g 
. K¸t qu£ l  ta t¼m ÷ñc c¡c c°p nghi»m nguy¶n (x; y) l  
(0; 0); (12; 0); (16; 8); (16;8); (4; 8); (4;8) 
. 
Nhªn x²t. óng nh÷ v§n · m  ta ¢ n¶u ra ð tr¶n, sè ÷îc qu¡ nhi·u 
º x²t. Cho n¶n ta s³ câ c¡c nhªn x²t sau · thüc hi»n thao t¡c si¶u 
ph m chuyºn tø con sè 18 xuèng ch¿ cán 2! 
Chuy¶n · Sè håc Di¹n  n To¡n håc
64 4.1. X²t t½nh chia h¸t 
V¼ y câ sè mô ch®n trong ph÷ìng tr¼nh n¶n câ thº gi£ sû y  0. Khi 
â x + 6  y  x + 6 + y, do vªy ta lo¤i ÷ñc t¡m tr÷íng hñp v  cán 
l¤i c¡c tr÷íng hñp sau: 
( 
x + 6 + y = 9 
x + 6  y = 4 
; 
( 
x + 6 + y = 9 
x + 6  y = 4 
; 
( 
x + y + 6 = 1 
x + y  6 = 36 
; 
( 
x + y + 6 = 36 
x  y + 6 = 1 
; 
( 
x + y + 6 = 2 
x  y + 6 = 18 
; 
( 
x + y + 6 = 18 
x  y + 6 = 2 
; 
( 
x + y + 6 = 3 
x  y + 6 = 12 
; 
( 
x + y + 6 = 12 
x  y + 6 = 3 
; 
( 
x + y + 6 = 6 
x  y + 6 = 6 
; 
( 
x + y + 6 = 6 
x + y  6 = 6 
: 
B¥y gií ta ¢ câ 10 tr÷íng hñp, ta s³ ti¸p töc l÷ñc bä. Nhªn th§y 
(x + y + 6)  (x + 6  y) = 2y n¶n x + 6  y v  x + 6 + y câ còng t½nh 
ch®n l´, do â ta lo¤i th¶m 6 tr÷íng hñp, ch¿ cán 
( 
x + y + 6 = 18 
x + y  6 = 2 
; 
( 
x + y + 6 = 2 
x + y  6 = 18 
; 
( 
x + y + 6 = 6 
x  y + 6 = 6 
; 
( 
x + y + 6 = 6 
x + y  6 = 6 
. 
Ti¸p töc x²t hai ph÷ìng tr¼nh 
( 
x + y + 6 = 6 
x  y + 6 = 6 
v  
( 
x + y + 6 = 6 
x + y  6 = 6 
, 
hai ph÷ìng tr¼nh n y ·u t¼m ÷ñc y = 0. Vªy sao khæng º ìn gi£n 
hìn, ta x²t y = 0 ngay tø ¦u. Ph÷ìng tr¼nh câ d¤ng x(x + 12) = y2, 
x²t hai kh£ n«ng: 
 N¸u y = 0 th¼ x = 0 ho°c x = 12. 
 N¸u y6= 0 th¼ x+6+y  x+6y, ¡p döng hai nhªn x²t tr¶n ta ch¿ 
câ hai tr÷íng hñp: 
( 
x + y + 6 = 2 
x  y + 6 = 18 
v  
( 
x + y + 6 = 18 
x  y + 6 = 2 
. 
 
Di¹n  n To¡n håc Chuy¶n · Sè håc
4.1. X²t t½nh chia h¸t 65 
Ph÷ìng tr¼nh ¢ cho câ 6 nghi»m nguy¶n 
(x; y) = (16; 8); (0; 0); (12; 0); (16; 8); (4; 8); (4;8) 
Nhªn x²t. Nh÷ vªy b i to¡n ng­n gån, ch½nh x¡c nhí linh ho¤t trong 
vi»c x²t t½nh ch®n l´, giîi h¤n hai sè º gi£m sè tr÷íng hñp c¦n x²t. 
Ngo i c¡c c¡ch ¡nh gi¡ tr¶n ta cán câ thº ¡p döng x²t sè d÷ tøng v¸ 
º ¡nh gi¡ (¥y công l  mët ph÷ìng ph¡p gi£i ph÷ìng tr¼nh nghi»m 
nguy¶n). 
B i tªp · nghà 
B i 1. Thû bi¸n êi c¡c b i to¡n gi£i ph÷ìng tr¼nh nghi»m nguy¶n 
ð ph÷ìng ph¡p Biºu thà mët ©n theo ©n cán l¤i b¬ng ph÷ìng 
ph¡p ÷a v· ÷îc sè. 
B i 2. T¼m ë d i c¤nh mët tam gi¡c vuæng sao cho t½ch hai c¤nh 
huy·n g§p ba l¦n chu vi tam gi¡c â. 
B i 3. Gi£i ph÷ìng tr¼nh nghi»m nguy¶n x  y + 2xy = 6 
B i 4. Gi£i ph÷ìng tr¼nh nghi»m nguy¶n 2x + 5y + 2xy = 8 
B i 5. (Thi HSG lîp 9 t¿nh Qu£ng Ng¢i n«m 2011-2012) Gi£i ph÷ìng 
tr¼nh nghi»m nguy¶n 6x + 5y + 18 = 2xy 
B i 6. T¼m nghi»m nguy¶n (xy  7)2 = x2 + y2 
B i 7. T¼m x; y 2 Z thäa m¢n 2x2  2xy = 5x  y  19. 
B i 8. T¼m nghi»m nguy¶n cõa ph÷ìng tr¼nh x2+6xy+8y2+3x+6y = 
2. 
B i 9. T¼m nghi»m nguy¶n d÷ìng cõa ph÷ìng tr¼nh x3 y3 = xy +61 
B i 10. T¼m nghi»m nguy¶n cõa ph÷ìng tr¼nh 4x2y2 = 22+x(1+x)+ 
y(1 + y) 
B i 11. Gi£i ph÷ìng tr¼nh nghi»m nguy¶n x(x + 1)(x + 7)(x + 8) = y2. 
Chuy¶n · Sè håc Di¹n  n To¡n håc
66 4.1. X²t t½nh chia h¸t 
B i 12. T¼m nghi»m nguy¶n d÷ìng cõa ph÷ìng tr¼nh 6x3  xy(11x + 
3y) + 2y3 = 6 (T¤p ch½ TTT2 sè 106). 
B i 13. T¼m nghi»m nguy¶n d÷ìng cõa ph÷ìng tr¼nh x(x+2y)3y(y+ 
2x)3 = 27 (t¤p ch½ THTT sè 398). 
B i 14. T¼m nghi»m nguy¶n cõa ph÷ìng tr¼nh 
p 
9x2 + 16x + 96 = 3x 
16y  24. 
B i 15. T¼m nghi»m nguy¶n d÷ìng cõa ph÷ìng tr¼nh 
2 + 
s 
x + 
1 
2 
+ 
r 
x + 
1 
4 
= y 
. 
B i 16. T¼m sè nguy¶n x º x2  4x  52 l  sè ch½nh ph÷ìng. 
B i 17. Gi£i ph÷ìng tr¼nh nghi»m nguy¶n x2 +2y2 +3xy 2xy = 6. 
B i 18. Gi£i ph÷ìng tr¼nh nghi»m nguy¶n x2 +3xy y2 +2x3y = 5. 
B i 19. Gi£i ph÷ìng tr¼nh nghi»m nguy¶n 2x2 +3y2 +xy 3x3 = y. 
B i 20. (Tuyºn sinh v o lîp 10 THPT chuy¶n tr÷íng KHTN H  Nëi 
n«m håc 2012-2013) T¼m t§t c£ c¡c c°p sè nguy¶n x; y thäa 
m¢n ¯ng thùc (x + y + 1)(xy + x + y) = 5 + 2(x + y). 
B i 21. Gi£i ph÷ìng tr¼nh nghi»m nguy¶n x42y4x2y24x27y2 
5 = 0. 
(Thi HSG lîp 9 t¿nh H÷ng Y¶n n«m 2011-2012) 
B i 22. (Romanian 1999) Chùng minh r¬ng ph÷ìng tr¼nh sau khæng câ 
nghi»m nguy¶n 
x5  x4y  13x3y2 + 13x2y3 + 36xy4  36y5 = 1937 
Di¹n  n To¡n håc Chuy¶n · Sè håc
4.1. X²t t½nh chia h¸t 67 
4.1.3 Biºu thà mët ©n theo ©n cán l¤i rçi sû döng t½nh chia 
h¸t 
V½ dö 4.10. T¼m nghi»m nguy¶n cõa ph÷ìng tr¼nh 
2x  xy + 3 = 0 (4.17) 
Líi gi£i. Nhªn x²t. Ð ph÷ìng tr¼nh n y ta khæng thº ¡p döng c¡c c¡ch 
¢ bi¸t, vªy ta ph£i l m sao? Chó þ hìn mët x½u núa ta th§y câ thº 
biºu di¹n y theo x ÷ñc rçi vªn döng ki¸n thùc t¼m gi¡ trà nguy¶n ð 
lîp 8 t¼m nghi»m nguy¶n cõa ph÷ìng tr¼nh, thû l m theo þ t÷ðng â 
xem sao. 
(4.17) , xy = 2x + 3 
N¸u x = 0 th¼ ph÷ìng tr¼nh (4.17) ¢ cho væ nghi»m nguy¶n y. 
N¸u x6= 0 th¼ 
(4.17) , y = 
2x + 3 
x 
= 2 + 
3 
x 
Nh÷ vªy muèn y nguy¶n th¼ ta c¦n 3 
x 
nguy¶n hay nâi c¡ch kh¡c x l  
÷îc cõa 3. Vîi méi gi¡ trà nguy¶n x ta t¼m ÷ñc mët gi¡ trà y nguy¶n. 
Tø â, ta câ bë nghi»m cõa (4.17) l  
(x; y) = (3; 1); (1;1); (1; 5); (3; 3) 
V½ dö 4.11 (Thi HSG lîp 9 Qu£ng Ng¢i n«m 2011-2012). T¼m c¡c sè 
nguy¶n d÷ìng x; y sao cho 
6x + 5y + 18 = 2xy (4.18) 
Nhªn x²t. H÷îng ph¥n t½ch v  ành h÷îng líi gi£i. ¢ x¡c ành ÷ñc 
ph÷ìng ph¡p cõa d¤ng n y th¼ b¥y gií ta s³ biºu di¹n ©n x theo y. 
Khæng khâ º vi¸t th nh x = 
5y  18 
6  2y 
. Ta d÷íng nh÷ nh¥n th§y biºu 
thùc n y r§t khâ ph¥n t½ch nh÷ biºu thùc ð v½ dö ¦u. Tuy nhi¶n, n¸u 
º þ k¾ s³ th§y b¶n m¨u l  2y v  tû l  5y, do â ta m¤nh d¤n nh¥n 2 
v o tû º xu§t hi»n 2y gièng nh÷ ð m¨u. 
Chuy¶n · Sè håc Di¹n  n To¡n håc
68 4.1. X²t t½nh chia h¸t 
Líi gi£i. Ta câ 
(4.18) , x = 
5y  18 
6  2y 
, 2x = 
10y  36 
6  2y 
, 2x = 
66 + 5(6  2y) 
6  2y 
= 
66 
6  2y 
+ 5 
, 2x = 
33 
3  y 
+ 5 
Nh÷ vªy muèn x l  sè nguy¶n d÷ìng th¼ 3  y l  ph£i l  ÷îc cõa 33. 
Hay 3y 2 f1;3;11;33g. L¤i º þ r¬ng v¼ y  1 n¶n 3 y  2. 
Do â ch¿ câ thº 3  y 2 f1;3;11;33g. Ta câ b£ng sau: 
3  y 1 1 3 11 33 
y 2 4 6 14 36 
x 14 19 8 4 3 
Thû l¤i th§y c¡c c°p (x; y) nguy¶n d÷ìng thäa m¢n (4.18) l  (x; y) = 
(19; 4); (8; 6); (4; 14); (3; 36).  
Nhªn x²t. B i n y ta công câ thº sû döng ph÷ìng ph¡p ÷a v· ph÷ìng 
tr¼nh ÷îc sè. Công xin chó þ vîi b¤n r¬ng ð líi gi£i tr¶n th¼ ta ¢ nh¥n 
2 ð x º bi¸n êi, do â ph£i câ mët b÷îc thû l¤i coi gi¡ trà x; y t¼m 
÷ñc câ thäa m¢n (4.18) hay khæng rçi mîi câ thº k¸t luªn. 
B i tªp · nghà 
B i 1. Gi£i ph÷ìng tr¼nh nghi»m nguy¶n x2  xy = 6x  5y  8. 
B i 2. Gi£i ph÷ìng tr¼nh nghi»m nguy¶n x2 + x + 1 = 2xy + y. 
B i 3. Gi£i ph÷ìng tr¼nh nghi»m nguy¶n x3  x2y + 3x  2y  5 = 0: 
B i 4. (V o 10 chuy¶n THPT HKHTN H  Nëi n«m 2001-2002) T¼m 
gi¡ trà x; y nguy¶n thäa m¢n ¯ng thùc (y  2)x2 + 1 = y2. 
Di¹n  n To¡n håc Chuy¶n · Sè håc
4.1. X²t t½nh chia h¸t 69 
B i 5. (V o 10 chuy¶n THPT HKHTN H  Nëi n«m 2000-2001) T¼m 
c°p sè nguy¶n (x; y) thäa m¢n ¯ng thùc y(x  1) = x2 + 2. 
B i 6. T¼m sè nhä nh§t trong c¡c sè nguy¶n d÷ìng l  bëi cõa 2007 v  
câ 4 chú sè cuèi còng l  2008. 
B i 7. T¼m nghi»m nguy¶n cõa ph÷ìng tr¼nh 5x  3y = 2xy  11. 
4.1.4 X²t sè d÷ tøng v¸ 
Cì sð ph÷ìng ph¡p. åc ngay ti¶u · ph÷ìng ph¡p th¼ ch­c b¤n s³ 
hiºu ngay ph÷ìng ph¡p n y nâi ¸n vi»c x²t sè d÷ ð tøng v¸ cho còng 
mët sè. Vªy, t¤i sao l¤i ph£i x²t v  x²t nh÷ vªy câ lñi ½ch g¼ trong cæng 
cuëc gi£i to¡n? H¢y còng t¼m hiºu qua v½ dö ¦u sau: 
V½ dö 4.12. T¼m nghi»m nguy¶n cõa ph÷ìng tr¼nh 
x2 + y2 = 2011 (4.19) 
Líi gi£i. Ta câ x2; y2 chia 4 câ thº d÷ 0 ho°c 1 n¶n têng chóng chia 4 
ch¿ câ thº d÷ 0; 1 ho°c 2. M°t kh¡c 2011 chia 4 d÷a 3 n¶n ph÷ìng tr¼nh 
(4.19) væ nghi»m nguy¶n.  
Nhªn x²t. Qua v½ dö ¦u n y th¼ ta ¢ th§y rã sè d÷ khi chia cho 4 cõa 
hai sè kh¡c nhau th¼ ph÷ìng tr¼nh væ nghi»m. Do â ta l¤i c ng hiºu 
th¶m möc ½ch cõa ph÷ìng ph¡p n y. Bªt m½ th¶m t½ núa th¼ ph÷ìng 
ph¡p n y chõ y¸u dòng cho c¡c ph÷ìng tr¼nh khæng câ nghi»m nguy¶n. 
Cho n¶n, n¸u b¤n b­t g°p mët ph÷ìng tr¼nh b§t k¼ m  b¤n khæng thº 
t¼m ra ÷ñc nghi»m cho ph÷ìng tr¼nh â, th¼ h¢y ngh¾ ¸n ph÷ìng 
ph¡p n y ¦u ti¶n. Cán b¥y gií ta ti¸p töc ¸n vîi v½ dö sau: 
V½ dö 4.13 (Balkan MO 1998). T¼m nghi»m nguy¶n cõa ph÷ìng tr¼nh 
x2 = y5  4 (4.20) 
Líi gi£i. Ta câ: x2  0; 1; 3; 4; 5; 9 (mod 11). Trong khi â y5  4  
6; 7; 8 (mod 11): væ lþ. Vªy ph÷ìng tr¼nh (4.20) væ nghi»m nguy¶n.  
Chuy¶n · Sè håc Di¹n  n To¡n håc
Xuctu.com chuyen-de-so-hoc-vmf
Xuctu.com chuyen-de-so-hoc-vmf
Xuctu.com chuyen-de-so-hoc-vmf
Xuctu.com chuyen-de-so-hoc-vmf
Xuctu.com chuyen-de-so-hoc-vmf
Xuctu.com chuyen-de-so-hoc-vmf
Xuctu.com chuyen-de-so-hoc-vmf
Xuctu.com chuyen-de-so-hoc-vmf
Xuctu.com chuyen-de-so-hoc-vmf
Xuctu.com chuyen-de-so-hoc-vmf
Xuctu.com chuyen-de-so-hoc-vmf
Xuctu.com chuyen-de-so-hoc-vmf
Xuctu.com chuyen-de-so-hoc-vmf
Xuctu.com chuyen-de-so-hoc-vmf
Xuctu.com chuyen-de-so-hoc-vmf
Xuctu.com chuyen-de-so-hoc-vmf
Xuctu.com chuyen-de-so-hoc-vmf
Xuctu.com chuyen-de-so-hoc-vmf
Xuctu.com chuyen-de-so-hoc-vmf
Xuctu.com chuyen-de-so-hoc-vmf
Xuctu.com chuyen-de-so-hoc-vmf
Xuctu.com chuyen-de-so-hoc-vmf
Xuctu.com chuyen-de-so-hoc-vmf
Xuctu.com chuyen-de-so-hoc-vmf
Xuctu.com chuyen-de-so-hoc-vmf
Xuctu.com chuyen-de-so-hoc-vmf
Xuctu.com chuyen-de-so-hoc-vmf
Xuctu.com chuyen-de-so-hoc-vmf
Xuctu.com chuyen-de-so-hoc-vmf
Xuctu.com chuyen-de-so-hoc-vmf
Xuctu.com chuyen-de-so-hoc-vmf
Xuctu.com chuyen-de-so-hoc-vmf
Xuctu.com chuyen-de-so-hoc-vmf
Xuctu.com chuyen-de-so-hoc-vmf
Xuctu.com chuyen-de-so-hoc-vmf
Xuctu.com chuyen-de-so-hoc-vmf
Xuctu.com chuyen-de-so-hoc-vmf
Xuctu.com chuyen-de-so-hoc-vmf
Xuctu.com chuyen-de-so-hoc-vmf
Xuctu.com chuyen-de-so-hoc-vmf
Xuctu.com chuyen-de-so-hoc-vmf
Xuctu.com chuyen-de-so-hoc-vmf
Xuctu.com chuyen-de-so-hoc-vmf
Xuctu.com chuyen-de-so-hoc-vmf
Xuctu.com chuyen-de-so-hoc-vmf
Xuctu.com chuyen-de-so-hoc-vmf
Xuctu.com chuyen-de-so-hoc-vmf
Xuctu.com chuyen-de-so-hoc-vmf
Xuctu.com chuyen-de-so-hoc-vmf
Xuctu.com chuyen-de-so-hoc-vmf
Xuctu.com chuyen-de-so-hoc-vmf
Xuctu.com chuyen-de-so-hoc-vmf
Xuctu.com chuyen-de-so-hoc-vmf
Xuctu.com chuyen-de-so-hoc-vmf
Xuctu.com chuyen-de-so-hoc-vmf
Xuctu.com chuyen-de-so-hoc-vmf
Xuctu.com chuyen-de-so-hoc-vmf
Xuctu.com chuyen-de-so-hoc-vmf
Xuctu.com chuyen-de-so-hoc-vmf
Xuctu.com chuyen-de-so-hoc-vmf
Xuctu.com chuyen-de-so-hoc-vmf
Xuctu.com chuyen-de-so-hoc-vmf
Xuctu.com chuyen-de-so-hoc-vmf
Xuctu.com chuyen-de-so-hoc-vmf
Xuctu.com chuyen-de-so-hoc-vmf
Xuctu.com chuyen-de-so-hoc-vmf
Xuctu.com chuyen-de-so-hoc-vmf
Xuctu.com chuyen-de-so-hoc-vmf
Xuctu.com chuyen-de-so-hoc-vmf
Xuctu.com chuyen-de-so-hoc-vmf
Xuctu.com chuyen-de-so-hoc-vmf
Xuctu.com chuyen-de-so-hoc-vmf
Xuctu.com chuyen-de-so-hoc-vmf

More Related Content

Viewers also liked

Xuctu.com sach ky yeu bdhsg dong thap 2013-2014
Xuctu.com sach ky yeu bdhsg dong thap 2013-2014Xuctu.com sach ky yeu bdhsg dong thap 2013-2014
Xuctu.com sach ky yeu bdhsg dong thap 2013-2014Minh Đức
 
Xuctu.com hoc nhanh-daiso
Xuctu.com hoc nhanh-daisoXuctu.com hoc nhanh-daiso
Xuctu.com hoc nhanh-daisoMinh Đức
 
Xuctu.com 30 de_thi_hoc_sinh_gioi_thcs_nguyen_vu_thanh
Xuctu.com 30 de_thi_hoc_sinh_gioi_thcs_nguyen_vu_thanhXuctu.com 30 de_thi_hoc_sinh_gioi_thcs_nguyen_vu_thanh
Xuctu.com 30 de_thi_hoc_sinh_gioi_thcs_nguyen_vu_thanhMinh Đức
 
Xuctu.com tuyen-tap-bat-dang-thuc
Xuctu.com tuyen-tap-bat-dang-thucXuctu.com tuyen-tap-bat-dang-thuc
Xuctu.com tuyen-tap-bat-dang-thucMinh Đức
 
Cac chuyen _de_boi_duong_hoc_sinh_gioi_toan_lop_7_
Cac chuyen _de_boi_duong_hoc_sinh_gioi_toan_lop_7_Cac chuyen _de_boi_duong_hoc_sinh_gioi_toan_lop_7_
Cac chuyen _de_boi_duong_hoc_sinh_gioi_toan_lop_7_ntmtam80
 
Tuyển chọn đề thi học sinh gioi lop 7
Tuyển chọn đề thi học sinh gioi lop 7Tuyển chọn đề thi học sinh gioi lop 7
Tuyển chọn đề thi học sinh gioi lop 7Nguuyen Truong
 
15 bai toan_boi_duong_hsg_toan_l8_8383
15 bai toan_boi_duong_hsg_toan_l8_838315 bai toan_boi_duong_hsg_toan_l8_8383
15 bai toan_boi_duong_hsg_toan_l8_8383Manh Tranduongquoc
 
Tuyển tập đề thi học sinh giỏi lớp 8
Tuyển tập đề thi học sinh giỏi lớp 8Tuyển tập đề thi học sinh giỏi lớp 8
Tuyển tập đề thi học sinh giỏi lớp 8Nhật Hiếu
 
Giáo án bồi dưỡng học sinh giỏi Toán 7
Giáo án bồi dưỡng học sinh giỏi Toán 7Giáo án bồi dưỡng học sinh giỏi Toán 7
Giáo án bồi dưỡng học sinh giỏi Toán 7Lớp 7 Gia sư
 
Một số chuyên đề nâng cao đại số lớp 7
Một số chuyên đề nâng cao đại số lớp 7Một số chuyên đề nâng cao đại số lớp 7
Một số chuyên đề nâng cao đại số lớp 7vukimhoanc2vinhhoa
 
đề Thi hsg toán 8 có đáp án
đề Thi hsg toán 8 có đáp ánđề Thi hsg toán 8 có đáp án
đề Thi hsg toán 8 có đáp ánCảnh
 
Bai tap-hinh-lop-7-hay-co-loi-giai
Bai tap-hinh-lop-7-hay-co-loi-giaiBai tap-hinh-lop-7-hay-co-loi-giai
Bai tap-hinh-lop-7-hay-co-loi-giaiSa Hong
 
Kĩ thuật giải hệ phương trình
Kĩ thuật giải hệ phương trìnhKĩ thuật giải hệ phương trình
Kĩ thuật giải hệ phương trìnhToàn Đinh
 
Các chuyên đề bồi dưỡng HSG môn Toán THCS hay nhất
Các chuyên đề bồi dưỡng HSG môn Toán THCS hay nhấtCác chuyên đề bồi dưỡng HSG môn Toán THCS hay nhất
Các chuyên đề bồi dưỡng HSG môn Toán THCS hay nhấtBồi dưỡng Toán lớp 6
 
bộ đề+ đáp án đề thi học sinh giỏi hình học 8
bộ đề+ đáp án đề thi học sinh giỏi hình học 8bộ đề+ đáp án đề thi học sinh giỏi hình học 8
bộ đề+ đáp án đề thi học sinh giỏi hình học 8Jackson Linh
 

Viewers also liked (20)

Xuctu.com sach ky yeu bdhsg dong thap 2013-2014
Xuctu.com sach ky yeu bdhsg dong thap 2013-2014Xuctu.com sach ky yeu bdhsg dong thap 2013-2014
Xuctu.com sach ky yeu bdhsg dong thap 2013-2014
 
Xuctu.com hoc nhanh-daiso
Xuctu.com hoc nhanh-daisoXuctu.com hoc nhanh-daiso
Xuctu.com hoc nhanh-daiso
 
Xuctu.com 30 de_thi_hoc_sinh_gioi_thcs_nguyen_vu_thanh
Xuctu.com 30 de_thi_hoc_sinh_gioi_thcs_nguyen_vu_thanhXuctu.com 30 de_thi_hoc_sinh_gioi_thcs_nguyen_vu_thanh
Xuctu.com 30 de_thi_hoc_sinh_gioi_thcs_nguyen_vu_thanh
 
Xuctu.com tuyen-tap-bat-dang-thuc
Xuctu.com tuyen-tap-bat-dang-thucXuctu.com tuyen-tap-bat-dang-thuc
Xuctu.com tuyen-tap-bat-dang-thuc
 
Cac chuyen _de_boi_duong_hoc_sinh_gioi_toan_lop_7_
Cac chuyen _de_boi_duong_hoc_sinh_gioi_toan_lop_7_Cac chuyen _de_boi_duong_hoc_sinh_gioi_toan_lop_7_
Cac chuyen _de_boi_duong_hoc_sinh_gioi_toan_lop_7_
 
Tuyển chọn đề thi học sinh gioi lop 7
Tuyển chọn đề thi học sinh gioi lop 7Tuyển chọn đề thi học sinh gioi lop 7
Tuyển chọn đề thi học sinh gioi lop 7
 
15 bai toan_boi_duong_hsg_toan_l8_8383
15 bai toan_boi_duong_hsg_toan_l8_838315 bai toan_boi_duong_hsg_toan_l8_8383
15 bai toan_boi_duong_hsg_toan_l8_8383
 
Tuyển tập đề thi học sinh giỏi lớp 8
Tuyển tập đề thi học sinh giỏi lớp 8Tuyển tập đề thi học sinh giỏi lớp 8
Tuyển tập đề thi học sinh giỏi lớp 8
 
Giáo án bồi dưỡng học sinh giỏi Toán 7
Giáo án bồi dưỡng học sinh giỏi Toán 7Giáo án bồi dưỡng học sinh giỏi Toán 7
Giáo án bồi dưỡng học sinh giỏi Toán 7
 
Một số chuyên đề nâng cao đại số lớp 7
Một số chuyên đề nâng cao đại số lớp 7Một số chuyên đề nâng cao đại số lớp 7
Một số chuyên đề nâng cao đại số lớp 7
 
Một số chuyên đề bồi dưỡng HSG môn Toán lớp 8
Một số chuyên đề bồi dưỡng HSG môn Toán lớp 8Một số chuyên đề bồi dưỡng HSG môn Toán lớp 8
Một số chuyên đề bồi dưỡng HSG môn Toán lớp 8
 
đề Thi hsg toán 8 có đáp án
đề Thi hsg toán 8 có đáp ánđề Thi hsg toán 8 có đáp án
đề Thi hsg toán 8 có đáp án
 
Bai tap-hinh-lop-7-hay-co-loi-giai
Bai tap-hinh-lop-7-hay-co-loi-giaiBai tap-hinh-lop-7-hay-co-loi-giai
Bai tap-hinh-lop-7-hay-co-loi-giai
 
Tuyen tap 410 cau he phuong trinh
Tuyen tap 410 cau he phuong trinh Tuyen tap 410 cau he phuong trinh
Tuyen tap 410 cau he phuong trinh
 
Kĩ thuật giải hệ phương trình
Kĩ thuật giải hệ phương trìnhKĩ thuật giải hệ phương trình
Kĩ thuật giải hệ phương trình
 
Các chuyên đề bồi dưỡng HSG môn Toán THCS hay nhất
Các chuyên đề bồi dưỡng HSG môn Toán THCS hay nhấtCác chuyên đề bồi dưỡng HSG môn Toán THCS hay nhất
Các chuyên đề bồi dưỡng HSG môn Toán THCS hay nhất
 
Chuyên đề bồi dưỡng HSG môn Toán lớp 7
Chuyên đề bồi dưỡng HSG môn Toán lớp 7Chuyên đề bồi dưỡng HSG môn Toán lớp 7
Chuyên đề bồi dưỡng HSG môn Toán lớp 7
 
Tổng hợp hệ pt
Tổng hợp hệ ptTổng hợp hệ pt
Tổng hợp hệ pt
 
Tuyển tập một số đề thi HSG môn Toán lớp 8 có đáp án - Toán Thầy Thích - Toan...
Tuyển tập một số đề thi HSG môn Toán lớp 8 có đáp án - Toán Thầy Thích - Toan...Tuyển tập một số đề thi HSG môn Toán lớp 8 có đáp án - Toán Thầy Thích - Toan...
Tuyển tập một số đề thi HSG môn Toán lớp 8 có đáp án - Toán Thầy Thích - Toan...
 
bộ đề+ đáp án đề thi học sinh giỏi hình học 8
bộ đề+ đáp án đề thi học sinh giỏi hình học 8bộ đề+ đáp án đề thi học sinh giỏi hình học 8
bộ đề+ đáp án đề thi học sinh giỏi hình học 8
 

Similar to Xuctu.com chuyen-de-so-hoc-vmf

Toan daisotohop-chuong2
Toan daisotohop-chuong2Toan daisotohop-chuong2
Toan daisotohop-chuong2Huynh ICT
 
Toan dai so to hop-chuong2
Toan dai so to hop-chuong2Toan dai so to hop-chuong2
Toan dai so to hop-chuong2Long Nguyen
 
Tv tlcm 01_131
Tv tlcm 01_131Tv tlcm 01_131
Tv tlcm 01_131Phi Phi
 
hoccokhi.vn Cơ Học Lý Thuyết (Tóm Tắt Lý Thuyết & Bài Tập Mẫu) - Trịnh Anh Ngọc
hoccokhi.vn Cơ Học Lý Thuyết (Tóm Tắt Lý Thuyết & Bài Tập Mẫu) - Trịnh Anh Ngọchoccokhi.vn Cơ Học Lý Thuyết (Tóm Tắt Lý Thuyết & Bài Tập Mẫu) - Trịnh Anh Ngọc
hoccokhi.vn Cơ Học Lý Thuyết (Tóm Tắt Lý Thuyết & Bài Tập Mẫu) - Trịnh Anh NgọcHọc Cơ Khí
 
Giáo trình Mật mã học.pdf
Giáo trình Mật mã học.pdfGiáo trình Mật mã học.pdf
Giáo trình Mật mã học.pdfMan_Ebook
 
108 bài toán chọn lọc 4.pdf
108 bài toán chọn lọc 4.pdf108 bài toán chọn lọc 4.pdf
108 bài toán chọn lọc 4.pdfdaothuybk
 
Bo truyen dai
Bo truyen daiBo truyen dai
Bo truyen daiBKMetalx
 
Bai giang mxd dc
Bai giang mxd dcBai giang mxd dc
Bai giang mxd dcpham manh
 
2b[1]. co hoc_dat_-_duong_hong_tham
2b[1]. co hoc_dat_-_duong_hong_tham2b[1]. co hoc_dat_-_duong_hong_tham
2b[1]. co hoc_dat_-_duong_hong_thamdavidcuong_lyson
 
Làm sao dịch chuyển núi phú sĩ
Làm sao dịch chuyển núi phú sĩ Làm sao dịch chuyển núi phú sĩ
Làm sao dịch chuyển núi phú sĩ Thanh An Nguyen
 
Tailieu.vncty.com qua trinh-va_thiet_bi_truyen_chat_3869
Tailieu.vncty.com   qua trinh-va_thiet_bi_truyen_chat_3869Tailieu.vncty.com   qua trinh-va_thiet_bi_truyen_chat_3869
Tailieu.vncty.com qua trinh-va_thiet_bi_truyen_chat_3869Trần Đức Anh
 

Similar to Xuctu.com chuyen-de-so-hoc-vmf (20)

Cac ham so so hoc
Cac ham so so hocCac ham so so hoc
Cac ham so so hoc
 
Baitap hoan vi
Baitap   hoan viBaitap   hoan vi
Baitap hoan vi
 
Toan daisotohop-chuong2
Toan daisotohop-chuong2Toan daisotohop-chuong2
Toan daisotohop-chuong2
 
Toan dai so to hop-chuong2
Toan dai so to hop-chuong2Toan dai so to hop-chuong2
Toan dai so to hop-chuong2
 
Vận Dụng Phép Đếm Nâng Cao Vào Giải Một Số Bài Toán Thi Học Sinh Giỏi.doc
Vận Dụng Phép Đếm Nâng Cao Vào Giải Một Số Bài Toán Thi Học Sinh Giỏi.docVận Dụng Phép Đếm Nâng Cao Vào Giải Một Số Bài Toán Thi Học Sinh Giỏi.doc
Vận Dụng Phép Đếm Nâng Cao Vào Giải Một Số Bài Toán Thi Học Sinh Giỏi.doc
 
Tv tlcm 01_131
Tv tlcm 01_131Tv tlcm 01_131
Tv tlcm 01_131
 
Cơ học lý thuyết.
Cơ học lý thuyết. Cơ học lý thuyết.
Cơ học lý thuyết.
 
Chuong 1
Chuong 1Chuong 1
Chuong 1
 
hoccokhi.vn Cơ Học Lý Thuyết (Tóm Tắt Lý Thuyết & Bài Tập Mẫu) - Trịnh Anh Ngọc
hoccokhi.vn Cơ Học Lý Thuyết (Tóm Tắt Lý Thuyết & Bài Tập Mẫu) - Trịnh Anh Ngọchoccokhi.vn Cơ Học Lý Thuyết (Tóm Tắt Lý Thuyết & Bài Tập Mẫu) - Trịnh Anh Ngọc
hoccokhi.vn Cơ Học Lý Thuyết (Tóm Tắt Lý Thuyết & Bài Tập Mẫu) - Trịnh Anh Ngọc
 
Chuong 2
Chuong 2Chuong 2
Chuong 2
 
Giáo trình Mật mã học.pdf
Giáo trình Mật mã học.pdfGiáo trình Mật mã học.pdf
Giáo trình Mật mã học.pdf
 
Chuong 4
Chuong 4Chuong 4
Chuong 4
 
108 bài toán chọn lọc 4.pdf
108 bài toán chọn lọc 4.pdf108 bài toán chọn lọc 4.pdf
108 bài toán chọn lọc 4.pdf
 
Bo truyen dai
Bo truyen daiBo truyen dai
Bo truyen dai
 
Bai giang mxd dc
Bai giang mxd dcBai giang mxd dc
Bai giang mxd dc
 
Các bài toán tổ hợp
Các bài toán tổ hợpCác bài toán tổ hợp
Các bài toán tổ hợp
 
2b[1]. co hoc_dat_-_duong_hong_tham
2b[1]. co hoc_dat_-_duong_hong_tham2b[1]. co hoc_dat_-_duong_hong_tham
2b[1]. co hoc_dat_-_duong_hong_tham
 
Làm sao dịch chuyển núi phú sĩ
Làm sao dịch chuyển núi phú sĩ Làm sao dịch chuyển núi phú sĩ
Làm sao dịch chuyển núi phú sĩ
 
Tailieu.vncty.com qua trinh-va_thiet_bi_truyen_chat_3869
Tailieu.vncty.com   qua trinh-va_thiet_bi_truyen_chat_3869Tailieu.vncty.com   qua trinh-va_thiet_bi_truyen_chat_3869
Tailieu.vncty.com qua trinh-va_thiet_bi_truyen_chat_3869
 
Nchuong4
Nchuong4Nchuong4
Nchuong4
 

More from Minh Đức

Untitled Presentation
Untitled PresentationUntitled Presentation
Untitled PresentationMinh Đức
 
De l10-dhqghn-2013-toan
De l10-dhqghn-2013-toanDe l10-dhqghn-2013-toan
De l10-dhqghn-2013-toanMinh Đức
 
50dethihsgtoan9 140928111901-phpapp01
50dethihsgtoan9 140928111901-phpapp0150dethihsgtoan9 140928111901-phpapp01
50dethihsgtoan9 140928111901-phpapp01Minh Đức
 
Xuctu.com cd pt_hpt_bpt_phuong_trinh_bat_phuong_trinh_sieu_viet
Xuctu.com cd pt_hpt_bpt_phuong_trinh_bat_phuong_trinh_sieu_vietXuctu.com cd pt_hpt_bpt_phuong_trinh_bat_phuong_trinh_sieu_viet
Xuctu.com cd pt_hpt_bpt_phuong_trinh_bat_phuong_trinh_sieu_vietMinh Đức
 
Xuctu.com de thi_tuyen_sinh_10_lhp_tdn_chuyen_dhsp(hcm)
Xuctu.com de thi_tuyen_sinh_10_lhp_tdn_chuyen_dhsp(hcm)Xuctu.com de thi_tuyen_sinh_10_lhp_tdn_chuyen_dhsp(hcm)
Xuctu.com de thi_tuyen_sinh_10_lhp_tdn_chuyen_dhsp(hcm)Minh Đức
 
Xuctu.com tuyen chon_cac_chuyen_de_tren_tap_chi_toan_hoc_va_tuoi_tre
Xuctu.com tuyen chon_cac_chuyen_de_tren_tap_chi_toan_hoc_va_tuoi_treXuctu.com tuyen chon_cac_chuyen_de_tren_tap_chi_toan_hoc_va_tuoi_tre
Xuctu.com tuyen chon_cac_chuyen_de_tren_tap_chi_toan_hoc_va_tuoi_treMinh Đức
 
Xuctu.com tuyen tap-de-thi-olympic-52-de-thi-va-giai-tap-1
Xuctu.com tuyen tap-de-thi-olympic-52-de-thi-va-giai-tap-1Xuctu.com tuyen tap-de-thi-olympic-52-de-thi-va-giai-tap-1
Xuctu.com tuyen tap-de-thi-olympic-52-de-thi-va-giai-tap-1Minh Đức
 
Xuctu.com ch de-cuctri-gtln-gtnn
Xuctu.com ch de-cuctri-gtln-gtnnXuctu.com ch de-cuctri-gtln-gtnn
Xuctu.com ch de-cuctri-gtln-gtnnMinh Đức
 

More from Minh Đức (8)

Untitled Presentation
Untitled PresentationUntitled Presentation
Untitled Presentation
 
De l10-dhqghn-2013-toan
De l10-dhqghn-2013-toanDe l10-dhqghn-2013-toan
De l10-dhqghn-2013-toan
 
50dethihsgtoan9 140928111901-phpapp01
50dethihsgtoan9 140928111901-phpapp0150dethihsgtoan9 140928111901-phpapp01
50dethihsgtoan9 140928111901-phpapp01
 
Xuctu.com cd pt_hpt_bpt_phuong_trinh_bat_phuong_trinh_sieu_viet
Xuctu.com cd pt_hpt_bpt_phuong_trinh_bat_phuong_trinh_sieu_vietXuctu.com cd pt_hpt_bpt_phuong_trinh_bat_phuong_trinh_sieu_viet
Xuctu.com cd pt_hpt_bpt_phuong_trinh_bat_phuong_trinh_sieu_viet
 
Xuctu.com de thi_tuyen_sinh_10_lhp_tdn_chuyen_dhsp(hcm)
Xuctu.com de thi_tuyen_sinh_10_lhp_tdn_chuyen_dhsp(hcm)Xuctu.com de thi_tuyen_sinh_10_lhp_tdn_chuyen_dhsp(hcm)
Xuctu.com de thi_tuyen_sinh_10_lhp_tdn_chuyen_dhsp(hcm)
 
Xuctu.com tuyen chon_cac_chuyen_de_tren_tap_chi_toan_hoc_va_tuoi_tre
Xuctu.com tuyen chon_cac_chuyen_de_tren_tap_chi_toan_hoc_va_tuoi_treXuctu.com tuyen chon_cac_chuyen_de_tren_tap_chi_toan_hoc_va_tuoi_tre
Xuctu.com tuyen chon_cac_chuyen_de_tren_tap_chi_toan_hoc_va_tuoi_tre
 
Xuctu.com tuyen tap-de-thi-olympic-52-de-thi-va-giai-tap-1
Xuctu.com tuyen tap-de-thi-olympic-52-de-thi-va-giai-tap-1Xuctu.com tuyen tap-de-thi-olympic-52-de-thi-va-giai-tap-1
Xuctu.com tuyen tap-de-thi-olympic-52-de-thi-va-giai-tap-1
 
Xuctu.com ch de-cuctri-gtln-gtnn
Xuctu.com ch de-cuctri-gtln-gtnnXuctu.com ch de-cuctri-gtln-gtnn
Xuctu.com ch de-cuctri-gtln-gtnn
 

Xuctu.com chuyen-de-so-hoc-vmf

  • 1. Chuyên · SÈ HÅC Di¹n àn Toán håc
  • 2.
  • 3. Chuy¶n · SÈ HÅC Ch¸ b£n Tr¦n Quèc Nhªt H¥n [perfectstrong] Tr¦n Trung Ki¶n [Ispectorgadget] Ph¤m Quang To n [Ph¤m Quang To n] L¶ Húu i·n Khu¶ [Nesbit] inh Ngåc Th¤ch [T*genie*] c 2012 Di¹n  n To¡n håc
  • 4.
  • 5. Líi giîi thi»u B¤n åc th¥n m¸n, Sè håc l  mët ph¥n mæn quan trång trong to¡n håc ¢ g­n bâ vîi chóng ta xuy¶n suèt qu¡ tr¼nh håc To¡n tø bªc tiºu håc ¸n trung håc phê thæng. Chóng ta ÷ñc ti¸p xóc vîi Sè håc b­t ¦u b¬ng nhúng kh¡i ni»m ìn gi£n nh÷ t½nh chia h¸t, ÷îc chung lîn nh§t, bëi chung nhä nh§t... gióp l m quen d¹ d ng hìn vîi sü k¼ di»u cõa nhúng con sè cho ¸n nhúng v§n · ái häi nhi·u t÷ duy hìn nh÷ çng d÷, sè nguy¶n tè, c¡c ph÷ìng tr¼nh Diophantine m  nêi ti¸ng nh§t l  ành lþ lîn Fermat..., ¥u ¥u tø t¦m vi mæ ¸n v¾ mæ, tø cªu b² lîp mët bi bæ 4 chia h¸t cho 2 ¸n Gi¡o s÷ thi¶n t i Andrew Wiles (ng÷íi gi£i quy¸t b i to¡n Fermat), chóng ta ·u câ thº th§y ÷ñc hìi thð cõa Sè håc trong â. Sè håc quan trång nh÷ vªy nh÷ng l¤ thay sè chuy¶n · vi¸t v· nâ l¤i khæng nhi·u n¸u em so vîi kho t ng ç së c¡c b i vi¸t v· b§t ¯ng thùc tr¶n c¡c di¹n  n m¤ng. Xu§t ph¡t tø sü thi¸u höt â công nh÷ º k¿ ni»m trán mët n«m Di¹n  n To¡n håc khai tr÷ìng trang chõ mîi (16/01/2012 - 16/01/2013), nhâm bi¶n tªp chóng tæi còng vîi nhi·u th nh vi¶n t½ch cüc cõa di¹n  n ¢ chung tay bi¶n so¤n mët chuy¶n · gûi ¸n b¤n åc. Chuy¶n · l  tªp hñp c¡c b i vi¸t ri¶ng l´ cõa c¡c t¡c gi£ Nguy¹n M¤nh Tròng D÷ìng (duongld) , Nguy¹n Tr¦n Huy (yeutoan11), Nguy¹n Trung Hi¸u (nguyentrunghieua), Ph¤m Quang To n (Ph¤m Quang To n), Tr¦n Nguy¹n Thi¸t Qu¥n (L Lawliet), Tr¦n Trung Ki¶n (Is-pectorgadget), Nguy¹n ¼nh Tòng (tungc3sp)... còng sü gâp sùc i
  • 6. ii gi¡n ti¸p cõa nhi·u th nh vi¶n t½ch cüc tr¶n Di¹n  n To¡n håc nh÷ Nguyen Lam Thinh, nguyenta98, Karl Heinrich Marx, The Gunner, perfectstrong... Ki¸n thùc · cªp trong chuy¶n · tuy khæng mîi nh÷ng câ thº gióp c¡c b¤n ph¦n n o hiºu s¥u hìn mët sè kh¡i ni»m cì b£n trong Sè håc công nh÷ trao êi còng c¡c b¤n nhi·u d¤ng b i tªp hay v  khâ tø c§p ë d¹ ¸n c¡c b i to¡n trong c¡c k¼ thi Håc sinh giäi quèc gia, quèc t¸. Chuy¶n · gçm 7 ch÷ìng. Ch÷ìng 1 · cªp ¸n c¡c kh¡i ni»m v· ×îc v  Bëi. Sè nguy¶n tè v  mët sè b i to¡n v· nâ ÷ñc giîi thi»u trong ch÷ìng 2. Ch÷ìng 3 nâi s¥u hìn v· C¡c b i to¡n chia h¸t. Ph÷ìng tr¼nh nghi»m nguy¶n, Ph÷ìng tr¼nh çng d÷ ÷ñc ph¡c håa trong c¡c ch÷ìng 4 v  5. H» th°ng d÷ v  ành lþ Th°ng d÷ Trung Hoa s³ ÷ñc gûi ¸n chóng ta qua ch÷ìng 6 tr÷îc khi k¸t thóc chuy¶n · b¬ng Mët sè b i to¡n sè håc hay tr¶n VMF ð ch÷ìng 7. Do thíi gian chu©n bà g§p rót nëi dung chuy¶n · ch÷a ÷ñc ¦u t÷ thªt sü t¿ m¿ công nh÷ câ thº cán nhi·u sai sât trong c¡c b i vi¸t, chóng tæi mong b¤n åc thæng c£m. Måi sü õng hë, âng gâp, ph¶ b¼nh cõa ëc gi£ s³ l  nguçn ëng vi¶n tinh th¦n to lîn cho ban bi¶n tªp công nh÷ cho c¡c t¡c gi£ º nhúng phi¶n b£n cªp nhªt sau cõa chuy¶n · ÷ñc tèt hìn, âng gâp nhi·u hìn núa cho kho t ng håc thuªt cõa cëng çng to¡n m¤ng. Chóng tæi hi vång qua chuy¶n · n y s³ gióp c¡c b¤n t¼m th¶m ÷ñc c£m hùng trong sè håc v  th¶m y¶u v´ µp cõa nhúng con sè. Måi trao êi gây þ xin gûi v· àa ch¿ email : contact@diendantoanhoc.net. Tr¥n trång, Nhâm bi¶n tªp Chuy¶n · Sè håc. Di¹n  n To¡n håc Chuy¶n · Sè håc
  • 7. Möc löc i Líi giîi thi»u 1 Ch÷ìng 1 ×îc v  Bëi 1.1 ×îc sè, ÷îc sè chung, ÷îc sè chung lîn nh§t 1 1.2 Bëi sè, bëi sè chung, bëi sè chung nhä nh§t 4 1.3 B i tªp · nghà 6 9 Ch÷ìng 2 Sè Nguy¶n Tè 2.1 Mët sè ki¸n thùc cì b£n v· sè nguy¶n tè 9 2.2 Mët sè b i to¡n cì b£n v· sè nguy¶n tè 13 2.3 B i tªp 19 2.4 Phö löc: B¤n n¶n bi¸t 24 29 Ch÷ìng 3 B i to¡n chia h¸t 3.1 Lþ thuy¸t cì b£n 29 3.2 Ph÷ìng ph¡p gi£i c¡c b i to¡n chia h¸t 31 57 Ch÷ìng 4 Ph÷ìng tr¼nh nghi»m nguy¶n iii
  • 8. iv Möc löc 4.1 X²t t½nh chia h¸t 57 4.2 Sû döng b§t ¯ng thùc 74 4.3 Nguy¶n t­c cüc h¤n, lòi væ h¤n 86 89 Ch÷ìng 5 Ph÷ìng tr¼nh çng d÷ 5.1 Ph÷ìng tr¼nh çng d÷ tuy¸n t½nh 89 5.2 Ph÷ìng tr¼nh çng d÷ bªc cao 90 5.3 H» ph÷ìng tr¼nh çng d÷ bªc nh§t mët ©n 90 5.4 Bªc cõa ph÷ìng tr¼nh çng d÷ 95 5.5 B i tªp 95 5.6 Ùng döng ành lþ Euler º gi£i ph÷ìng tr¼nh çng d÷ 96 5.7 B i tªp 101 103 Ch÷ìng 6 H» th°ng d÷ v  ành lþ Th°ng d÷ Trung Hoa 6.1 Mët sè k½ hi»u sû döng trong b i vi¸t 103 6.2 H» th°ng d÷ 104 6.3 ành l½ th°ng d÷ Trung Hoa 117 6.4 B i tªp · nghà gñi þ ¡p sè 125 129 Ch÷ìng 7 Mët sè b i to¡n sè håc hay tr¶n VMF 7.1 m3 + 17 ... 3n 129 7.2 c(ac + 1)2 = (5c + 2)(2c + b) 136 141 T i li»u tham kh£o Di¹n  n To¡n håc Chuy¶n · Sè håc
  • 9. Ch÷ìng 1 ×îc v  Bëi 1.1 ×îc sè, ÷îc sè chung, ÷îc sè chung lîn nh§t 1 1.2 Bëi sè, bëi sè chung, bëi sè chung nhä nh§t 4 1.3 B i tªp · nghà 6 Nguy¹n M¤nh Tròng D÷ìng (duongld) Nguy¹n Tr¦n Huy (yeutoan11) ×îc v  bëi l  2 kh¡i ni»m quan trång trong ch÷ìng tr¼nh sè håc THCS. Chuy¶n · n y s³ giîi thi»u nhúng kh¡i ni»m v  t½nh ch§t cì b£n v· ÷îc, ÷îc sè chung, ÷îc chung lîn nh§t, bëi, bëi sè chung, bëi chung nhä nh§t. Mët sè b i tªp · nghà v· c¡c v§n · n y công s³ ÷ñc · cªp ¸n ð cuèi b i vi¸t. 1.1 ×îc sè, ÷îc sè chung, ÷îc sè chung lîn nh§t Trong ph¦n n y, chóng tæi s³ tr¼nh b y mët sè kh¡i ni»m v· ÷îc sè, ÷îc sè chung v  ÷îc sè chung lîn nh§t k±m theo mët v i t½nh ch§t cõa chóng. Mët sè b i tªp v½ dö cho b¤n åc tham kh£o công s³ ÷ñc ÷a ra. 1.1.1 ành ngh¾a ành ngh¾a 1.1 Sè tü nhi¶n d6= 0 ÷ñc gåi l  mët ÷îc sè cõa sè tü nhi¶n a khi v  ch¿ khi a chia h¸t cho d. Ta nâi d chia h¸t a, k½ hi»u dja. Tªp hñp c¡c ÷îc cõa a l : U(a) = fd 2 N : djag. 4 1
  • 10. 2 1.1. ×îc sè, ÷îc sè chung, ÷îc sè chung lîn nh§t T½nh ch§t 1.1 N¸u U(a) = f1; ag th¼ a l  sè nguy¶n tè. ành ngh¾a 1.2 N¸u U(a) v  U(b) câ nhúng ph¦n tû chung th¼ nhúng ph¦n tû â gåi l  ÷îc sè chung cõa a v  b. Ta k½ hi»u: USC(a; b) = fd 2 N : (dja) ^ (djb)g = fd 2 N : (d 2 U(a)) ^ (d 2 U(b))g: T½nh ch§t 1.2 N¸u USC(a; b) = f1g th¼ a v  b nguy¶n tè còng nhau. ành ngh¾a 1.3 Sè d 2 N ÷ñc gåi l  ÷îc sè chung lîn nh§t cõa a v  b (a; b 2 Z) khi d l  ph¦n tû lîn nh§t trong tªp USC(a; b). Kþ hi»u ÷îc chung lîn nh§t cõa a v  b l  UCLN(a; b), (a; b) hay gcd(a; b). 4 1.1.2 T½nh ch§t Sau ¥y l  mët sè t½nh ch§t cõa ÷îc chung lîn nh§t: N¸u (a1; a2; : : : :; an) = 1 th¼ ta nâi c¡c sè a1; a2; : : : ; an nguy¶n tè còng nhau. N¸u (am; ak) = 1; 8m6= k; fm; kg 2 f1; 2; : : : ; ng th¼ ta nâi c¡c a1; a2; : : : ; an æi mët nguy¶n tè còng nhau. c 2 USC(a; b) th¼ a c ; b c = (a; b) c . d = (a; b) , a d ; b d = 1. (ca; cb) = c(a; b). (a; b) = 1 v  bjac th¼ bjc. (a; b) = 1 v  (a; c) = 1 th¼ (a; bc) = 1. (a; b; c) = ((a; b); c). Cho a b 0 N¸u a = b:q th¼ (a; b) = b. N¸u a = bq + r(r6= 0) th¼ (a; b) = (b; r). Di¹n  n To¡n håc Chuy¶n · Sè håc
  • 11. 1.1. ×îc sè, ÷îc sè chung, ÷îc sè chung lîn nh§t 3 1.1.3 C¡ch t¼m ÷îc chung lîn nh§t b¬ng thuªt to¡n Euclide º t¼m (a; b) khi a khæng chia h¸t cho b ta dòng thuªt to¡n Euclide sau: a = b:q + r1 th¼ (a; b) = (b; r1). b = r1:q1 + r2 th¼ (b; r1) = (r1; r2). rn2 = rn1:qn1 + rn th¼ (rn2; rn1) = (rn1; rn). rn1 = rn:qn th¼ (rn1; rn) = rn. (a; b) = rn. (a; b) l  sè d÷ cuèi còng kh¡c 0 trong thuªt to¡n Euclide. 1.1.4 B i tªp v½ dö V½ dö 1.1. T¼m (2k 1; 9k + 4); k 2 N. 4 Líi gi£i. Ta °t d = (2k 1; 9k + 4). Theo t½nh ch§t v· ÷îc sè chung ta câ dj2k1 v  dj9k+4. Ti¸p töc ¡p döng t½nh ch§t v· chia h¸t ta l¤i câ dj9(2k 1) v  dj2(9k + 4). Suy ra dj2(9k + 4) 9(2k 1) hay dj17. Vªy (2k 1; 9k + 4) = 1. V½ dö 1.2. T¼m (123456789; 987654321). 4 Líi gi£i. °t b = 123456789; a = 987654321. Ta nhªn th§y a v  b ·u chia h¸t cho 9. Ta l¤i câ : a + b = 1111111110 = 1010 10 9 : , 9a + 9b = 1010 10 (1.1) M°t kh¡c : 10b + a = 9999999999 = 1010 1: (1.2) Chuy¶n · Sè håc Di¹n  n To¡n håc
  • 12. 4 1.2. Bëi sè, bëi sè chung, bëi sè chung nhä nh§t Trø (1.2) v  (1.1) v¸ theo v¸ ta ÷ñc b8a = 9. Do â n¸u °t d = (a; b) th¼ 9 .. .d. M  a v  b ·u chia h¸t cho 9, suy ra d = 9. Düa v o thuªt to¡n Euclide, ta câ líi gi£i kh¡c cho V½ dö 1.2 nh÷ sau : Líi gi£i. 987654321 = 123456789:8+9 th¼ (987654321; 123456789) = (123456789; 9). 123456789 = 9:1371421. (123456789; 987654321) = 9. V½ dö 1.3. Chùng minh r¬ng d¢y sè An = 1 2 n(n + 1); n 2 N chùa nhúng d¢y sè væ h¤n nhúng sè æi mët nguy¶n tè còng nhau. 4 Líi gi£i. Gi£ sû trong d¢y ang x²t câ k sè æi mët nguy¶n tè còng nhau l  t1 = 1; t2 = 3; : : : ; tk = m(m 2 N). °t a = t1t2: : : tk. X²t sè h¤ng t2a+1 trong d¢y An: t2a+1 = 1 2 (2a + 1)(2a + 2) = (a + 1)(2a + 1) tk M°t kh¡c ta câ (a + 1; a) = 1 v  (2a + 1; a) = 1 n¶n (t2a+1; a) = 1. Do â t2a+1 nguy¶n tè còng nhau vîi t§t c£ k sè ft1; t2; : : : tkg. Suy ra d¢y sè An chùa væ h¤n nhúng sè æi mët nguy¶n tè còng nhau. 1.2 Bëi sè, bëi sè chung, bëi sè chung nhä nh§t T÷ìng tü nh÷ c§u tróc ¢ tr¼nh b y ð ph¦n tr÷îc, trong ph¦n n y chóng tæi công s³ ÷a ra nhúng ành ngh¾a, t½nh ch§t cì b£n cõa bëi sè, bëi sè chung, bëi sè chung nhä nh§t v  mët sè b i tªp v½ dö minh håa. Di¹n  n To¡n håc Chuy¶n · Sè håc
  • 13. 1.2. Bëi sè, bëi sè chung, bëi sè chung nhä nh§t 5 1.2.1 ành ngh¾a ành ngh¾a 1.4 Sè tü nhi¶n m ÷ñc gåi l  mët bëi sè cõa a6= 0 khi v  ch¿ khi m chia h¸t cho a hay a l  mët ÷îc sè cõa m. 4 Nhªn x²t. Tªp hñp c¡c bëi sè cõa a6= 0 l : B(a) = f0; a; 2a; : : : ; kag; k 2 Z. ành ngh¾a 1.5 Sè tü nhi¶n m ÷ñc gåi l  mët bëi sè cõa a6= 0 khi v  ch¿ khi m chia h¸t cho a hay a l  mët ÷îc sè cõa m 4 ành ngh¾a 1.6 N¸u 2 tªp B(a) v  B(b) câ ph¦n tû chung th¼ c¡c ph¦n tû chung â gåi l  bëi sè chung cõa a v  b. Ta kþ hi»u bëi sè chung cõa a v  b: BSC(a; b). ành ngh¾a 1.7 Sè m6= 0 ÷ñc gåi l  bëi chung nhä nh§t cõa a v  b khi m l  ph¦n tû d÷ìng nhä nh§t trong tªp BSC(a; b). Kþ hi»u : BCNN(a; b), [a; b] hay lcm(a; b). 4 1.2.2 T½nh ch§t Mët sè t½nh ch§t cõa bëi chung lîn nh§t: N¸u [a; b] = M th¼ M a ; M b = 1. [a; b; c] = [[a; b]; c]. [a; b]:(a; b) = a:b. 1.2.3 B i tªp v½ dö V½ dö 1.4. T¼m [n; n + 1; n + 2]. 4 Líi gi£i. °t A = [n; n + 1] v  B = [A; n + 2]. p döng t½nh ch§t [a; b; c] = [[a; b]; c], ta câ: B = [n; n + 1; n + 2]. D¹ th§y (n; n + 1) = 1, suy ra [n; n + 1] = n(n + 1). Chuy¶n · Sè håc Di¹n  n To¡n håc
  • 14. 6 1.3. B i tªp · nghà L¤i ¡p döng t½nh ch§t [a; b] = a:b (a; b) th¸ th¼ [n; n + 1; n + 2] = n(n + 1)(n + 2) (n(n + 1); n + 2) . Gåi d = (n(n + 1); n + 2). Do (n + 1; n + 2) = 1 n¶n d = (n; n + 2) = (n; 2): X²t hai tr÷íng hñp: N¸u n ch®n th¼ d = 2, suy ra [n; n + 1; n + 2] = n(n + 1)(n + 2) 2 . N¸u n l´ th¼ d = 1, suy ra [n; n + 1; n + 2] = n(n + 1)(n + 2) . V½ dö 1.5. Chùng minh r¬ng [1; 2; : : : 2n] = [n + 1; n + 2; : : : ; 2n]. 4 Líi gi£i. Ta th§y ÷ñc trong k sè nguy¶n li¶n ti¸p câ mët v  ch¿ mët sè chia h¸t cho k. Do â b§t trong c¡c sè f1; 2; : : : ; 2ng ·u l  ÷îc cõa mët sè n o â trong c¡c sè fn + 1; n + 2; : : : ; 2ng. Do â [1; 2; : : : n; 2n] = [n + 1; n + 2; : : : ; 2n]. 1.3 B i tªp · nghà Thay cho líi k¸t, chóng tæi xin gûi ¸n b¤n åc mët sè b i tªp · nghà º luy»n tªp nh¬m gióp c¡c b¤n quen hìn vîi c¡c kh¡i ni»m v  c¡c t½nh ch§t tr¼nh b y trong chuy¶n ·. B i 1. a: Cho A = 5a + 3b;B = 13a + 8b(a; b 2 N) chùng minh (A;B) = (a; b). b: Têng qu¡t A = ma+nb;B = pa+qb thäa m¢n jmqnpj = 1 vîi a; b; m; n; p; q 2 N. Chùng minh (A;B) = (a; b). B i 2. T¼m (6k + 5; 8k + 3)(k 2 N). Di¹n  n To¡n håc Chuy¶n · Sè håc
  • 15. 1.3. B i tªp · nghà 7 B i 3. Tø c¡c chú sè 1; 2; 3; 4; 5; 6 th nh lªp t§t c£ sè câ s¡u chú sè (méi sè ch¿ vi¸t mët l¦n). T¼m UCLN cõa t§t c£ c¡c sè â. B i 4. Cho A = 2n + 1;B = n(n + 1) 2 (n 2 N). T¼m (A;B). B i 5. a: Chùng minh r¬ng trong 5 sè tü nguy¶n li¶n ti¸p bao gií công chån ÷ñc mët sè nguy¶n tè còng nhau vîi c¡c sè cán l¤i. b: Chùng minh r¬ng trong 16 sè nguy¶n li¶n ti¸p bao gií công chån ÷ñc mët sè nguy¶n tè còng nhau vîi c¡c sè cán l¤i. B i 6. Cho 1 m n(m; n 2 N). a: Chùng minh r¬ng (22n 1; 22n + 1) = 1. b: T¼m (2m 1; 2n 1). B i 7. Cho m; n 2 N vîi (m; n) = 1. T¼m (m2 + n2;m + n). B i 8. Cho A = 2n+3;B = 2n+1+3n+1(n 2 N);C = 2n+2+3n+2(n 2 N). T¼m (A;B) v  (A;C). B i 9. Cho s¡u sè nguy¶n d÷ìng a; b; a0; b0; d; d0 sao cho (a; b) = d; (a0; b0) = d0. Chùng minh r¬ng (aa0; bb0; ab0; a0b) = dd0. B i 10. Chùng minh r¬ng d¢y sè Bn = 1 6 n(n + 1)(n + 2)(n 2 N) chùa væ h¤n nhúng sè nguy¶n tè còng nhau. B i 11. Chùng minh r¬ng d¢y sè 2n 3 vîi måi n 2 N v  n 2 chùa d¢y sè væ h¤n nhúng sè nguy¶n tè còng nhau. B i 12. Chùng minh d¢y Mersen Mn = 2n 1(n 2 N) chùa d¢y sè væ h¤n nhúng sè nguy¶n tè còng nhau. B i 13. Chùng minh r¬ng d¢y Fermat Fn = 22n + 1(n 2 N) l  d¢y sè nguy¶n tè còng nhau. B i 14. Cho n 2 N; n 1 v  2n 2 chia h¸t cho n. T¼m (22n ; 2n 1). Chuy¶n · Sè håc Di¹n  n To¡n håc
  • 16. 8 1.3. B i tªp · nghà B i 15. Chùng minh r¬ng vîi måi n 2 N, ph¥n sè 21n + 1 14n + 3 tèi gi£n. B i 16. Cho ba sè tü nhi¶n a; b; c æi mët nguy¶n tè còng nhau. Chùng minh r¬ng (ab + bc + ca; abc) = 1. B i 17. Cho a; b 2 N. Chùng minh r¬ng tçn t¤i væ sè n 2 N sao cho (a + n; b + n) = 1. B i 18. Gi£ sû m; n 2 N(m n) thäa m¢n (199k1;m) = (19931; n). Chùng minh r¬ng tçn t¤i t(t 2 N) sao cho m = 1993t:n. B i 19. Chùng minh r¬ng n¸u a;m 2 N; a 1 th¼ am 1 a 1 ; a 1 = (m; a 1). B i 20. T¼m sè nguy¶n d÷ìng n nhä nh§t º c¡c ph¥n sè sau tèi gi£n: a: 1 n1996 + 1995n + 2 , b: 2 n1996 + 1995n + 3 , c: 1994 n1996 + 1995n + 1995 , d: 1995 n1996 + 1995n + 1996 . B i 21. Cho 20 sè tü nhi¶n kh¡c 0 l  a1; a2; : : : an câ têng b¬ng S v  UCLN b¬ng d. Chùng minh r¬ng UCLN cõa S a1; S a2; : : : ; S an b¬ng t½ch cõa d vîi mët ÷îc n o â cõa n 1. Di¹n  n To¡n håc Chuy¶n · Sè håc
  • 17. Ch÷ìng 2 Sè Nguy¶n Tè 2.1 Mët sè ki¸n thùc cì b£n v· sè nguy¶n tè 9 2.2 Mët sè b i to¡n cì b£n v· sè nguy¶n tè 13 2.3 B i tªp 19 2.4 Phö löc: B¤n n¶n bi¸t 24 Nguy¹n Trung Hi¸u (nguyentrunghieua) Ph¤m Quang To n (Ph¤m Quang To n) 2.1 Mët sè ki¸n thùc cì b£n v· sè nguy¶n tè 2.1.1 ành ngh¾a, ành lþ cì b£n ành ngh¾a 2.1 Sè nguy¶n tè l  nhúng sè tü nhi¶n lîn hìn 1, ch¿ câ 2 ÷îc sè l  1 v  ch½nh nâ. 4 ành ngh¾a 2.2 Hñp sè l  sè tü nhi¶n lîn hìn 1 v  câ nhi·u hìn 2 ÷îc. 4 Nhªn x²t. C¡c sè 0 v  1 khæng ph£i l  sè nguy¶n tè công khæng ph£i l  hñp sè. B§t ký sè tü nhi¶n lîn hìn 1 n o công câ ½t nh§t mët ÷îc sè nguy¶n tè. ành lþ 2.1 D¢y sè nguy¶n tè l  d¢y sè væ h¤n. 9
  • 18. 10 2.1. Mët sè ki¸n thùc cì b£n v· sè nguy¶n tè Chùng minh. Gi£ sû ch¿ câ húu h¤n sè nguy¶n tè l  p1; p2; p3; :::; pn; trong â pn l  sè lîn nh§t trong c¡c nguy¶n tè. X²t sè N = p1p2:::pn + 1 th¼ N chia cho méi sè nguy¶n tè pi(i = 1; n) ·u d÷ 1 (*) M°t kh¡c N l  mët hñp sè (v¼ nâ lîn hìn sè nguy¶n tè lîn nh§t l  pn) do â N ph£i câ mët ÷îc nguy¶n tè n o â, tùc l  N chia h¸t cho mët trong c¡c sè pi (**). Ta th§y (**) m¥u thu¨n (*). Vªy khæng thº câ húu h¤n sè nguy¶n tè. ành lþ 2.2 Måi sè tü nhi¶n lîn hìn 1 ·u ph¥n t½ch ÷ñc ra thøa sè nguy¶n tè mët c¡ch duy nh§t (khæng kº thù tü c¡c thøa sè). Chùng minh. * Måi sè tü nhi¶n lîn hìn 1 ·u ph¥n t½ch ÷ñc ra thøa sè nguy¶n tè: Thªt vªy: gi£ sû i·u kh¯ng ành tr¶n l  óng vîi måi sè m tho£ m¢n: 1 m n ta chùng minh i·u â óng ¸n n. N¸u n l  nguy¶n tè, ta câ i·u ph£i chùng minh. N¸u n l  hñp sè, theo ành ngh¾a hñp sè, ta câ: n = a:b (vîi a; b n) Theo gi£ thi¸t quy n¤p: a v  b l  t½ch c¡c thøa sè nhä hìn n n¶n n l  t½ch cu£ c¡c thøa sè nguy¶n tè. * Sü ph¥n t½ch l  duy nh§t: Gi£ sû måi sè m n ·u ph¥n t½ch ÷ñc ra thøa sè nguy¶n tè mët c¡ch duy nh§t, ta chùng minh i·u â óng ¸n n: N¸u n l  sè nguy¶n tè th¼ ta ÷ñc i·u ph£i chùng minh. N¸u n l  hñp sè: Gi£ sû câ 2 c¡ch ph¥n t½ch n ra thøa sè nguy¶n tè kh¡c nhau: n = p:q:r:::: n = p0:q0:r0:::: Trong â p; q; r::::: v  p0; q0; r0:::: l  c¡c sè nguy¶n tè v  khæng câ sè nguy¶n tè n o công câ m°t trong c£ hai ph¥n t½ch â (v¼ n¸u câ sè tho£ m¢n i·u ki»n nh÷ tr¶n, ta câ thº chia n cho sè â lóc â th÷íng s³ nhä hìn n, th÷ìng n y câ hai c¡ch ph¥n t½ch ra thøa sè nguy¶n tè kh¡c nhau, tr¡i vîi gi£ thi¸t cõa quy n¤p). Khæng m§t t½nh têng qu¡t, ta câ thº gi£ thi¸t p v  p0 l¦n l÷ñt l  c¡c sè nguy¶n tè nhä nh§t trong ph¥n t½ch thù nh§t v  thù hai. V¼ n l  hñp sè n¶n n p2 v  n p02. Do p6= p ) n p:p0 Di¹n  n To¡n håc Chuy¶n · Sè håc
  • 19. 2.1. Mët sè ki¸n thùc cì b£n v· sè nguy¶n tè 11 X²t m = n pp0 n ÷ñc ph¥n t½ch ra thøa sè nguy¶n tè mët c¡ch duy nh§t ta th§y: pjn ) pjn pp0 hay pjm Khi ph¥n t½ch ra thøa sè nguy¶n tè ta câ: m = npp0 = p0p:P:Q::: vîi P;Q 2 P ( P l  tªp c¡c sè nguy¶n tè). ) pp0jn ) pp0jp:q:r::: ) pjq:r::: ) p l  ÷îc nguy¶n tè cõa q:r::: M  p khæng tròng vîi mët thøa sè n o trong q; r::: (i·u n y tr¡i vîi g¿a thi¸t quy n¤p l  måi sè nhä hìn n ·u ph¥n t½ch ÷ñc ra thøa sè nguy¶n tè mët c¡ch duy nh§t). Vªy, i·u gi£ sû khæng óng. ành lþ ÷ñc chùng minh. 2.1.2 C¡ch nhªn bi¸t mët sè nguy¶n tè C¡ch 1 Chia sè â l¦n l÷ñt cho c¡c nguy¶n tè tø nhä ¸n lîn: 2; 3; 5; 7::: N¸u câ mët ph²p chia h¸t th¼ sè â khæng nguy¶n tè. N¸u thüc hi»n ph²p chia cho ¸n lóc th÷ìng sè nhä hìn sè chia m  c¡c ph²p chia v¨n câ sè d÷ th¼ sè â l  nguy¶n tè. C¡ch 2 Mët sè câ hai ÷îc sè lîn hìn 1 th¼ sè â khæng ph£i l  sè nguy¶n tè. Cho håc sinh lîp 6 håc c¡ch nhªn bi¸t 1 sè nguy¶n tè b¬ng ph÷ìng ph¡p thù nh§t (n¶u ð tr¶n), l  düa v o ành lþ cì b£n: ×îc sè nguy¶n tè nhä nh§t cõa mët hñp sè A l  mët sè khæng v÷ñt p qu¡ A. Vîi quy t­c tr¶n trong mët kho£n thíi gian ng­n, vîi c¡c d§u hi»u chia h¸t th¼ ta nhanh châng tr£ líi ÷ñc mët sè câ hai chú sè n o â l  Chuy¶n · Sè håc Di¹n  n To¡n håc
  • 20. 12 2.1. Mët sè ki¸n thùc cì b£n v· sè nguy¶n tè nguy¶n tè hay khæng. H» qu£ 2.1 N¸u câ sè A 1 khæng câ mët ÷îc sè nguy¶n tè n o tø 2 ¸n p A th¼ A l  mët nguy¶n tè. 2.1.3 Sè c¡c ÷îc sè v  têng c¡c ÷îc sè cõa 1 sè Gi£ sû: A = px1 1 :px2 2 ::::::pnxn; trong â: pi 2 P; xi 2 N; i = 1; n T½nh ch§t 2.1 Sè c¡c ÷îc sè cõa A t½nh b¬ng cæng thùc: T(A) = (x1 + 1)(x2 + 1):::::(xn + 1) V½ dö 2.1. 30 = 2:3:5 th¼ T(A) = (1 + 1)(1 + 1)(1 + 1) = 8. Kiºm tra: (30) = f1; 2; 3; 5; 6; 10; 15; 30g n¶n (30) câ 8 ph¥n tû. 4 T½nh ch§t 2.2 Têng c¡c ÷îc mët sè cõa A t½nh b¬ng cæng thùc: (A) = Yn i=1 pxi+1 i 1 pi 1 2.1.4 Hai sè nguy¶n tè còng nhau ành ngh¾a 2.3 Hai sè tü nhi¶n ÷ñc gåi l  nguy¶n tè còng nhau khi v  ch¿ khi chóng câ ÷îc chung lîn nh§t (×CLN) b¬ng 1. 4 T½nh ch§t 2.3 Hai sè tü nhi¶n li¶n ti¸p luæn nguy¶n tè còng nhau. T½nh ch§t 2.4 Hai sè nguy¶n tè kh¡c nhau luæn nguy¶n tè còng nhau. T½nh ch§t 2.5 C¡c sè a; b; c nguy¶n tè còng nhau khi v  ch¿ khi (a; b; c) = 1. ành ngh¾a 2.4 Nhi·u sè tü nhi¶n ÷ñc gåi l  nguy¶n tè s¡nh æi khi chóng æi mët nguy¶n tè còng nhau. 4 Di¹n  n To¡n håc Chuy¶n · Sè håc
  • 21. 2.2. Mët sè b i to¡n cì b£n v· sè nguy¶n tè 13 2.1.5 Mët sè ành lþ °c bi»t ành lþ 2.3 (Dirichlet) Tçn t¤i væ sè sè nguy¶n tè p câ d¤ng: p = ax + b (x; a; b 2 N, a; b l  2 sè nguy¶n tè còng nhau). Vi»c chùng minh ành lþ n y kh¡ phùc t¤p, trø mët sè tr÷íng hñp °c bi»t, ch¯ng h¤n câ væ sè sè nguy¶n tè d¤ng: 2x1; 3x1; 4x+3; 6x+ 5; : : : ành lþ 2.4 (Tchebycheff-Betrand) Trong kho£ng tø sè tü nhi¶n n ¸n sè tü nhi¶n 2n câ ½t nh§t mët sè nguy¶n tè (n 2). ành lþ 2.5 (Vinogradow) Måi sè l´ lîn hìn 33 l  têng cõa 3 sè nguy¶n tè. 2.2 Mët sè b i to¡n cì b£n v· sè nguy¶n tè 2.2.1 Câ bao nhi¶u sè nguy¶n tè d¤ng ax + b V½ dö 2.2. Chùng minh r¬ng: câ væ sè sè nguy¶n tè câ d¤ng 3x 1.4 Líi gi£i. Måi sè tü nhi¶n khæng nhä hìn 2 câ 1 trong 3 d¤ng: 3x; 3x+1 ho°c 3x 1 Nhúng sè câ d¤ng 3x (vîi x 1) l  hñp sè X²t 2 sè câ d¤ng 3x + 1: â l  sè 3m + 1 v  sè 3n + 1. X²t t½ch (3m + 1)(3n + 1) = 9mn + 3m + 3n + 1. T½ch n y câ d¤ng: 3x + 1 L§y mët sè nguy¶n tè p b§t câ d¤ng 3x 1, ta lªp t½ch cõa p vîi t§t c£ c¡c sè nguy¶n tè nhä hìn p rçi trø i 1 ta câ: M = 2:3:5:7::::p 1 = 3(2:5:7::::p) 1 th¼ M câ d¤ng 3x 1. Câ 2 kh£ n«ng x£y ra: 1. Kh£ n«ng 1: M l  sè nguy¶n tè, â l  sè nguy¶n tè câ d¤ng 3x 1 p, b i to¡n ÷ñc chùng minh. Chuy¶n · Sè håc Di¹n  n To¡n håc
  • 22. 14 2.2. Mët sè b i to¡n cì b£n v· sè nguy¶n tè 2. Kh£ n«ng 2: M l  hñp sè: Ta chia M cho 2; 3; 5; ::::; p ·u tçn t¤i mët sè d÷ kh¡c 0 n¶n c¡c ÷îc nguy¶n tè cõa M ·u lîn hìn p, trong c¡c ÷îc n y khæng câ sè n o câ d¤ng 3x+1 (¢ chùng minh tr¶n). Do â ½t nh§t mët trong c¡c ÷îc nguy¶n tè cõa M ph£i câ d¤ng 3x (hñp sè) ho°c 3x + 1 V¼ n¸u t§t c£ câ d¤ng 3x+1 th¼ M ph£i câ d¤ng 3x+1 (¢ chùng minh tr¶n). Do â, ½t nh§t mët trong c¡c ÷îc nguy¶n tè cõa M ph£i câ d¤ng 3x 1, ÷îc n y luæn lîn hìn p. Vªy: Câ væ sè sè nguy¶n tè d¤ng 3x 1. V½ dö 2.3. Chùng minh r¬ng: Câ væ sè sè nguy¶n tè câ d¤ng 4x+3.4 Líi gi£i. Nhªn x²t. C¡c sè nguy¶n tè l´ khæng thº câ d¤ng 4x ho°c 4x + 2. Vªy chóng ch¿ câ thº tçn t¤i d÷îi 1 trong 2 d¤ng 4x + 1 ho°c 4x + 3. Ta s³ chùng minh câ væ sè sè nguy¶n tè câ d¤ng 4x + 3. X²t t½ch 2 sè câ d¤ng 4x + 1 l : 4m + 1 v  4n + 1. Ta câ: (4m+1)(4n+1) = 16mn+4m+4n+1 = 4(4mn+m+n)+1. Vªy t½ch cõa 2 sè câ d¤ng 4x + 1 l  mët sè công câ d¤ng 4x + 1. L§y mët sè nguy¶n tè p b§t ký câ d¤ng 4x+3, ta lªp t½ch cõa 4p vîi t§t c£ c¡c sè nguy¶n tè nhä hìn p rçi trø i 1 khi â ta câ: N = 4(2:3:5:7:::::p) 1. Câ 2 kh£ n«ng x£y ra 1. N l  sè nguy¶n tè ) N = 4(2:3:5:7::::p)1 câ d¤ng 4x1. Nhúng sè nguy¶n tè câ d¤ng 4x 1 công ch½nh l  nhúng sè câ d¤ng 4x + 3 v  b i to¡n ÷ñc chùng minh. 2. N l  hñp sè. Chia N cho 2; 3; 5; ::::; p ·u ÷ñc c¡c sè d÷ kh¡c 0. Suy ra c¡c ÷îc nguy¶n tè cõa N ·u lîn hìn p. C¡c ÷îc n y khæng thº câ d¤ng 4x ho°c 4x+2 (v¼ â l  hñp sè). Công khæng thº to n c¡c ÷îc câ d¤ng 4x + 1 v¼ nh÷ th¸ N ph£i câ d¤ng 4x + 1. Nh÷ vªy trong c¡c ÷îc nguy¶n tè cõa N câ ½t nh§t 1 ÷îc câ d¤ng 4x 1 m  ÷îc n y hiºn nhi¶n lîn hìn p. Di¹n  n To¡n håc Chuy¶n · Sè håc
  • 23. 2.2. Mët sè b i to¡n cì b£n v· sè nguy¶n tè 15 Vªy: Câ væ sè sè nguy¶n tè câ d¤ng 4x 1 (hay câ d¤ng 4x + 3). Tr¶n ¥y l  mët sè b i to¡n chùng minh ìn gi£n cõa ành lþ Dirichlet: Câ væ sè sè nguy¶n tè d¤ng ax + b trong â a; b; x 2 N; (a; b) = 1. 2.2.2 Chùng minh sè nguy¶n tè V½ dö 2.4. Chùng minh r¬ng: (p 1)! chia h¸t cho p n¸u p l  hñp sè, khæng chia h¸t cho p n¸u p l  sè nguy¶n tè. 4 Líi gi£i. X²t tr÷íng hñp p l  hñp sè: N¸u p l  hñp sè th¼ p l  t½ch cõa c¡c thøa sè nguy¶n tè nhä hìn p v  sè mô c¡c luÿ thøa n y khæng thº lîn hìn sè mô cõa ch½nh c¡c luÿ thøa §y chùa trong (p 1)!. Vªy: (p 1)! ... p (pcm). X²t tr÷íng hñp p l  sè nguy¶n tè: V¼ p 2 P ) p nguy¶n tè còng nhau vîi måi thøa sè cõa (p 1)! (pcm). V½ dö 2.5. Cho 2m 1 l  sè nguy¶n tè. Chùng minh r¬ng m công l  sè nguy¶n tè. 4 Líi gi£i. Gi£ sû m l  hñp sè ) m = p:q (p; q 2 N; p; q 1) Khi â: 2m1 = 2pq1 = (2p)q1 = (2p1)((2p)q1+(2p)q2+:::::+1) v¼ p 1 ) 2p 1 1 v  (2p)q1 + (2p)q2 + ::::: + 1 1 D¨n ¸n 2m 1 l  hñp sè :tr¡i vîi gi£ thi¸t 2m1 l  sè nguy¶n tè. Vªy m ph£i l  sè nguy¶n tè (pcm) V½ dö 2.6. Chùng minh r¬ng: måi ÷îc nguy¶n tè cõa 1994!1 ·u lîn hìn 1994. 4 Líi gi£i. Gåi p l  ÷îc sè nguy¶n tè cõa 1994! 1 Gi£ sû p 1994 ) 1994:1993:::::3:2:1 .. .p ) 1994! .. .p. M  1994! 1 .. .p ) 1 .. .p (væ lþ) Vªy: p 1994 (pcm). V½ dö 2.7. Chùng minh r¬ng: n 2 th¼ giúa n v  n! câ ½t nh§t 1 sè nguy¶n tè (tø â suy ra câ væ sè sè nguy¶n tè). 4 Chuy¶n · Sè håc Di¹n  n To¡n håc
  • 24. 16 2.2. Mët sè b i to¡n cì b£n v· sè nguy¶n tè Líi gi£i. V¼ n 2 n¶n k = n! 1 1, do â k câ ½t nh§t mët ÷îc sè nguy¶n tè p. T÷ìng tü b i tªp 3, ta chùng minh ÷ñc måi ÷îc nguy¶n tè p cõa k ·u lîn hìn k. Vªy: p n ) n p n! 1 n! (pcm) 2.2.3 T¼m sè nguy¶n tè thäa m¢n i·u ki»n cho tr÷îc V½ dö 2.8. T¼m t§t c£ c¡c gi¡ trà cõa sè nguy¶n tè p º: p + 10 v  p + 14 công l  sè nguy¶n tè. 4 Líi gi£i. N¸u p = 3 th¼ p + 10 = 3 + 10 = 13 v  p + 14 = 3 + 14 = 17 ·u l  c¡c sè nguy¶n tè n¶n p = 3 l  gi¡ trà c¦n t¼m. N¸u p 3 ) p câ d¤ng 3k + 1 ho°c d¤ng 3k 1 N¸u p = 3k + 1 th¼ p + 14 = 3k + 15 = 3(k + 5) .. .3 N¸u p = 3k 1 th¼ p + 10 = 3k + 9 = 3(k + 3) ... 3 Vªy n¸u p 3 th¼ ho°c p + 10 ho°c p + 14 l  hñp sè : khæng thäa m¢n b i. Vªy p = 3. V½ dö 2.9. T¼m k 2 N º trong 10 sè tü nhi¶n li¶n ti¸p: k + 1; k + 2; k + 3; ::::k + 10 câ nhi·u sè nguy¶n tè nh§t. 4 Líi gi£i. N¸u k = 0: tø 1 ¸n 10 câ 4 sè nguy¶n tè: 2; 3; 5; 7. N¸u k = 1: tø 2 ¸n 11 câ 5 sè nguy¶n tè: 2; 3; 5; 7; 11. N¸u k 1: tø 3 trð i khæng câ sè ch®n n o l  sè nguy¶n tè. Trong 5 sè l´ li¶n ti¸p, ½t nh§t câ 1 sè l  bëi sè cõa 3 do â, d¢y s³ câ ½t hìn 5 sè nguy¶n tè. Vªy vîi k = 1, d¢y t÷ìng ùng: k + 1; k + 2; :::::k + 10 câ chùa nhi·u sè nguy¶n tè nh§t (5 sè nguy¶n tè). V½ dö 2.10. T¼m t§t c£ c¡c sè nguy¶n tè p º: 2p+p2 công l  sè nguy¶n tè. 4 Líi gi£i. X²t 3 tr÷íng hñp: Di¹n  n To¡n håc Chuy¶n · Sè håc
  • 25. 2.2. Mët sè b i to¡n cì b£n v· sè nguy¶n tè 17 p = 2 ) 2p + p2 = 22 + 22 = 862 P p = 3 ) 2p + p2 = 23 + 32 = 17 2 P p 3 ) p6 .. .3. Ta câ 2p + p2 = (p2 1) + (2p + 1). V¼ p l´ ) 2p + 1 ... ... 3 v  p2 1 = (p + 1)(p 1) 3 ) 2p + p262 P Vªy câ duy nh§t 1 gi¡ trà p = 3 tho£ m¢n. V½ dö 2.11. T¼m t§t c£ c¡c sè nguy¶n tè p sao cho: pj2p + 1. 4 Líi gi£i. V¼ p 2 P : pj2p + 1 ) p 2 ) (2; p) = 1 Theo ành lþ Fermat, ta câ: pj2p1 1. M  pj2p + 1 ) pj2(2p1 1) + 3 ) pj3 ) p = 3 Vªy: p = 3. 2.2.4 Nhªn bi¸t sè nguy¶n tè V½ dö 2.12. N¸u p l  sè nguy¶n tè v  1 trong 2 sè 8p + 1 v  8p 1 l  sè nguy¶n tè th¼ sè cán l¤i l  sè nguy¶n tè hay hñp sè? 4 Líi gi£i. N¸u p = 2 ) 8p + 1 = 17 2 P; 8p 1 = 1562 P N¸u p = 3 ) 8p 1 = 23 2 P; 8p 1 = 2562 P N¸u p 3, x²t 3 sè tü nhi¶n li¶n ti¸p: 8p1; 8p v  8p+1. Trong 3 sè n y ­t câ 1 sè chia h¸t cho 3. N¶n mët trong hai sè 8p + 1 v  8p 1 chia h¸t cho 3. K¸t luªn: N¸u p 2 P v  1 trong 2 sè 8p + 1 v  8p 1 l  sè nguy¶n tè th¼ sè cán l¤i ph£i l  hñp sè. V½ dö 2.13. N¸u p 5 v  2p + 1 l  c¡c sè nguy¶n tè th¼ 4p + 1 l  nguy¶n tè hay hñp sè? 4 Líi gi£i. X²t 3 sè tü nhi¶n li¶n ti¸p: 4p; 4p + 1; 4p + 2. Trong 3 sè ­t câ mët sè l  bëi cõa 3. M  p 5; p 2 P n¶n p câ d¤ng 3k + 1 ho°c 3k + 2 N¸u p = 3k + 1 th¼ 2p + 1 = 6k + 3 .. .3: (tr¡i vîi gi£ thi¸t) Chuy¶n · Sè håc Di¹n  n To¡n håc
  • 26. 18 2.2. Mët sè b i to¡n cì b£n v· sè nguy¶n tè N¸u p = 3k+2. Khi â 4p+1 = 4(3k+2)+1 = 12k+9 .. .3 ) 4p+1 l  hñp sè V½ dö 2.14. Trong d¢y sè tü nhi¶n câ thº t¼m ÷ñc 1997 sè li¶n ti¸p nhau m  khæng câ sè nguy¶n tè n o hay khæng ? 4 Líi gi£i. Chån d¢y sè: (ai) : ai = 1998! + i + 1 (i = 1; 1997) ) ai .. .i + 1 8i = 1; 1997 Nh÷ vªy: D¢y sè a1; a2; a3; :::::a1997 gçm câ 1997 sè tü nhi¶n li¶n ti¸p khæng câ sè n o l  sè nguy¶n tè. V½ dö 2.15 (Têng qu¡t b i tªp 2.14). Chùng minh r¬ng câ thº t¼m ÷ñc 1 d¢y sè gçm n sè tü nhi¶n li¶n ti¸p (n 1) khæng câ sè n o l  sè nguy¶n tè ? 4 Líi gi£i. Ta chån d¢y sè sau: (ai) : ai = (n+1)!+i+1 ) ai .. .i+1 8i = 1; n. B¤n åc h¢y tü chùng minh d¢y (ai) ð tr¶n s³ gçm câ n sè tü nhi¶n li¶n ti¸p trong â khæng câ sè n o l  sè nguy¶n tè c£. 2.2.5 C¡c d¤ng kh¡c V½ dö 2.16. T¼m 3 sè nguy¶n tè sao cho t½ch cõa chóng g§p 5 l¦n têng cõa chóng. 4 Líi gi£i. Gåi 3 sè nguy¶n tè ph£i t¼m l  a; b; c. Ta câ: abc = 5(a + b + c) ) abc ... 5 V¼ a; b; c câ vai trá b¼nh ¯ng n¶n khæng m§t t½nh têng qu¡t, gi£ sû: a ... 5 ) a = 5 Khi â: 5bc = 5(5 + b + c) , 5 + b + c = bc , (c 1)(b 1) = 6 Do vªy: 2 664 b 1 = 1 c 1 = 6 , b = 2 c = 7 chån b 1 = 2 c 1 = 3 , b = 3 c = 4 lo¤i Vªy bë sè (a; b; c) c¦n t¼m l  ho¡n và cõa (2; 5; 7). V½ dö 2.17. T¼m p; q 2 P sao cho p2 = 8q + 1. 4 Di¹n  n To¡n håc Chuy¶n · Sè håc
  • 27. 2.3. B i tªp 19 Líi gi£i. Ta câ: p2 = 8q + 1 ) 8q = p2 1 = (p + 1)(p 1) (2.1) Do p2 = 8q + 1 : l´ ) p2 : l´ ) p : l´. °t p = 2k + 1. Thay v o (2.1) ta câ: 8q = 2k(2k + 2) ) 2q = k(k + 1) (2.2) N¸u q = 2 ) 4 = k(k + 1) ) khæng t¼m ÷ñc k 2 N Vªy q 2. V¼ q 2 P ) (2; q) = 1. Tø (2.2) ta câ: a) k = 2 v  q = k + 1 ) k = 2; q = 3. Thay k¸t qu£ tr¶n v o (2.2) ta câ: p = 2:2 + 1 = 5 b) q = k v  2 = k + 1 ) q = 1 :lo¤i. Vªy (q; p) = (5; 3). 2.3 B i tªp 2.3.1 B i tªp câ h÷îng d¨n B i 1. Ta bi¸t r¬ng câ 25 sè nguy¶n tè nhä hìn 100. Têng cõa 25 sè nguy¶n tè nhä hìn 100 l  sè ch®n hay sè l´? HD :Trong 25 sè nguy¶n tè nhä hìn 100 câ chùa mët sè nguy¶n tè ch®n duy nh§t l  2, cán 24 sè nguy¶n tè cán l¤i l  sè l´. Do â têng cõa 25 sè nguy¶n tè l  sè ch®n. B i 2. Têng cõa 3 sè nguy¶n tè b¬ng 1012. T¼m sè nguy¶n tè nhä nh§t trong ba sè nguy¶n tè â. HD: V¼ têng cõa 3 sè nguy¶n tè b¬ng 1012, n¶n trong 3 sè nguy¶n tè â tçn t¤i ½t nh§t mët sè nguy¶n tè ch®n. M  sè nguy¶n tè ch®n duy nh§t l  2 v  l  sè nguy¶n tè nhä nh§t. Vªy sè nguy¶n tè nhä nh§t trong 3 sè nguy¶n tè â l  2. Chuy¶n · Sè håc Di¹n  n To¡n håc
  • 28. 20 2.3. B i tªp B i 3. Têng cõa 2 sè nguy¶n tè câ thº b¬ng 2003 hay khæng? V¼ sao? HD: V¼ têng cõa 2 sè nguy¶n tè b¬ng 2003, n¶n trong 2 sè nguy¶n tè â tçn t¤i 1 sè nguy¶n tè ch®n. M  sè nguy¶n tè ch®n duy nh§t l  2. Do â sè nguy¶n tè cán l¤i l  2001. Do 2001 chia h¸t cho 3 v  2001 3. Suy ra 2001 khæng ph£i l  sè nguy¶n tè. B i 4. T¼m sè nguy¶n tè p, sao cho p + 2; p + 4 công l  c¡c sè nguy¶n tè. B i 5. Cho p v  p + 4 l  c¡c sè nguy¶n tè (p 3). Chùng minh r¬ng p + 8 l  hñp sè. HD: V¼ p l  sè nguy¶n tè v  p 3, n¶n sè nguy¶n tè p câ 1 trong 2 d¤ng: ... N¸u p = 3k +2 th¼ p+4 = 3k +6 = 3(k +2) ) p+4 3 v  p + 4 3. Do â p + 4 l  hñp sè: tr¡i · b i. ... N¸u p = 3k +1 th¼ p+8 = 3k +9 = 3(k +3) ) p+8 3 v  p + 8 3. Do â p + 8 l  hñp sè. B i 6. Chùng minh r¬ng måi sè nguy¶n tè lîn hìn 2 ·u câ d¤ng 4n+1 ho°c 4n 1. B i 7. T¼m sè nguy¶n tè, bi¸t r¬ng sè â b¬ng têng cõa hai sè nguy¶n tè v  b¬ng hi»u cõa hai sè nguy¶n tè. HD: Gi£ sû a; b; c; d; e l  c¡c sè nguy¶n tè v  d e. Theo · b i: a = b + c = d e () Tø (*) ) a 2 n¶n a l  sè nguy¶n tè l´ ) b+c; de l  sè l´. Do b; d l  c¡c sè nguy¶n tè ) b; d l  sè l´ ) c; e l  sè ch®n. ) c = e = 2 (do c; el  sè nguy¶n tè) ) a = b + 2 = d 2 ) d = b + 4. Vªy ta c¦n t¼m sè nguy¶n tè b sao cho b + 2 v  b + 4 công l  c¡c sè nguy¶n tè. Di¹n  n To¡n håc Chuy¶n · Sè håc
  • 29. 2.3. B i tªp 21 B i 8. T¼m t§t c£ c¡c sè nguy¶n tè x, y sao cho: x2 6y2 = 1. B i 9. Cho p v  p + 2 l  c¡c sè nguy¶n tè (p 3). Chùng minh r¬ng p + 1 ... 6. 2.3.2 B i tªp khæng câ h÷îng d¨n B i 1. T¼m sè nguy¶n tè p sao cho c¡c sè sau công l  sè nguy¶n tè: a) p + 2 v  p + 10. b) p + 10 v  p + 20. c) p + 10 v  p + 14. d) p + 14 v  p + 20. e) p + 2 v  p + 8. f) p + 2 v  p + 14. g) p + 4 v  p + 10. h) p + 8 v  p + 10. B i 2. T¼m sè nguy¶n tè p sao cho c¡c sè sau công l  sè nguy¶n tè: a) p + 2; p + 8; p + 12; p + 14 b) p + 2; p + 6; p + 8; p + 14 c) p + 6; p + 8; p + 12; p + 14 d) p + 2; p + 6; p + 8; p + 12; p + 14 e) p + 6; p + 12; p + 18; p + 24 f) p + 18; p + 24; p + 26; p + 32 g) p + 4; p + 6; p + 10; p + 12; p + 16 B i 3. Cho tr÷îc sè nguy¶n tè p 3 thäa a) p + 4 2 P. Chùng minh r¬ng: p + 8 l  hñp sè. b) 2p + 1 2 P. Chùng minh r¬ng: 4p + 1 l  hñp sè. c) 10p + 1 2 P. Chùng minh r¬ng: 5p + 1 l  hñp sè. Chuy¶n · Sè håc Di¹n  n To¡n håc
  • 30. 22 2.3. B i tªp d) p + 8 2 P. Chùng minh r¬ng: p + 4 l  hñp sè. e) 4p + 1 2 P. Chùng minh r¬ng: 2p + 1 l  hñp sè. f) 5p + 1 2 P. Chùng minh r¬ng: 10p + 1 l  hñp sè. g) 8p + 1 2 P. Chùng minh r¬ng: 8p 1 l  hñp sè. h) 8p 1 2 P. Chùng minh r¬ng: 8p + 1 l  hñp sè. i) 8p2 1 2 P. Chùng minh r¬ng: 8p2 + 1 l  hñp sè. j) 8p2 + 1 2 P. Chùng minh r¬ng: 8p2 1 l  hñp sè. B i 4. Chùng minh r¬ng: a) N¸u p v  q l  hai sè nguy¶n tè lîn hìn 3 th¼ p2 q2.. .24. b) N¸u a; a+k; a+2k(a; k 2 N) l  c¡c sè nguy¶n tè lîn hìn 3 th¼ k ... 6. B i 5. a) Mët sè nguy¶n tè chia cho 42 câ sè d÷ r l  hñp sè. T¼m sè d÷ r. b) Mët sè nguy¶n tè chia cho 30 câ sè d÷ r. T¼m sè d÷ r bi¸t r¬ng r khæng l  sè nguy¶n tè. B i 6. T¼m sè nguy¶n tè câ ba chú sè, bi¸t r¬ng n¸u vi¸t sè â theo thù tü ng÷ñc l¤i th¼ ta ÷ñc mët sè l  lªp ph÷ìng cõa mët sè tü nhi¶n. B i 7. T¼m sè tü nhi¶n câ 4 chú sè, chú sè h ng ngh¼n b¬ng chú sè h ng ìn và, chú sè h ng tr«m b¬ng chú sè h ng chöc v  sè â vi¸t ÷ñc d÷îi d¤ng t½ch cõa 3 sè nguy¶n tè li¶n ti¸p. B i 8. T¼m 3 sè nguy¶n tè l  c¡c sè l´ li¶n ti¸p. B i 9. T¼m 3 sè nguy¶n tè li¶n ti¸p p; q; r sao cho p2 + q2 + r2 2 P. B i 10. T¼m t§t c£ c¡c bë ba sè nguy¶n tè a; b; c sao cho abc ab + bc + ca. B i 11. T¼m 3 sè nguy¶n tè p; q; r sao cho pq + qp = r. Di¹n  n To¡n håc Chuy¶n · Sè håc
  • 31. 2.3. B i tªp 23 B i 12. T¼m c¡c sè nguy¶n tè x; y; z tho£ m¢n xy + 1 = z. B i 13. T¼m sè nguy¶n tè abcd thäa ab; ac l  c¡c sè nguy¶n tè v  b2 = cd + b c. B i 14. Cho c¡c sè p = bc + a; q = ab + c; r = ca + b(a; b; c 2 N) l  c¡c sè nguy¶n tè. Chùng minh r¬ng 3 sè p; q; r câ ½t nh§t hai sè b¬ng nhau. B i 15. T¼m t§t c£ c¡c sè nguy¶n tè x; y sao cho: a) x2 12y2 = 1 b) 3x2 + 1 = 19y2 c) 5x2 11y2 = 1 d) 7x2 3y2 = 1 e) 13x2 y2 = 3 f) x2 = 8y + 1 B i 16. Chùng minh r¬ng i·u ki»n c¦n v  õ º p v  8p2 +1 l  c¡c sè nguy¶n tè l  p = 3. B i 17. Chùng minh r¬ng: N¸u a2b2 l  mët sè nguy¶n tè th¼ a2b2 = a + b. B i 18. Chùng minh r¬ng måi sè nguy¶n tè lîn hìn 3 ·u câ d¤ng 6n+1 ho°c 6n 1. B i 19. Chùng minh r¬ng têng b¼nh ph÷ìng cõa 3 sè nguy¶n tè lîn hìn 3 khæng thº l  mët sè nguy¶n tè. B i 20. Cho sè tü nhi¶n n 2. Gåi p1; p2; :::; pn l  nhúng sè nguy¶n tè sao cho pn n+1. °t A = p1:p2:::pn. Chùng minh r¬ng trong d¢y sè c¡c sè tü nhi¶n li¶n ti¸p: A + 2;A + 3; :::;A + (n + 1), khæng chùa mët sè nguy¶n tè n o. B i 21. Chùng minh r¬ng: N¸u p l  sè nguy¶n tè th¼ 2:3:4:::(p3)(p 2) 1 .. .p. Chuy¶n · Sè håc Di¹n  n To¡n håc
  • 32. 24 2.4. Phö löc: B¤n n¶n bi¸t B i 22. Chùng minh r¬ng: N¸u p l  sè nguy¶n tè th¼ 2:3:4:::(p2)(p 1) + 1 .. .p. 2.4 Phö löc: B¤n n¶n bi¸t M÷íi sè nguy¶n tè câ 93 chú sè lªp th nh c§p sè cëng Sau ¥y l  mët sè nguy¶n tè gçm 93 chú sè: 100996972469714247637786655587969840329509324689190041 803603417758904341703348882159067229719 K löc n y do 70 nh  to¡n håc lªp ÷ñc n«m 1998 thªt khâ m  ¡nh b¤i ÷ñc. Hå m§t nhi·u th¡ng t½nh to¡n mîi t¼m ÷ñc m÷íi sè nguy¶n tè t¤o th nh mët c§p sè cëng. Tø möc trá chìi trong 1 t¤p ch½ khoa håc, hai nh  nghi¶n cùu ð tr÷íng ¤i håc Lyonl (Ph¡p) ¢  o s¥u þ t÷ðng: T¼m 6 sè nguy¶n tè sao cho hi»u 2 sè li¶n ti¸p luæn luæn nh÷ nhau. i·u â l  d¹ èi vîi c¡c chuy¶n gia nh÷ng hå muèn i xa hìn. Công khæng câ v§n · g¼ khâ kh«n èi vîi mët d¢y 7 sè. Hå c¦n sü hé trñ mët chót º ¤t ÷ñc 8 sè, mët sü hé trñ hìn núa º ¤t tîi 9 sè. Cuèi còng th¡ng 3 n«m 1998 câ 70 nh  to¡n håc tø kh­p tr¶n th¸ giîi còng vîi 200 m¡y i»n to¡n ho¤t ëng li¶n töc ¢ t¼m ra 10 sè, méi sè câ 93 chú sè, m  hi»u sè cõa 2 sè li¶n ti¸p luæn luæn l  210. Tø sè nguy¶n tè ð tr¶n ch¿ c¦n th¶m v o 210 l  ÷ñc sè nguy¶n tè thù 2.... K löc câ l³ døng ð â: Theo ÷îc t½nh cõa c¡c nh  khoa håc muèn t¼m ÷ñc 1 d¢y 11 sè nguy¶n tè th¼ ph£i m§t hìn 10 t¿ n«m. Sinh ba r§t ½t, ph£i ch«ng sinh æi l¤i r§t nhi·u Ta bi¸t r¬ng c¡c sè nguy¶n tè câ thº xa nhau tuý þ i·u n y thº hi»n ð b i tªp: Di¹n  n To¡n håc Chuy¶n · Sè håc
  • 33. 2.4. Phö löc: B¤n n¶n bi¸t 25 B i to¡n 2.1. Cho tr÷îc sè nguy¶n d÷ìng n tuý þ. Chùng minh r¬ng tçn t¤i n sè tü nhi¶n li¶n ti¸p m  méi sè trong chóng ·u l  hñp sè.4 Vªy nh÷ng, c¡c sè nguy¶n tè công câ thº r§t g¦n nhau. C°p sè (2; 3) l  c°p sè tü nhi¶n li¶n ti¸p duy nh§t m  c£ hai b¶n ·u l  sè nguy¶n tè. C°p sè (p; q)÷ñc gåi l  c°p sè sinh æi, n¸u c£ 2 ·u l  sè nguy¶n tè v  q = p + 2. Bë 3 sè (p; q; r) gåi l  bë sè nguy¶n tè sinh ba n¸u c£ 3 sè p,q,r ·u l  c¡c sè nguy¶n tè v  q = p + 2; r = q + 2. B i to¡n 2.2. T¼m t§t c£ c¡c bë sè nguy¶n tè sinh ba? 4 ¥y l  mët b i to¡n d¹, dòng ph÷ìng ph¡p chùng minh duy nh§t ta t¼m ra bë (3; 5; 7) l  bë ba sè nguy¶n tè sinh ba duy nh§t, c¡c bë 3 sè l´ lîn hìn 3 luæn câ 1 sè l  hñp sè v¼ nâ chia h¸t cho 3. Tø b i to¡n 2.2 th¼ b i to¡n sau trð th nh mët gi£ thuy¸t lîn ang chí c¥u tr£ líi. Dü o¡n 2.1 Tçn t¤i væ h¤n c°p sè sinh æi. Sè ho n h£o (ho n to n) cõa nhúng ng÷íi Hy L¤p cê ¤i Ng÷íi Hy L¤p cê ¤i câ quan ni»m th¦n b½ v· c¡c sè. Hå r§t thó và ph¡t hi»n ra c¡c sè ho n h£o, ngh¾a l  c¡c sè tü nhi¶n m  têng c¡c ÷îc sè tü nhi¶n thüc sü cõa nâ (c¡c ÷îc sè nhä hìn sè â) b¬ng ch½nh nâ. Ch¯ng h¤n: 6 = 1 + 2 + 3 28 = 1 + 2 + 4 + 7 + 14 Ng÷íi Hy L¤p cê ¤i ¢ bi¸t t¼m t§t c£ c¡c sè ho n h£o ch®n ngh¾a l  hå ¢ l m ÷ñc b i to¡n sau ¥y: B i to¡n 2.3. Mët sè tü nhi¶n ch®n n6= 0 l  sè ho n h£o n¸u v  ch¿ n¸u: n = 2m+1(2m 1). Trong â m l  sè tü nhi¶n kh¡c 0 sao cho 2m 1 l  sè nguy¶n tè. 4 Tø â ta câ gi£ thuy¸t Chuy¶n · Sè håc Di¹n  n To¡n håc
  • 34. 26 2.4. Phö löc: B¤n n¶n bi¸t Dü o¡n 2.2 Khæng tçn t¤i sè ho n h£o l´. Ð b i to¡n 2.3 tr¶n, sè nguy¶n tè d¤ng 2m 1 gåi l  sè nguy¶n tè Merseme. C¡c sè nguy¶n tè Merseme câ vai trá r§t quan trång. Cho ¸n nay ng÷íi ta v¨n ch÷a bi¸t câ húu h¤n hay væ h¤n sè nguy¶n tè Merseme. Dü o¡n 2.3 Tçn t¤i væ h¤n sè nguy¶n tè Merseme. N«m 1985 sè nguy¶n tè lîn nh§t m  ng÷íi ta bi¸t l  sè 21320491 gçm 39751 chú sè ghi trong h» thªp ph¥n. G¦n ¥y 2 sinh vi¶n Mÿ ¢ t¼m ra mët sè nguy¶n tè lîn hìn núa â l  sè 22160911 gçm 65050 chú sè. Ta bi¸t r¬ng vîi håc sinh lîp 6 º thû xem sè A câ ½t hìn 20 chú sè câ l  sè nguy¶n tè khæng b¬ng c¡ch thû xem A câ chia h¸t cho sè n o nhä hìn A hay khæng, th¼ º t¼m h¸t c¡c sè nguy¶n tè vîi chi¸c m¡y si¶u i»n to¡n c¦n h ng th¸ k !!! David SlowinSky ¢ so¤n mët ph¦n m·m, l m vi»c tr¶n m¡y si¶u i»n to¡n Gray-2 , sau 19 gií æng ¢ t¼m ra sè nguy¶n tè 2756839 1. Sè n y vi¸t trong h» thªp ph¥n s³ câ 227832 chú sè- vi¸t h¸t sè n y c¦n 110 trang v«n b£n b¼nh th÷íng. Ho°c n¸u vi¸t h ng ngang nhúng sè tr¶n phæng chú .VnTime Size 14 th¼ ta c¦n kho£ng 570 m. Líi K¸t Thæng qua · t i n y, chóng ta câ thº kh¯ng ành r¬ng: To¡n håc câ m°t trong måi cæng vi»c, måi l¾nh vüc cõa cuëc sèng quanh ta, nâ khæng thº t¡ch ríi v  l¢ng qu¶n ÷ñc, n¶n chóng ta ph£i hiºu bi¸t v  n­m b­t ÷ñc nâ mët c¡ch tü gi¡c v  hi»u qu£. Möc ½ch cõa · t i n y l  trang bà nhúng ki¸n thùc cì b£n câ  o s¥u câ n¥ng cao v  r±n luy»n t÷ duy to¡n håc cho håc sinh, t¤o ra n·n t£ng tin cªy º c¡c em câ vèn ki¸n thùc nh§t ành l m h nh trang cho Di¹n  n To¡n håc Chuy¶n · Sè håc
  • 35. 2.4. Phö löc: B¤n n¶n bi¸t 27 nhúng n«m håc ti¸p theo. Vîi i·u ki»n câ nhi·u h¤n ch¸ v· thíi gian, v· n«ng lüc tr¼nh ë n¶n trong khuæn khê · t i n y ph¥n chia d¤ng to¡n, lo¤i to¡n ch¿ câ t½nh t÷ìng èi. çng thíi công mîi ch¿ ÷a ra líi gi£i chù ch÷a câ ph÷ìng ph¡p, thuªt l m rã r ng. Tuy ¢ câ cè g­ng nhi·u nh÷ng chnsg tæi tü th§y trong · t i n y cán nhi·u h¤n ch¸. Chóng tæi r§t mong nhªn ÷ñc nhúng þ ki¸n âng gâp cõa c¡c th¦y cæ gi¡o còng b¤n åc º to¡n håc thªt sü câ þ ngh¾a cao µp nh÷ c¥u ng¤n ngú Ph¡p ¢ vi¸t: To¡n håc l  Vua cõa c¡c khoa håc Sè håc l  Nú ho ng Chuy¶n · Sè håc Di¹n  n To¡n håc
  • 36.
  • 37. Ch÷ìng 3 B i to¡n chia h¸t 3.1 Lþ thuy¸t cì b£n 29 3.2 Ph÷ìng ph¡p gi£i c¡c b i to¡n chia h¸t 31 Ph¤m Quang To n (Ph¤m Quang To n) Chia h¸t l  mët · t i quan trång trong ch÷ìng tr¼nh Sè håc cõa bªc THCS. i k±m theo â l  c¡c b i to¡n khâ v  hay. B i vi¸t n y xin giîi thi»u vîi b¤n åc nhúng ph÷ìng ph¡p gi£i c¡c b i to¡n chia h¸t: ph÷ìng ph¡p x²t sè d÷, ph÷ìng ph¡p quy n¤p, ph÷ìng ph¡p çng d÷, v.v... 3.1 Lþ thuy¸t cì b£n 3.1.1 ành ngh¾a v· chia h¸t ành ngh¾a 3.1 Cho hai sè nguy¶n a v  b trong â b6= 0, ta luæn t¼m ÷ñc hai sè nguy¶n q v  r duy nh§t sao cho a = bq + r vîi 0 r b. Trong â, ta nâi a l  sè bà chia, b l  sè chia, q l  th÷ìng, r l  sè d÷.4 Nh÷ vªy, khi a chia cho b th¼ câ thº ÷a ra c¡c sè d÷ r 2 f0; 1; 2; ; jbjg. °c bi»t, vîi r = 0 th¼ a = bq, khi â ta nâi a chia h¸t cho b (ho°c a l  bëi cõa b, ho°c b l  ÷îc cõa a). Ta k½ hi»u b j a. Cán khi a khæng chia 29
  • 38. 30 3.1. Lþ thuy¸t cì b£n h¸t cho b, ta k½ hi»u b - a. Sau ¥y l  mët sè t½nh ch§t th÷íng dòng, chùng minh ÷ñc suy ra trüc ti¸p tø ành ngh¾a. 3.1.2 T½nh ch§t Sau ¥y xin giîi thi»u mët sè t½nh ch§t v· chia h¸t, vi»c chùng minh kh¡ l  d¹ d ng n¶n s³ d nh cho b¤n åc. Ta câ vîi a; b; c; d l  c¡c sè nguy¶n th¼: T½nh ch§t 3.1 N¸u a6= 0 th¼ a j a, 0 j a. T½nh ch§t 3.2 N¸u b j a th¼ b j ac. T½nh ch§t 3.3 N¸u b j a v  c j b th¼ c j a. T½nh ch§t 3.4 N¸u c j a v  c j b th¼ c j (ax by) vîi x; y nguy¶n. T½nh ch§t 3.5 N¸u b j a v  a j b th¼ a = b ho°c a = b. T½nh ch§t 3.6 N¸u c j a v  d j b th¼ cd j ab. T½nh ch§t 3.7 N¸u b j a; c j a th¼ BCNN(b; c) j a. T½nh ch§t 3.8 N¸u c j ab v  UCLN(b; c) = 1 th¼ c j a. T½nh ch§t 3.9 N¸u p j ab, p l  sè nguy¶n tè th¼ p j a ho°c p j b. Tø t½nh ch§t tr¶n ta suy ra h» qu£ H» qu£ 3.1 N¸u p j an vîi p l  sè nguy¶n tè, n nguy¶n d÷ìng th¼ pn j an. Di¹n  n To¡n håc Chuy¶n · Sè håc
  • 39. 3.2. Ph÷ìng ph¡p gi£i c¡c b i to¡n chia h¸t 31 3.1.3 Mët sè d§u hi»u chia h¸t Ta °t N = anan1 : : : a1a0 D§u hi»u chia h¸t cho 2; 5; 4; 25; 8; 125 2 j N , 2 j a0 , a0 2 f0; 2; 4; 6; 8g 5 j N , 5 j a0 , a0 2 f0; 5g 4; 25 j N , 4; 25 j a1a0 8; 125 j N , 8; 125 j a2a1a0 D§u hi»u chia h¸t cho 3 v  9 3; 9 j N , 3; 9 j (a0 + a1 + + an1 + an) Mët sè d§u hi»u chia h¸t kh¡c 11 j N , 11 j [(a0 + a2 + ) (a1 + a3 + )] 101 j N , 101 j [(a1a0 + a5a4 + ) (a3a2 + a7a6 + )] 7; 13 j N , 7; 37 j [(a2a1a0 + a8a7a6 + ) (a5a4a3 + a11a10a9 + )] 37 j N , 37 j (a2a1a0 + a5a4a3 + + anan1an2) 19 j N , 19 j an + 2an1 + 22an2 + + 2na0 3.2 Ph÷ìng ph¡p gi£i c¡c b i to¡n chia h¸t 3.2.1 p döng ành lþ Fermat nhä v  c¡c t½nh ch§t cõa chia h¸t ành lþ Fermat nhä ành lþ 3.1 (ành lþ Fermat nhä) Vîi måi sè nguy¶n a v  sè nguy¶n tè p th¼ ap p (mod p). Chùng minh. 1. N¸u p j a th¼ p j (a5 a). 2. N¸u p - a th¼ 2a; 3a; 4a; ; (p 1)a công khæng chia h¸t cho p. Gåi r1; r2; ; rp1 l¦n l÷ñt l  sè d÷ khi chia a; 2a; 3a; ; (p1)a cho p. th¼ chóng s³ thuëc tªp f1; 2; 3; ; p1g v  æi mët kh¡c nhau (v¼ ch¯ng h¤n n¸u r1 = r3 th¼ p j (3a a) hay p j 2a, Chuy¶n · Sè håc Di¹n  n To¡n håc
  • 40. 32 3.2. Ph÷ìng ph¡p gi£i c¡c b i to¡n chia h¸t ch¿ câ thº l  p = 2, m  p = 2 th¼ b i to¡n khæng óng). Do â r1r2 rp1 = 1 2 3 (p 1). Ta câ a r1 (mod p) 2a r2 (mod p) (p 1)a rp1 (mod p) Nh¥n v¸ theo v¸ ta suy ra 123 (p1)ap1 r1r2 rp1 (mod p) ) ap1 1 (mod p) V¼ UCLN(a; p) = 1 n¶n ap a (mod p). Nh÷ vªy vîi måi sè nguy¶n a v  sè nguy¶n tè p th¼ ap a (mod p). Nhªn x²t. Ta câ thº chùng minh ành lþ b¬ng quy n¤p. Ngo i ra, ành lþ cán ÷ñc ph¡t biºu d÷îi d¤ng sau: ành lþ 3.2 Vîi måi sè nguy¶n a, p l  sè nguy¶n tè, UCLN(a; p) = 1 th¼ ap1 1 (mod p). Ph÷ìng ph¡p sû döng t½nh ch§t chia h¸t v  ¡p döng ành lþ Fermat nhä Cì sð: Sû döng c¡c t½nh ch§t chia h¸t v  ành lþ Fermat nhä º gi£i to¡n. V½ dö 3.1. Cho a v  b l  hai sè tü nhi¶n. Chùng minh r¬ng 5a2+15ab b2 chia h¸t cho 49 khi v  ch¿ khi 3a + b chia h¸t cho 7. 4 Líi gi£i. )) Gi£ sû 49 j 5a2 + 15ab b2 ) 7 j 5a2 + 15ab b2 ) 7 j (14a2 + 21ab) (5a2 + 15ab b2) ) 7 j (9a2 + 6ab + b2) ) 7 j (3a + b)2 ) 7 j 3a + b. () Gi£ sû 7 j 3a+b. °t 3a+b = 7c (c 2 Z. Khi â b = 7c3a. Nh÷ vªy ) 5a2 + 15ab b2 = 5a2 + 15a(7c 3a) (7c 3a)2 = 49(c2 + 3ac a2) Di¹n  n To¡n håc Chuy¶n · Sè håc
  • 41. 3.2. Ph÷ìng ph¡p gi£i c¡c b i to¡n chia h¸t 33 chia h¸t cho 49. Vªy 5a2 + 15ab b2 chia h¸t cho 49 khi v  ch¿ khi 3a + b chia h¸t cho 7. V½ dö 3.2. Cho 11 j (16a + 17b)(17a + 16b) vîi a; b l  hai sè nguy¶n. Chùng minh r¬ng 121 j (16a + 17b)(17a + 16b). 4 Líi gi£i. Ta câ theo ¦u b i, v¼ 11 nguy¶n tè n¶n ½t nh§t mët trong hai sè 16a + 17b v  17a + 16b chia h¸t cho 11. Ta l¤i câ (16a + 17b) + (17a + 16b) = 33(a + b) chia h¸t cho 11. Do â n¸u mët trong hai sè 16a + 17b v  17a + 16b chia h¸t cho 11 th¼ sè cán l¤i công chia h¸t cho 11. Cho n¶n 121 j (16a + 17b)(17a + 16b). V½ dö 3.3. Chùng minh r¬ng A = 130 + 230 + + 1130 khæng chia h¸t cho 11. 4 Líi gi£i. Vîi måi a = 1; 2; ; 10 th¼ (a; 10) = 1. Do â theo ành lþ Fermat b² th¼ a10 1 (mod 11) ) a30 1 (mod 11) vîi måi a = 1; 2; ; 10 v  1130 0 (mod 11). Nh÷ vªy A |1 + 1 +{z + 1} 10 sè 1 +0 (mod 11) 10 (mod 11) ) 11 - A V½ dö 3.4. Cho p v  q l  hai sè nguy¶n tè ph¥n bi»t. Chùng minh r¬ng pq1 + qp1 1 chia h¸t cho pq. 4 Líi gi£i. V¼ q nguy¶n tè n¶n theo ành lþ Fermat nhä th¼ pq1 1 (mod q) Do â pq1 + qp1 1 (mod q) V¼ q v  p câ vai trá b¼nh ¯ng n¶n ta công d¹ d ng suy ra qp1 + pq1 1 (mod p): Cuèi còng v¼ UCLN(q; p) = 1 n¶n pq1 + qp1 1 (mod pq) hay pq1 + qp1 1 chia h¸t cho pq. Chuy¶n · Sè håc Di¹n  n To¡n håc
  • 42. 34 3.2. Ph÷ìng ph¡p gi£i c¡c b i to¡n chia h¸t B i tªp · nghà B i 1. Chùng minh r¬ng 11a+2b chia h¸t cho 19 khi v  ch¿ khi 18a+5b chia h¸t cho 19 vîi a; b l  c¡c sè nguy¶n. B i 2. Chùng minh r¬ng 2a + 7 chia h¸t cho 7 khi v  ch¿ khi 3a2 + 10ab 8b2. B i 3. Cho p l  sè nguy¶n tè lîn hìn 5. Chùng minh r¬ng n¸u n l  sè tü nhi¶n câ p1 chú sè v  c¡c chú sè â ·u b¬ng 1 th¼ n chia h¸t cho p. B i 4. Gi£ sû n 2 N; n 2. X²t c¡c sè tü nhi¶n an = 11 1 ÷ñc vi¸t bði n chú sè 1. Chùng minh r¬ng n¸u an l  mët sè nguy¶n tè th¼ n l  ÷îc cõa an 1. B i 5. Gi£ sû a v  b l  c¡c sè nguy¶n d÷ìng sao cho 2a 1; 2b 1 v  a + b ·u l  sè nguy¶n tè. Chùng minh r¬ng ab + ba v  aa + bb ·u khæng chia h¸t cho a + b. B i 6. Chùng minh r¬ng vîi måi sè nguy¶n tè p th¼ tçn t¤i sè nguy¶n n sao cho 2n + 3n + 6n 1 chia h¸t cho p. 3.2.2 X²t sè d÷ Cì sð: º chùng minh A(n) chia h¸t cho p, ta x²t c¡c sè n d¤ng n = kp + r vîi r 2 f0; 1; 2; ; p 1g. Ch¯ng h¤n, vîi p = 5 th¼ sè nguy¶n n câ thº vi¸t l¤i th nh 5k; 5k + 1; 5k + 2; 5k + 3; 5k + 4. Ta th¸ méi d¤ng n y v o c¡c và tr½ cõa n rçi lþ luªn ra ¡p sè. Sau ¥y l  mët sè v½ dö V½ dö 3.5. T¼m k 2 N º tçn t¤i n 2 N sao cho 4 j n2 k vîi k 2 f0; 1; 2; 3g. 4 Líi gi£i. Gi£ sû tçn t¤i k 2 N º tçn t¤i n 2 N thäa m¢n 4 j n2 k. Ta x²t c¡c Tr÷íng hñp: (m 2 N) Di¹n  n To¡n håc Chuy¶n · Sè håc
  • 43. 3.2. Ph÷ìng ph¡p gi£i c¡c b i to¡n chia h¸t 35 1. N¸u n = 4m th¼ n2 k = 16m2 k chia h¸t cho 4 khi v  ch¿ khi 4 j k n¶n k = 0: 2. N¸u n = 4m 1 th¼ n2 k = 16m2 8m + 1 k chia h¸t cho 4 khi v  ch¿ khi 4 j 1 k n¶n k = 1. 3. N¸u n = 4m 2 th¼ n2 k = 16m2 16m+ 4 k chia h¸t cho 4 khi v  ch¿ khi 4 j k n¶n k = 0. Vªy k = 0 ho°c k = 1. V½ dö 3.6. Chùng minh r¬ng vîi måi n 2 N th¼ 6 j n(2n+7)(7n+1).4 Líi gi£i. Ta th§y mët trong hai sè n v  7n + 1 l  sè ch®n 8n 2 N. Do â 2 j n(2n+7)(7n+1). Ta s³ chùng minh 3 j n(2n+7)(7n+1). Thªt vªy, x²t 1. Vîi n = 3k th¼ 3 j n(2n + 7)(7n + 1). 2. Vîi n = 3k + 1 th¼ 2n + 7 = 6k + 9 chia h¸t cho 3 n¶n 3 j n(2n + 7)(7n + 1). 3. Vîi n = 3k + 2 th¼ 7n + 1 = 21k + 15 chia h¸t cho 3 n¶n 3 j n(2n + 7)(7n + 1). Do â 3 j n(2n+7)(7n+1) m  (2; 3) = 1 n¶n 6 j n(2n+7)(7n+1) 8n 2 N. V½ dö 3.7. (HSG 9, Tp Hç Ch½ Minh, váng 2, 1995) Cho x; y; z l  c¡c sè nguy¶n thäa m¢n (x y)(y z)(z x) = x + y + z (3.1) Chùng minh r¬ng 27 j (x + y + z). 4 Líi gi£i. X²t hai tr÷íng hñp sau Chuy¶n · Sè håc Di¹n  n To¡n håc
  • 44. 36 3.2. Ph÷ìng ph¡p gi£i c¡c b i to¡n chia h¸t 1. N¸u ba sè x; y; z chia h¸t cho 3 câ c¡c sè d÷ kh¡c nhau th¼ c¡c hi»u xy; yz; zx còng khæng chia h¸t cho 3. M  3 j (x+y+z) n¶n tø (3.1) suy ra væ l½ . 2. N¸u ba sè x; y; z ch¿ câ hai sè chia cho 3 câ còng sè d÷ th¼ trong ba hi»u xy; yz; zx câ mët hi»u chia h¸t cho 3. M  3 - (x+y+z) n¶n tø (3.1) suy ra væ l½. Vªy x; y; z chia cho 3 câ còng sè d÷, khi â x y; y z; z x ·u chia h¸t cho 3. Tø (3.1) ta suy ra 27 j (x + y + z), ta câ pcm. B i tªp · nghà B i 1. i) T¼m sè tü nhi¶n n º 7 j (2n 1). ii) Chùng minh r¬ng 7 - (2n + 1) 8n 2 N. B i 2. Chùng minh r¬ng vîi måi sè nguy¶n a th¼ a(a6 1) chia h¸t cho 7. B i 3. T¼m n º 13 j 32n + 3n + 1. B i 4. Chùng minh r¬ng vîi måi a; b 2 N th¼ ab(a2b2)(4a2b2) luæn luæn chia h¸t cho 5. B i 5. Chùng minh r¬ng 24 j (p 1)(p + 1) vîi p l  sè nguy¶n tè lîn hìn 3. B i 6. Chùng minh r¬ng khæng tçn t¤i sè nguy¶n a º a2 +1 chia h¸t cho 12. B i 7. Chùng minh r¬ng vîi måi sè nguy¶n x; y; z n¸u 6 j x+y+z th¼ 6 j x3 + y3 + z3. B i 8. Cho ab = 20112012, vîi a; b 2 N. Häi têng a + b câ chia h¸t cho 2012 hay khæng ? B i 9. Sè 3n+2003 trong â n l  sè nguy¶n d÷ìng câ chia h¸t cho 184 khæng ? Di¹n  n To¡n håc Chuy¶n · Sè håc
  • 45. 3.2. Ph÷ìng ph¡p gi£i c¡c b i to¡n chia h¸t 37 B i 10. Cho c¡c sè nguy¶n d÷ìng x; y; z thäa m¢n x2 +y2 = z2. Chùng minh r¬ng xyz chia h¸t cho 60. B i 11. Cho c¡c sè nguy¶n d÷ìng x; y; z thäa m¢n x2+y2 = 2z2. Chùng minh r¬ng x2 y2 chia h¸t cho 84: B i 12. Cho n 3; (n 2 N). Chùng minh r¬ng n¸u 2n = 10a+b; (0 b 9) th¼ 6 j ab. 3.2.3 Ph¥n t½ch Ph¥n t½ch th nh t½ch Cì sð: º chùng minh A(n) chia h¸t cho p, ta ph¥n t½ch A(n) = D(n)p, cán n¸u trong ta khæng thº ÷a ra c¡ch ph¥n t½ch nh÷ vªy, ta câ thº vi¸t p = kq. N¸u (k; q) = 1 th¼ ta chùng minh A(n) còng chia h¸t cho k v  q. N¸u (k; q)6= 1 th¼ ta vi¸t A(n) = B(n)C(n) v  chùng minh B(n) chia h¸t cho k, C(n) chia h¸t cho q. V½ dö 3.8. Cho n l  mët sè nguy¶n d÷ìng. Chùng minh r¬ng 2n j (n + 1) (n + 2) (2n) : Líi gi£i. Ta câ (n + 1) (n + 2) (2n) = (2n)! n! = (1:3:5:::(2n 1)) (2:4:6:::2n) n! = 1:3:5:::(2n 1):2n: n! n! = 1:3:5:::(2n 1):2n: Do â 2n j (n + 1) (n + 2) (2n) : Chuy¶n · Sè håc Di¹n  n To¡n håc
  • 46. 38 3.2. Ph÷ìng ph¡p gi£i c¡c b i to¡n chia h¸t V½ dö 3.9. Chùng minh r¬ng vîi måi sè nguy¶n n th¼ 6 j n3 n. 4 Líi gi£i. Ph¥n t½ch n3 n = n(n2 1) = n(n 1)(n + 1) Biºu thùc l  t½ch ba sè nguy¶n li¶n ti¸p n¶n tçn t¤i ½t nh§t mët trong ba sè mët sè chia h¸t cho 2 v  mët sè chia h¸t cho 3. M  (2; 3) = 1 n¶n 6 j n3 n. V½ dö 3.10. Chùng minh r¬ng n6 n4 n2 + 1 chia h¸t cho 128 vîi n l´. 4 Líi gi£i. Ta câ n6 n4 n2 + 1 = (n2 1)2(n + 1) = (n 1)2(n + 1)2 V¼ n l´ n¶n °t n = 2k; k 2 N, suy ra (n2 1)2 = (2k + 1)2 1 = (4k2 + 4k)2 = [4k(k + 1)]2 Vªy 64 j (n2 1)2. V¼ n l´ n¶n 2 j n + 1, suy ra pcm. V½ dö 3.11. Cho ba sè nguy¶n d÷ìng kh¡c nhau x; y; z. Chùng minh r¬ng (xy)5+(yz)5+(xz)5 chia h¸t cho 5(xy)(yz)(xz).4 Líi gi£i. Ta câ (x y)5 + (y z)5 + (x z)5 = (x z + z y)5 + (y z)5 + (z x)5 = (x z)5 + 5(x z)4(z y) + 10(x z)3(z y)2 +10(x z)4(z y) + 10(x z)3(z y)2 +10(x z)2(z y)3 + 5(x z)(z y)4 = 5(x z)(z y) (x z)3 + 2(x z)2(z y) + 2(x z)(z y)2 + (z y)3 : Di¹n  n To¡n håc Chuy¶n · Sè håc
  • 47. 3.2. Ph÷ìng ph¡p gi£i c¡c b i to¡n chia h¸t 39 Nh÷ng ta công câ: (x z)3 + 2(x z)2(z y) + 2(x z)(z y)2 + (z y)3 = (x y + y z)3 + 2(x y + y z)2(z y) +2(x y + y z)(z y)2 + (z y)3 = (x y)3 + 2(x y)2(y z) + 3(x y)(y z)2 +(y z)3 + 2(x y)2(z y) +4(x y)(y z)(z y) + 2(y z)2(z y) +2(x y)(z y)2 + 2(y z)(z y)2 + (z y)3 = (x y)3 + 3(x y)2(y z) + 3(x y)(y z)2 +2(x y)2(z y) + 4(x y)(y z)(z y) + 2(x y)(z y)2; Biºu thùc cuèi còng câ nh¥n tû chung (x y): Ta suy ra i·u ph£i chùng minh. B i tªp · nghà B i 1. Chùng minh r¬ng n¸u a; k l  c¡c sè nguy¶n, a l´ th¼ 2k+1 j (a2k 1). B i 2. Chùng minh r¬ng n5 n chia h¸t cho 30 vîi måi n 2 Z. B i 3. Chùng minh r¬ng 3n4 14n3 + 21n2 10n chia h¸t cho 24 vîi måi n 2 Z. B i 4. Chùng minh r¬ng n55n3+4n chia h¸t cho 120 vîi måi n 2 Z. B i 5. Chùng minh r¬ng n3 3n2 n + 3 chia h¸t cho 48 vîi måi n l´, n 2 Z. B i 6. Chùng minh r¬ng n8 n6 n4 + n2 chia h¸t cho 1152 vîi måi sè nguy¶n n l´. B i 7. Chùng minh r¬ng n44n34n2+16n chia h¸t cho 348 vîi måi n l  sè nguy¶n ch®n. B i 8. Chùng minh r¬ng n4 14n3 + 71n2 154n + 120 chia h¸t cho 24 vîi måi sè tü nhi¶n n. Chuy¶n · Sè håc Di¹n  n To¡n håc
  • 48. 40 3.2. Ph÷ìng ph¡p gi£i c¡c b i to¡n chia h¸t B i 9. Cho x; y; z l  c¡c sè nguy¶n kh¡c 0. Chùng minh r¬ng n¸u x2 yz = a; y2 zx = b; z2 xy = c th¼ têng (ax + by + cz) chia h¸t cho têng (a + b + c). B i 10. Cho m; n l  hai sè ch½nh ph÷ìng l´ li¶n ti¸p. Chùng minh r¬ng mn m n + 1 chia h¸t cho 192. B i 11. (HSG 9 TQ 1970) Chùng minh r¬ng n12 n8 n4 +1 chia h¸t cho 512 vîi måi sè tü nhi¶n n l´. B i 12. (HSG 9 TQ 1975) Chùng minh r¬ng n4 +6n3 +11n2 +6n chia h¸t cho 24 vîi måi sè nguy¶n d÷ìng n. T¡ch têng Cì sð: º chùng minh A(n) chia h¸t cho p, ta bi¸n êi A(n) th nh têng nhi·u h¤ng tû rçi chùng minh méi h¤ng tû ·u chia h¸t cho p. Ta câ thº sû döng mët sè h¬ng ¯ng thùc ¡p döng v o chia h¸t, v½ dö nh÷: Cho a; b l  c¡c sè thüc v  n l  sè nguy¶n d÷ìng. Khi â ta câ an bn = (a b)(an1 + an2b + + abn2 + bn1) Ta s³ câ h» qu£ l : H» qu£ 3.2 N¸u a b6= 0 th¼ an bn chia h¸t cho a b. H» qu£ 3.3 N¸u a + b6= 0 v  n l´ th¼ an + bn chia h¸t cho a + b. H» qu£ 3.4 N¸u a+b6= 0 v  n ch®n th¼ an bn chia h¸t cho a+b V½ dö 3.12. Chùng minh r¬ng ax2 +bx+c 2 Z; 8x 2 Z khi v  ch¿ khi 2a; a + b; c 2 Z 4 Di¹n  n To¡n håc Chuy¶n · Sè håc
  • 49. 3.2. Ph÷ìng ph¡p gi£i c¡c b i to¡n chia h¸t 41 Líi gi£i. Ph¥n t½ch ax2 + bx + c = ax2 ax + (a + b)x + c = 2a: x(x 1) 2 + (a + b)x + c 2 Z; 8x 2 Z: V½ dö 3.13. Chùng minh r¬ng 6 j (a3 + 5a) 8a 2 N. 4 Líi gi£i. Ph¥n t½ch a3+5a = (a3a)+6a. Hiºn nhi¶n óng v¼ 6 j n3n (chùng minh ð v½ dö Equation 4.27). Nhªn x²t. Tø v½ dö Equation 4.27 ta công câ thº ÷a ra c¡c b i to¡n sau, chùng minh công b¬ng c¡ch vªn döng ph÷ìng ph¡p t¡ch têng: B i to¡n 3.1. Cho m; n 2 Z. Chùng minh r¬ng 6 j m2n2(m n). 4 B i to¡n 3.2. Cho a; b; c 2 Z. Chùng minh r¬ng 6 j (a3 + b3 + c3) khi v  ch¿ khi 6 j (a + b + c) 4 B i to¡n 3.3. Cho a 2 Z. Chùng minh r¬ng a 3 + a2 2 + a3 6 2 Z 4 B i to¡n 3.4. Vi¸t sè 20112012 th nh têng c¡c sè nguy¶n d÷ìng. em têng lªp ph÷ìng t§t c£ c¡c sè h¤ng â chia cho 3 th¼ ÷ñc d÷ l  bao nhi¶u ? 4 V½ dö 3.14. Cho m; n l  c¡c sè nguy¶n thäa m¢n: m n = 1 1 2 + 1 3 1 4 + 1 1334 + 1 1335 Chùng minh r¬ng 2003 j m. 4 Chuy¶n · Sè håc Di¹n  n To¡n håc
  • 50. 42 3.2. Ph÷ìng ph¡p gi£i c¡c b i to¡n chia h¸t Líi gi£i. º þ r¬ng 2003 l  sè nguy¶n tè. Ta câ m 1 1 1 1 1 = 1 + + + n 2 3 4 1334 1335 = 1 + 1 2 + 1 3 + + 1 1335 2 1 2 + 1 4 + 1 6 + + 1 1334 = 1 + 1 2 + 1 3 + + 1 1335 1 + 1 2 + 1 3 + + 1 667 = 1 668 + 1 669 + + 1 1335 = 1 668 + 1 1335 + 1 669 + 1 1334 + + 1 1001 + 1 1002 = 2003 1 668:1335 + 1 669:1334 + + 1 1001:1002 = 2003: p q Ð ¥y p l  sè nguy¶n cán q = 668 669 1335. V¼ 2003 nguy¶n tè n¶n (q; 2003) = 1. Do â tø () suy ra 2003pn = mq: V¼ p; n nguy¶n n¶n suy ra 2003jmq m  (q; 2003) = 1 n¶n 2003jm. V½ dö 3.15. Chùng minh r¬ng vîi måi sè tü nhi¶n n th¼ A = 2005n + 60n 1897n 168n chia h¸t cho 2004. 4 Líi gi£i. Ta câ 2004 = 12 167. V¼ (12; 167) = 1 n¶n º chùng minh A chia h¸t cho 2004 ta chùng minh A chia h¸t cho 12 v  167. p döng t½nh ch§t an bn chia h¸t cho a b vîi måi n tü nhi¶n v  ab6= 0 suy ra 2005n1897n chia h¸t cho 20051897 = 108 = 129, hay 2005n 1897n chia h¸t cho 12. T÷ìng tü th¼ 168n 60n chia h¸t cho 12. Vªy A chia h¸t cho 12. Ti¸p töc ph¥n t½ch A = (2005n 168n) (1897n 60n): Lªp luªn t÷ìng tü nh÷ tr¶n th¼ 2005n 168n v  1897n 60n chia h¸t cho 167, tùc A chia h¸t cho 167. Vªy ta câ i·u ph£i chùng minh. Di¹n  n To¡n håc Chuy¶n · Sè håc
  • 51. 3.2. Ph÷ìng ph¡p gi£i c¡c b i to¡n chia h¸t 43 V½ dö 3.16. (· thi tuyºn sinh HKHTN-HQG H  Nëi, váng 1, n«m 2007-2008) Cho a; b l  hai sè nguy¶n d÷ìng v  a+1; b+2007 ·u chia h¸t cho 6. Chùng minh r¬ng 4a + a + b chia h¸t cho 6. 4 Líi gi£i. Ph¥n t½ch 4a + a + b = (4a + 2) + (a + 1) + (b + 2007) 2010 4a + 2 = 4a 1 + 3 = (4 1)(4a1 + 1) + 3 Nh÷ vªy 3 j 4a + 2. Do â 4a + a + b l  têng cõa c¡c sè nguy¶n d÷ìng chia h¸t cho 6 n¶n 4a + a + b chia h¸t cho 6. B i tªp · nghà B i 1. ÷a ra c¡c mð rëng tø b i tªp · nghà cõa ph÷ìng ph¡p ph¥n t½ch th nh t½ch th nh c¡c b i to¡n vªn döng ph÷ìng ph¡p t¡ch têng (gièng nh÷ c¡ch mð rëng cõa v½ dö 1.9). B i 2. (Hungary MO 1947) Chùng minh r¬ng 46n + 296:13n chia h¸t cho 1947 vîi måi sè tü nhi¶n n l´. B i 3. Chùng minh r¬ng 20n + 16n 3n 1 chia h¸t cho 323 vîi måi sè tü nhi¶n n ch®n. B i 4. Chùng minh r¬ng 2903n803n464n+261n chia h¸t cho 1897 vîi måi sè tü nhi¶n n. B i 5. Chùng minh r¬ng vîi måi sè nguy¶n n 1 ta câ nn + 5n2 11n + 5 chia h¸t cho (n 1)2. B i 6. (HSG 9 Tp H  Nëi, váng 2, 1998) Chùng minh r¬ng 1997 j m vîi m; n 2 N thäa m¢n m n = 1 1 2 + 1 3 1 4 + + 1 1329 1 1330 + 1 1331 : B i 7. Chùng minh r¬ng 32n+1 + 2n+2 chia h¸t cho 7 vîi måi n 2 N: Chuy¶n · Sè håc Di¹n  n To¡n håc
  • 52. 44 3.2. Ph÷ìng ph¡p gi£i c¡c b i to¡n chia h¸t B i 8. Chùng minh r¬ng 20032005 + 20172015 chia h¸t cho 12. B i 9. Cho p l  sè tü nhi¶n l´ v  c¡c sè nguy¶n a; b; c; d; e thäa m¢n a + b + c + d + e v  a2 + b2 + c2 + d2 + e2 ·u chia h¸t cho p. Chùng minh r¬ng sè a5 + b5 + c5 + d5 + e5 5abcde công chia h¸t cho p. B i 10. (Canada Training for IMO 1987) K½ hi»u: 1 3 5 (2n 1) = (2n 1)!! 2 4 6 (2n) = (2n)!!: Chùng minh r¬ng (1985)!! + (1986)!! chia h¸t cho 1987: B i 11. Chùng minh r¬ng sè 22225555 + 55552222 chia h¸t cho 7. B i 12. Cho k l  sè nguy¶n d÷ìng sao cho sè p = 3k + 1 l  sè nguy¶n tè v  1 1 2 + 1 3 4 + + 1 (2k 1)2k = m n vîi hai sè nguy¶n d÷ìng nguy¶n tè còng nhau m v  n.Chùng minh m chia h¸t cho p. (T¤p ch½ Mathematics Reflections, «ng bði T.Andreescu) 3.2.4 X²t çng d÷ ành ngh¾a v  mët sè t½nh ch§t ành ngh¾a 3.2 Cho a; b l  c¡c sè nguy¶n v  n l  sè nguy¶n d÷ìng. Ta nâi, a çng d÷ vîi b theo modun n v  k½ hi»u a b (mod n) n¸u a v  b câ còng sè d÷ khi chia cho n. 4 Nh÷ vªy a n (mod n) () n j (a b). V½ dö: 2012 2 (mod 5). T½nh ch§t (b¤n åc tü chùng minh) Cho a; b; c; d; n l  c¡c sè nguy¶n. T½nh ch§t 3.10 a a (mod n); a b (mod n) , b a (mod n); a b (mod n); b c (mod n) ) a c (mod n): Di¹n  n To¡n håc Chuy¶n · Sè håc
  • 53. 3.2. Ph÷ìng ph¡p gi£i c¡c b i to¡n chia h¸t 45 T½nh ch§t 3.11 ( a b (mod n) c d (mod n) ) ( a c b d (mod n) ac bd (mod n) T½nh ch§t 3.12 a b (mod n) ) ak bk (mod n); 8k 1: T½nh ch§t 3.13 a b (mod n) ) ac bc (mod mc); c 0 T½nh ch§t 3.14 (a + b)n bn (mod a); (a 0): T½nh ch§t 3.15 N¸u d l  ÷îc chung d÷ìng cõa a; b v  m th¼ a b (mod m) th¼ a d b d (mod m d ). T½nh ch§t 3.16 a b (mod m), c l  ÷îc chung cõa a v  b, (c;m) = 1 th¼ a c b c (mod m). Ph÷ìng ph¡p çng d÷ thùc º gi£i c¡c b i to¡n chia h¸t Cì sð: Sû döng c¡c t½nh ch§t v  ành ngh¾a tr¶n º gi£i c¡c b i to¡n chia h¸t. V½ dö 3.17. Chùng minh r¬ng vîi måi sè tü nhi¶n n th¼ 7 j 8n + 6. 4 Líi gi£i. Ta câ 8n 1 (mod 7) =) 8n + 6 7 0 (mod 7): V½ dö 3.18. Chùng minh r¬ng 19 j 7 52n + 12 6n: vîi måi sè nguy¶n d÷ìng n. 4 Líi gi£i. Ta câ 52 = 25 6 (mod 19) =) (52)n 6n (mod 19) =) 7 52n 7 6n (mod 19) =) 7 52n +12 6n 19 6n 0 (mod 19): V½ dö 3.19. Vi¸t li¶n ti¸p c¡c sè 111; 112; ; 888 º ÷ñc sè A = 111112 888. Chùng minh r¬ng 1998 j A. 4 Chuy¶n · Sè håc Di¹n  n To¡n håc
  • 54. 46 3.2. Ph÷ìng ph¡p gi£i c¡c b i to¡n chia h¸t Líi gi£i. Ta th§y A ch®n n¶n 2 j A. M°t kh¡c A = 111 1000777 + 112 1000776 + + 888: Do 1000k 1 (mod 999); 8k 2 N n¶n A 111 + 112 + + 888 0 (mod 999): Suy ra 999 j A, v  (999; 2) = 1 n¶n 1998 j A: V½ dö 3.20. Chùng minh r¬ng 7 j 55552222 + 22225555. 4 Líi gi£i. Ta câ 2222 4 (mod 7) =) 22225555 (4)5555 (mod 7) 5555 4 (mod 7) =) 55552222 4 (mod 7) =) 55552222 + 22225555 45555 + 42222 (mod 7) L¤i câ 45555 + 42222 = 42222 43333 1 = 42222 641111 1 V  64 1 (mod 7) =) 641111 1 0 (mod 7). Do â 7 j 55552222 + 22225555 B i tªp · nghà B i 1. Mët sè b i tªp ð ph÷ìng ph¡p ph¥n t½ch câ thº gi£i b¬ng ph÷ìng ph¡p çng d÷ thùc. B i 2. Chùng minh r¬ng 333555777 + 777555333 chia h¸t cho 10. B i 3. Chùng minh r¬ng sè 11101967 1 chia h¸t cho 101968. B i 4. Cho 9 j a3 +b3 +c3; 8a; b; c 2 Z. Chùng minh r¬ng 3 j a b c. B i 5. Chùng minh r¬ng 222333 + 333222 chia h¸t cho 13. Di¹n  n To¡n håc Chuy¶n · Sè håc
  • 55. 3.2. Ph÷ìng ph¡p gi£i c¡c b i to¡n chia h¸t 47 B i 6. Chùng minh r¬ng 9n + 1 khæng chia h¸t cho 100; 8n 2 N: B i 7. Chùng minh r¬ng vîi måi sè nguy¶n khæng ¥m n th¼ 25n+3 + 5n 3n+1 chia h¸t cho 17. B i 8. T¼m n 2 N sao cho 2n3 + 3n = 19851986. B i 9. Vi¸t li¶n ti¸p 2000 sè 1999 ta ÷ñc sè X = 19991999 1999: T¼m sè d÷ trong ph²p chia X cho 10001. B i 10. Chùng minh r¬ng 100 j 77777 777 . B i 11. Cho b2 4ac v  b2 + 4ac l  hai sè ch½nh ph÷ìng vîi a; b; c 2 N. Chùng minh r¬ng 30 j abc. 3.2.5 Quy n¤p Cì sð : º chùng minh m»nh · óng vîi måi sè tü nhi¶n n p, ta l m nh÷ sau: Kiºm tra m»nh · óng vîi n = p. Gi£ sû m»nh · óng vîi n = k. Ta i chùng minh m»nh · công óng vîi n = k + 1. V½ dö 3.21. Chùng minh r¬ng A = 4n + 15 1 chia h¸t cho 9 vîi måi n 2 N. 4 Líi gi£i. Vîi n = 1 =) A = 18 chia h¸t cho 9. Gi£ sû b i to¡n óng vîi n = k. Khi â 9 j 4k+15k1, hay 4k+15k1 = 9q vîi q 2 N. Suy ra 4k = 9q 15k + 1. Ta i chùng minh b i to¡n óng vîi n = k+1, tùc 9 j 4k+1+15(k+1)1. Thªt vªy: 4k+1 + 15(k + 1) 1 = 4 4k + 15k + 14 = 4 (9q 15k + 1) + 15k + 14 = 36q 45k + 18 chia h¸t cho 9. Ta câ pcm. Chuy¶n · Sè håc Di¹n  n To¡n håc
  • 56. 48 3.2. Ph÷ìng ph¡p gi£i c¡c b i to¡n chia h¸t V½ dö 3.22. (HSG 9 TQ 1978)Chùng minh r¬ng sè ÷ñc t¤o bði 3n chú sè gièng nhau th¼ chia h¸t cho 3n vîi 1 n; n 2 N: 4 Líi gi£i. Vîi n = 1, b i to¡n hiºn nhi¶n óng. Gi£ sû b i to¡n óng vîi n = k, tùc 3k j |aa{z a} 3n sè a : Vîi n = k + 1 ta câ: |aa{z a} 3k+1 = |aa{z a} 3k |aa{z a} 3k |aa{z a} 3k = |aa{z a} 3k 1 |00{z 0} 3k1 |00{z 0} 3k1 1 chia h¸t cho 3k+1. Ta câ pcm. V½ dö 3.23. Chùng minh r¬ng vîi måi n 2 N; k l  sè tü nhi¶n l´ th¼ 2n+2 j k2n 1 Líi gi£i. Vîi n = 1 th¼ k2n 1 = k2 1 = (k + 1)(k 1). Do k l´,n¶n °t k = 2m+ 1 vîi m 2 N, th¼ khi â (k + 1)(k 1) = 4k(k + 1) chia h¸t cho 23 = 8. Gi£ sû b i to¡n óng vîi n = p, tùc 2p+2 j k2p 1 hay k2p = q 2p+2+1 vîi q 2 N. Ta chùng minh b i to¡n óng vîi n = p + 1. Thªt vªy A = k2p+1 1 = k22p 1 = k2p2 1 = k2p 1 k2p + 1 = q 2p+2 2 + q 2p+2 = q 2p+3 1 + q 2p+1 chia h¸t cho 2p+3. Ta câ pcm. Di¹n  n To¡n håc Chuy¶n · Sè håc
  • 57. 3.2. Ph÷ìng ph¡p gi£i c¡c b i to¡n chia h¸t 49 B i tªp · nghà B i 1. Mët sè b i to¡n ð c¡c ph÷ìng ph¡p n¶u tr¶n câ thº gi£i b¬ng ph÷ìng ph¡p quy n¤p. B i 2. Chùng minh r¬ng 255 j 16n 15n 1 vîi n 2 N. B i 3. Chùng minh r¬ng 64 j 32n+3 + 40n 27 vîi n 2 N. B i 4. Chùng minh r¬ng 16 j 32n+2 + 8n 9 vîi n 2 N. B i 5. Chùng minh r¬ng 676 j 33n+3 16n 27 vîi n 2 N; n 1. B i 6. Chùng minh r¬ng 700 j 292n 140n 1 vîi n 2 N. B i 7. Chùng minh r¬ng 270 j 2002n 138n 1 vîi n 2 N. B i 8. Chùng minh r¬ng 22 j 324n+1 + 234n+1 + 5 vîi n 2 N. B i 9. Chùng minh r¬ng sè 23n + 1 chia h¸t cho 3n nh÷ng khæng chia h¸t cho 3n+1 vîi n 2 N. B i 10. Chùng minh r¬ng sè 20012n 1 chia h¸t cho 2n+4 nh÷ng khæng chia h¸t cho 2n+5 vîi n 2 N. B i 11. Chùng minh r¬ng vîi måi sè tü nhi¶n n 2, tçn t¤i mët sè tü nhi¶n m sao cho 3n j (m3 + 17), nh÷ng 3n+1 - (m3 + 17). B i 12. Câ tçn t¤i hay khæng mët sè nguy¶n d÷ìng l  bëi cõa 2007 v  câ bèn chú sè tªn còng l  2008. B i 13. Chùng minh r¬ng tçn t¤i mët sè câ 2011 chú sè gçm to n chú sè 1 v  2 sao cho sè â chia h¸t cho 22011. B i 14. T¼m ph¦n d÷ khi chia 32n cho 2n+3, trong â n l  sè nguy¶n d÷ìng. B i 15. Cho n 2 N; n 2. °t A = 77::: (lôy thøa n l¦n). Chùng minh r¬ng An + 17 chia h¸t cho 20. Chuy¶n · Sè håc Di¹n  n To¡n håc
  • 58. 50 3.2. Ph÷ìng ph¡p gi£i c¡c b i to¡n chia h¸t 3.2.6 Sû döng nguy¶n l½ Dirichlet Nëi dung: Nhèt 5 con thä v o 3 chuçng th¼ tçn t¤i chuçng chùa ½t nh§t 2 con. ành lþ 3.3 Nhèt m = nk + 1 con thä v o k chuçng (k n) th¼ tçn t¤i chuçng chùa ½t nh§t n + 1 con thä. Chùng minh. Gi£ sû khæng câ chuçng n o chùa ½t nh§t n+1 con thä, khi â méi chuçng chùa nhi·u nh§t n con thä, n¶n k chuçng chùa nhi·u nh§t kn con thä, m¥u thu¨n vîi sè thä l  nk + 1. ành lþ 3.4 (p döng v o sè håc) Trong m = nk + 1 sè câ ½t nh§t n + 1 sè chia cho k câ còng sè d÷. Tuy nguy¶n lþ ÷ñc ph¡t biºu kh¡ ìn gi£n nh÷ng l¤i câ nhúng ùng döng h¸t sùc b§t ngí, thó và. B i vi¸t n y ch¿ xin n¶u mët sè ùng döng cõa nguy¶n l½ trong vi»c gi£i c¡c b i to¡n v· chia h¸t. V½ dö 3.24. Chùng minh r¬ng luæn tçn t¤i sè câ d¤ng 20112011 201100 0 chia h¸t cho 2012. 4 Líi gi£i. L§y 2013 sè câ d¤ng 2011; 20112011; ; |2011201{1z 2011} 2012 sè 2011 : L§y 2013 sè n y chia cho 2012. Theo nguy¶n l½ Dirichlet th¼ tçn t¤i hai sè câ còng sè d÷ khi chia cho 2012. Gi£ sû hai sè â l  |2011201{1z 2011} m sè 2011 v  2|011201{1z 2011} n sè 2011 (m n 0). =) 2012 j |2011201{1z 2011} m sè 2011 2|011201{1z 2011} n sè 2011 Di¹n  n To¡n håc Chuy¶n · Sè håc
  • 59. 3.2. Ph÷ìng ph¡p gi£i c¡c b i to¡n chia h¸t 51 =) 2012 j |2011201{1z 2011} mn sè 2011 |00 {z 00} n sè 2011 Vªy tçn t¤i sè thäa m¢n · b i. V½ dö 3.25. Chùng minh r¬ng trong 101 sè nguy¶n b§t k¼ câ thº t¼m ÷ñc hai sè câ 2 chú sè tªn còng gièng nhau. 4 Líi gi£i. L§y 101 sè nguy¶n ¢ cho chia cho 100 th¼ theo nguy¶n l½ Dirichlet tçn t¤i hai sè câ còng sè d÷ khi chia cho 100. Suy ra trong 101 sè nguy¶n ¢ cho tçn t¤i hai sè câ chú sè tªn còng gièng nhau. V½ dö 3.26 (Tuyºn sinh 10 chuy¶n HSPHN, 1993). Cho 5 sè nguy¶n ph¥n bi»t tòy þ a1; a2; a3; a4; a5. Chùng minh r¬ng t½ch P = (a1 a2)(a1 a3)(a1 a4)(a1 a5)(a2 a3) (a2 a4)(a2 a5)(a3 a4)(a3 a5)(a4 a5) chia h¸t cho 288. 4 Líi gi£i. Ph¥n t½ch 288 = 25 32. 1. Chùng minh 9 j P: Theo nguy¶n l½ Dirichlet th¼ trong 4 sè a1; a2; a3 câ hai sè câ hi»u chia h¸t cho 3. Khæng m§t t½nh têng qu¡t, gi£ sû: 3 j a1 a2. X²t 4 sè a2; a3; a4; a5 công câ hai sè câ hi»u chia h¸t cho 3. Nh÷ vªy P câ ½t nh§t hai hi»u kh¡c nhau chia h¸t cho 3, tùc 9 j p. 2. Chùng minh 32 j P: Theo nguy¶n l½ Dirichlet th¼ täng 5 sè ¢ cho tçn t¤i ½t nh§t 3 sè câ còng t½nh ch®n l´. Ch¿ câ thº câ hai kh£ n«ng sau x£y ra: N¸u câ ½t nh§t 4 sè câ còng t½nh ch®n l´, th¼ tø bèn sè câ thº lªp th nh s¡u hi»u kh¡c nhau chia h¸t cho 2. Do â 32 j P. Chuy¶n · Sè håc Di¹n  n To¡n håc
  • 60. 52 3.2. Ph÷ìng ph¡p gi£i c¡c b i to¡n chia h¸t N¸u câ 3 sè câ còng t½nh ch®n l´. Khæng m§t t½nh têng qu¡t, gi£ sû ba sè â l  a1; a2; a3. Khi â a4; a5 công còng t½nh ch®n l´ nh÷ng l¤i kh¡c t½nh ch®n l´ cõa a1; a2; a3. Khi â c¡c hi»u sau chia h¸t cho 2: a1 a2; a1 a3; a2 a3; a4 a5. M°t kh¡c, trong 5 sè ¢ cho câ ½t nh§t hai hi»u chia h¸t cho 4, cho n¶n trong 4 hi»u a1 a2; a1 a3; a2 a3; a4 a5 câ ½t nh§t mët hi»u chia h¸t cho 4. Vªy 32 j P. Ta câ pcm. V½ dö 3.27. Cho 2012 sè tü nhi¶n b§t k¼ a1; a2; ; a2012. Chùng minh r¬ng tçn t¤i mët sè chia h¸t cho 2012 ho°c têng mët sè sè chia h¸t cho 2012. 4 Líi gi£i. X²t 2012 sè S1 = a2 S2 = a1 + a2 S2012 = a1 + a2 + + a2012 Tr÷íng hñp 1: N¸u tçn t¤i sè Si (i = 1; 2; ; 2012) chia h¸t cho 2012 th¼ b i to¡n chùng minh xong. Tr÷íng hñp 2: N¸u 2012 - Si vîi måi i = 1; 2; ; 2012. em 2012 sè n y chia cho 2012 nhªn ÷ñc 2012 sè d÷. C¡c sè d÷ nhªn gi¡ trà thuëc tªp f1; 2; ; 2011g. V¼ câ 2012 sè d÷ m  ch¿ câ 2011 gi¡ trà n¶n theo nguy¶n l½ Dirichlet ch­c ch­n câ hai sè d÷ b¬ng nhau. G¿a sû gåi hai sè â l  Sm v  Sn câ còng sè d÷ khi chia cho 2012 (m; n 2 N; 1 n m 2012) th¼ hi»u Sm Sn = an+1 + an+2 + + am chia h¸t cho 2012. Di¹n  n To¡n håc Chuy¶n · Sè håc
  • 61. 3.2. Ph÷ìng ph¡p gi£i c¡c b i to¡n chia h¸t 53 Nhªn x²t. Ta câ thº rót ra b i to¡n têng qu¡t v  b i to¡n mð rëng sau: B i to¡n 3.5 (B i to¡n têng qu¡t). Cho n sè a1; a2; ; an. Chùng minh r¬ng trong n sè tr¶n tçn t¤i mët sè chia h¸t cho n ho°c têng mët sè sè chia h¸t cho n. 4 B i to¡n 3.6 (B i to¡n mð rëng). (T¤p ch½ To¡n Tuêi Thì sè 115) Cho n l  mët sè chuy¶n d÷ìng v  n sè nguy¶n d÷ìng a1; a2; ; an câ têng b¬ng 2n 1. Chùng minh r¬ng tçn t¤i mët sè sè trong n sè ¢ cho câ têng b¬ng n. 4 B i tªp · nghà B i 1. Chùng minh r¬ng câ væ sè sè chia h¸t cho 201311356 m  trong biºu di¹n thªp ph¥n cõa c¡c sè â khæng câ c¡c chú sè 0; 1; 2; 3. B i 2. (HSG 9 H  Nëi, 2006) Chùng minh r¬ng tçn t¤i sè tü nhi¶n n6= 0 thäa m¢n 313579 j (13579n 1). B i 3. Chùng minh r¬ng trong 52 sè nguy¶n d÷ìng b§t k¼ luæn luæn t¼m ÷ñc hai sè câ têng ho°c hi»u chia h¸t cho 100. B i 4. Cho 10 sè nguy¶n d÷ìng a1; a2; ; a10. Chùng minh r¬ng tçn t¤i c¡c sè ci 2 f0;1; 1g; (i = 1; 10) khæng çng thíi b¬ng 0 sao cho A = c1a1 + c2a2 + + c10a10 chia h¸t cho 1032: B i 5. Chùng minh r¬ng tçn t¤i sè tü nhi¶n k sao cho 2002k 1 chia h¸t cho 200310. B i 6. Bi¸t r¬ng ba sè a; a+k; a+2k ·u l  c¡c sè nguy¶n tè lîn hìn 3. Chùng minh r¬ng khi â k chia h¸t cho 6. Chuy¶n · Sè håc Di¹n  n To¡n håc
  • 62. 54 3.2. Ph÷ìng ph¡p gi£i c¡c b i to¡n chia h¸t 3.2.7 Ph£n chùng Cì sð: º chùng minh p - A(n), ta l m nh÷ sau: Gi£ sû ng÷ñc l¤i p j A(n). Chùng minh i·u ng÷ñc l¤i sai. V½ dö 3.28. Chùng minh r¬ng vîi måi sè nguy¶n n th¼ n2+n+1 khæng chia h¸t cho 9. 4 Líi gi£i. Gi£ sû 9 j (n2 +n+1). Khi â n2 +n+1 = (n+2)(n1)+3 chia h¸t cho 3. Suy ra 3 j n + 2 v  3 j n 1. Nh÷ vªy (n + 2)(n 1) chia h¸t cho 9, tùc n2 + n + 1 chia 9 d÷ 3, m¥u thu¨n. Ta câ pcm. Nhªn x²t. B i to¡n n y v¨n câ thº gi£i theo ph÷ìng ph¡p x²t sè d÷. V½ dö 3.29. Gi£ sû p = k:2t + 1 l  sè nguy¶n tè l´, t l  sè nguy¶n d÷ì ng v  k l  sè tü nhi¶n l´. Gi£ thi¸t x v  y l  c¡c sè tü nhi¶n m  p j x2t + y2t . Chùng minh r¬ng khi â x v  y çng thíi chia h¸t cho p. 4 Líi gi£i. Gi£ sû tr¡i l¤i p - x, suy ra p - y. Do p l  sè nguy¶n tè n¶n theo ành lþ Fermat nhä ta câ xp1 1 (mod p) yp1 1 (mod p) Theo gi£ thi¸t th¼ p 1 = k:2t, do â xk:2t 1 (mod p) yk:2t 1 (mod p) Tø â ta câ xk:2t + yk:2t 2 (mod p): (i) Theo gi£ thi¸t th¼ x2t + y2t 0 (mod p): Di¹n  n To¡n håc Chuy¶n · Sè håc
  • 63. 3.2. Ph÷ìng ph¡p gi£i c¡c b i to¡n chia h¸t 55 Do k l´ n¶n xk:2t + yk:2t = x2t k + y2t k ... x2t + y2t ) xk:2t + yk:2t 0 (mod p) (ii) Tø (i) v  (ii) suy ra i·u m¥u thu¨n. Vªy gi£ thi¸t ph£n chùng sai. Do â x; y çng thíi chia h¸t cho p. B i tªp · nghà B i 1. Chùng minh n2 + n + 2 khæng chia h¸t cho 15 vîi måi n 2 Z. B i 2. Chùng minh n2 +3n+5 khæng chia h¸t cho 121 vîi måi n 2 N. B i 3. Chùng minh 9n3 + 9n2 + 3n 16 khæng chia h¸t cho 343 vîi måi n 2 N. B i 4. Chùng minh 4n3 6n2 + 3n + 37 khæng chia h¸t cho 125 vîi måi n 2 N. B i 5. Chùng minh n3 +3n38 khæng chia h¸t cho 49 vîi måi n 2 N. Chuy¶n · Sè håc Di¹n  n To¡n håc
  • 64.
  • 65. Ch÷ìng 4 Ph÷ìng tr¼nh nghi»m nguy¶n 4.1 X²t t½nh chia h¸t 57 4.2 Sû döng b§t ¯ng thùc 74 4.3 Nguy¶n t­c cüc h¤n, lòi væ h¤n 86 Tr¦n Nguy¹n Thi¸t Qu¥n (L Lawliet) Ph¤m Quang To n (Ph¤m Quang To n) Trong ch÷ìng tr¼nh THCS v  THPT th¼ ph÷ìng tr¼nh nghi»m nguy¶n v¨n luæn l  mët · t i hay v  khâ èi vîi håc sinh. C¡c b i to¡n nghi»m nguy¶n th÷íng xuy¶n xu§t hi»n t¤i c¡c k¼ thi lîn, nhä, trong v  ngo i n÷îc. Trong b i vi¸t n y tæi ch¿ muèn · cªp ¸n c¡c v§n · cì b£n cõa nghi»m nguy¶n (c¡c d¤ng, c¡c ph÷ìng ph¡p gi£i) chù khæng i nghi¶n cùu s¥u s­c v· nâ. Tæi công khæng · cªp tîi ph÷ìng tr¼nh Pell, ph÷ìng tr¼nh Pythagore, ph÷ìng tr¼nh Fermat v¼ nâ câ nhi·u trong c¡c s¡ch, c¡c chuy¶n · kh¡c. 4.1 X²t t½nh chia h¸t 4.1.1 Ph¡t hi»n t½nh chia h¸t cõa 1 ©n V½ dö 4.1. Gi£i ph÷ìng tr¼nh nghi»m nguy¶n 13x + 5y = 175 (4.1) 57
  • 66. 58 4.1. X²t t½nh chia h¸t Líi gi£i. Gi£ sû x; y l  c¡c sè nguy¶n thäa m¢n ph÷ìng tr¼nh (4.1). Ta th§y 175 v  5y ·u chia h¸t cho 5 n¶n 13x .. .5 ) x ... 5 (do GCD(13; 5) = 1). °t x = 5t (t 2 Z). Thay v o ph÷ìng tr¼nh (4.1), ta ÷ñc 13:5t + 5y = 175 , 13t + y = 35 , y = 35 13t Do â, ph÷ìng tr¼nh (4.1) câ væ sè nghi»m nguy¶n biºu di¹n d÷îi d¤ng (x; y) = (5t; 35 13t); (t 2 Z) B i tªp · nghà B i 1. Gi£i ph÷ìng tr¼nh nghi»m nguy¶n 12x 19y = 285 B i 2. Gi£i ph÷ìng tr¼nh nghi»m nguy¶n 7x + 13y = 65 B i 3. Gi£i ph÷ìng tr¼nh nghi»m nguy¶n 5x + 7y = 112 4.1.2 ÷a v· ph÷ìng tr¼nh ÷îc sè V½ dö 4.2. T¼m nghi»m nguy¶n cõa ph÷ìng tr¼nh 3xy + 6x + y 52 = 0 (4.2) Líi gi£i. Nhªn x²t. èi vîi ph÷ìng tr¼nh n y, ta khæng thº ¡p döng ph÷ìng ph¡p tr¶n l  ph¡t hi»n t½nh chia h¸t, vªy ta ph£i gi£i nh÷ th¸ n o? Ta gi£i nh÷ sau: (4.2) , 3xy + y + 6x + 2 54 = 0 , y (3x + 1) + 2 (3x + 1) 54 = 0 , (3x + 1) (y + 2) = 54 Nh÷ vªy, ¸n ¥y ta câ x v  y nguy¶n n¶n 3x + 1 v  y + 2 ph£i l  ÷îc cõa 54. Nh÷ng n¸u nh÷ vªy th¼ ta ph£i x²t ¸n hìn 10 tr÷íng hñp sao? V¼: 4 = 1:54 = 2:27 = 3:18 = 6:9 = (1):(54) = (2):(27) = (3):(18) = (6):(9) Di¹n  n To¡n håc Chuy¶n · Sè håc
  • 67. 4.1. X²t t½nh chia h¸t 59 Câ c¡ch n o kh¡c khæng? C¥u tr£ líi l  câ! N¸u ta º þ mët chót ¸n thøa sè 3x + 1, biºu thùc n y chia cho 3 luæn d÷ 1 vîi måi x nguy¶n. Vîi lªp luªn tr¶n, ta ÷ñc: 2 664 3x + 1 = 1 y + 2 = 54 , x = 0 y = 52 3x + 1 = 2 y + 2 = 54 , x = 1 y = 56 V½ dö 4.3. Gi£i ph÷ìng tr¼nh nghi»m nguy¶n sau: 2x + 5y + 3xy = 8 (4.3) Líi gi£i. Ta câ (4:3) , x(2 + 3y) + 5y = 8 , 3x(2 + 3y) + 15y = 24 , 3x(2 + 3y) + 5(2 + 3y) = 34 , (3x + 5)(3y + 3) = 34 ¸n ¥y ph¥n t½ch 34 = 1 34 = 2 17 rçi x²t c¡c tr÷íng hñp. Chó þ r¬ng 3x + 5; 3y + 2 l  hai sè nguy¶n chia 3 d÷ 2, vªn döng i·u n y ta câ thº gi£m bît sè tr÷íng hñp c¦n x²t. V½ dö 4.4. Gi£i ph÷ìng tr¼nh nghi»m nguy¶n x2 y2 = 2011 (4.4) Líi gi£i. (4:4) , (x y)(x + y) = 2011. V¼ 2011 l  sè nguy¶n tè n¶n ÷îc nguy¶n cõa 2011 ch¿ câ thº l  1;2011. Tø â suy ra nghi»m (x; y) l  (1006; 1005); (1006;1005); (1006;1005); (1006; 1005). V½ dö 4.5. T¼m c¡c sè nguy¶n x; y tho£ m¢n i·u ki»n x2 + y2 = (x y)(xy + 2) + 9 (4.5) Chuy¶n · Sè håc Di¹n  n To¡n håc
  • 68. 60 4.1. X²t t½nh chia h¸t Líi gi£i. °t a = x y; b = xy. Khi â (4.5) trð th nh a2 + 2b = a(b + 2) + 9 , (a 2)(a b) = 9 (4.6) V¼ x; y 2 Z n¶n a; ; a2; ab ·u l  c¡c sè nguy¶n. Tø (4.6) ta câ c¡c tr÷íng hñp sau: ( a 2 = 9 a b = 1 , ( a = 11 b = 10 , ( x y = 11 xy = 10 (4.7) ( a 2 = 3 a b = 3 , ( a = 5 b = 2 , ( x y = 5 xy = 2 (4.8) ( a 2 = 1 a b = 9 , ( a = 3 b = 6 , ( x y = 3 xy = 6 (4.9) ( a 2 = 1 a b = 9 , ( a = 1 b = 10 , ( x y = 1 xy = 10 (4.10) ( a 2 = 3 a b = 3 , ( a = 1 b = 2 , ( x y = 1 xy = 2 (4.11) ( a 2 = 3 a b = 3 , ( a = 1 b = 2 , ( x y = 1 xy = 2 (4.12) D¹ th§y c¡c h» (4.7),(4.8),(4.10) khæng câ nghi»m nguy¶n, h» (4.9) væ nghi»m, h» (4.11) câ hai nghi»m nguy¶n (1; 2) v  (2;1), h» (4.12) câ hai nghi»m nguy¶n (1; 6) v  (6; 1). Tâm l¤i ph÷ìng tr¼nh (4.5) câ c¡c c°p nghi»m nguy¶n (x; y) l  (1; 2); (2;1); (1; 6); (6; 1). V½ dö 4.6. T¼m nghi»m nguy¶n cõa ph÷ìng tr¼nh: x2 + 1 y2 + 1 + 2 (x y) (1 xy) = 4 (1 + xy) (4.13) Di¹n  n To¡n håc Chuy¶n · Sè håc
  • 69. 4.1. X²t t½nh chia h¸t 61 Líi gi£i. Ph÷ìng tr¼nh (4.13) t÷ìng ÷ìng vîi: x2y2 + x2 + y2 + 1 + 2x 2x2y 2y + 2xy2 = 4 + 4xy , (x2 + 2x + 1)y2 2(x2 + 2x + 1)y + (x2 + 2x + 1) = 4 , (x + 1)2(y 1)2 = 4 , (x + 1)(y 1) = 2 (x + 1)(y 1) = 2 Vîi (x + 1)(y 1) = 2 m  x; y 2 Z n¶n ta câ c¡c tr÷íng hñp sau: x + 1 = 1 y 1 = 2 , x = 0 y = 3 x + 1 = 2 y 1 = 1 , x = 1 y = 2 x + 1 = 2 y 1 = 1 , x = 3 y = 0 x + 1 = 1 y 1 = 2 , x = 2 y = 1 Vîi (x + 1)(y 1) = 2 , t÷ìng tü ta công suy ra ÷ñc: x + 1 = 1 y 1 = 2 , x = 2 y = 3 x + 1 = 1 y 1 = 2 , x = 0 y = 1 x + 1 = 2 y 1 = 1 , x = 1 y = 0 x + 1 = 2 y 1 = 1 , x = 3 y = 2 Vªy ph÷ìng tr¼nh ¢ cho câ c¡c c°p nghi»m nguy¶n: (x; y) = f(0; 3); (1; 2); (3; 0); (2;1); (2; 3); (0;1); (1; 0); (3; 2)g V½ dö 4.7. T¼m nghi»m nguy¶n cõa ph÷ìng tr¼nh x6 + 3x3 + 1 = y4 (4.14) Chuy¶n · Sè håc Di¹n  n To¡n håc
  • 70. 62 4.1. X²t t½nh chia h¸t Líi gi£i. Nh¥n hai v¸ cõa ph÷ìng tr¼nh (4.14) cho 4, ta ÷ñc: 4x6 + 12x3 + 4 = 4y4 , (4x6 + 12x3 + 9) 4y4 = 5 , (2x3 + 3)2 4y4 = 5 , (2x3 2y2 + 3)(2x3 + 2y2 + 3) = 5: Vîi l÷u þ r¬ng 5 = 1:5 = 5:1 = (1):(5) = (5):(1) v  x; y 2 Z n¶n ta suy ra ÷ñc c¡c tr÷íng hñp sau: 2x3 2y2 + 3 = 1 2x3 + 2y2 + 3 = 5 , x3 y2 = 1 x3 + y2 = 1 , x3 = 0 y2 = 1 , 8 : x = 0 y = 1 x = 0 y = 1 2x3 2y2 + 3 = 1 2x3 + 2y2 + 3 = 5 , x3 y2 = 2 x3 + y2 = 4 , x3 = 3 y2 = 1 (lo¤i) 2x3 2y2 + 3 = 5 2x3 + 2y2 + 3 = 1 , x3 y2 = 1 x3 + y2 = 1 , x3 = 0 y2 = 1 (lo¤i) 2x3 2y2 + 3 = 5 2x3 + 2y2 + 3 = 1 , x3 y2 = 4 x3 + y2 = 2 , x3 = 3 y2 = 1 (lo¤i) Vªy ph÷ìng tr¼nh ¢ cho câ c¡c c°p nghi»m nguy¶n: (x; y) = f(0; 1); (0;1)g Nhªn x²t. B i to¡n n y công câ thº gi£i b¬ng ph÷ìng ph¡p kµp. V½ dö 4.8. Gi£i ph÷ìng tr¼nh nghi»m nguy¶n d÷ìng: 1 x + 1 y = 1 p (4.15) trong â p l  sè nguy¶n tè. 4 Di¹n  n To¡n håc Chuy¶n · Sè håc
  • 71. 4.1. X²t t½nh chia h¸t 63 Líi gi£i. (4:15) , xy = px + py ) (x y)(y p) = p2: V¼ p l  sè nguy¶n tè n¶n ÷îc sè nguy¶n cõa p2 ch¿ câ thº l  1;p;p2. Thû l¦n l÷ñt vîi c¡c ÷îc tr¶n ta d¹ t¼m ÷ñc k¸t qu£. Ph¦n tr¼nh b y xin d nh cho b¤n åc. Nhªn x²t. Ph÷ìng ph¡p n y c¦n hai b÷îc ch½nh: Ph¥n t½ch th nh ÷îc sè v  x²t tr÷íng hñp º t¼m k¸t qu£. Hai b÷îc n y câ thº nâi l  khæng qu¡ khâ èi vîi b¤n åc, nh÷ng xin nâi mët sè l÷u þ th¶m v· b÷îc x²t tr÷íng hñp. Trong mët sè b i to¡n, h¬ng sè nguy¶n ð v¸ ph£i sau khi ph¥n t½ch l  mët sè câ nhi·u ÷îc, nh÷ vªy ái häi x²t tr÷íng hñp v  t½nh to¡n r§t nhi·u. Mët c¥u häi °t ra l : L m th¸ n o º gi£m sè tr÷íng hñp bà x²t ¥y? V  º tr£ líi ÷ñc c¥u häi â, ta s³ tham kh£o v½ dö d÷îi ¥y. V½ dö 4.9. T¼m nghi»m nguy¶n cõa ph÷ìng tr¼nh: x2 + 12x = y2: (4.16) Líi gi£i. (thæng th÷íng) Ph÷ìng tr¼nh (4.16) ¢ cho t÷ìng ÷ìng vîi: (x + 6)2 y2 = 36 , (x + 6 + y)(x + 6 y) = 36 Suy ra x + y + 6; x + 6 y l  ÷îc cõa 36. M  sè 36 câ t§t c£ 18 ÷îc n¶n ta ph£i x²t 18 tr÷íng hñp t÷ìng ùng vîi x + 6 + y 2 f1;2;3;4;6;9;12;18;36g . K¸t qu£ l  ta t¼m ÷ñc c¡c c°p nghi»m nguy¶n (x; y) l  (0; 0); (12; 0); (16; 8); (16;8); (4; 8); (4;8) . Nhªn x²t. óng nh÷ v§n · m  ta ¢ n¶u ra ð tr¶n, sè ÷îc qu¡ nhi·u º x²t. Cho n¶n ta s³ câ c¡c nhªn x²t sau · thüc hi»n thao t¡c si¶u ph m chuyºn tø con sè 18 xuèng ch¿ cán 2! Chuy¶n · Sè håc Di¹n  n To¡n håc
  • 72. 64 4.1. X²t t½nh chia h¸t V¼ y câ sè mô ch®n trong ph÷ìng tr¼nh n¶n câ thº gi£ sû y 0. Khi â x + 6 y x + 6 + y, do vªy ta lo¤i ÷ñc t¡m tr÷íng hñp v  cán l¤i c¡c tr÷íng hñp sau: ( x + 6 + y = 9 x + 6 y = 4 ; ( x + 6 + y = 9 x + 6 y = 4 ; ( x + y + 6 = 1 x + y 6 = 36 ; ( x + y + 6 = 36 x y + 6 = 1 ; ( x + y + 6 = 2 x y + 6 = 18 ; ( x + y + 6 = 18 x y + 6 = 2 ; ( x + y + 6 = 3 x y + 6 = 12 ; ( x + y + 6 = 12 x y + 6 = 3 ; ( x + y + 6 = 6 x y + 6 = 6 ; ( x + y + 6 = 6 x + y 6 = 6 : B¥y gií ta ¢ câ 10 tr÷íng hñp, ta s³ ti¸p töc l÷ñc bä. Nhªn th§y (x + y + 6) (x + 6 y) = 2y n¶n x + 6 y v  x + 6 + y câ còng t½nh ch®n l´, do â ta lo¤i th¶m 6 tr÷íng hñp, ch¿ cán ( x + y + 6 = 18 x + y 6 = 2 ; ( x + y + 6 = 2 x + y 6 = 18 ; ( x + y + 6 = 6 x y + 6 = 6 ; ( x + y + 6 = 6 x + y 6 = 6 . Ti¸p töc x²t hai ph÷ìng tr¼nh ( x + y + 6 = 6 x y + 6 = 6 v  ( x + y + 6 = 6 x + y 6 = 6 , hai ph÷ìng tr¼nh n y ·u t¼m ÷ñc y = 0. Vªy sao khæng º ìn gi£n hìn, ta x²t y = 0 ngay tø ¦u. Ph÷ìng tr¼nh câ d¤ng x(x + 12) = y2, x²t hai kh£ n«ng: N¸u y = 0 th¼ x = 0 ho°c x = 12. N¸u y6= 0 th¼ x+6+y x+6y, ¡p döng hai nhªn x²t tr¶n ta ch¿ câ hai tr÷íng hñp: ( x + y + 6 = 2 x y + 6 = 18 v  ( x + y + 6 = 18 x y + 6 = 2 . Di¹n  n To¡n håc Chuy¶n · Sè håc
  • 73. 4.1. X²t t½nh chia h¸t 65 Ph÷ìng tr¼nh ¢ cho câ 6 nghi»m nguy¶n (x; y) = (16; 8); (0; 0); (12; 0); (16; 8); (4; 8); (4;8) Nhªn x²t. Nh÷ vªy b i to¡n ng­n gån, ch½nh x¡c nhí linh ho¤t trong vi»c x²t t½nh ch®n l´, giîi h¤n hai sè º gi£m sè tr÷íng hñp c¦n x²t. Ngo i c¡c c¡ch ¡nh gi¡ tr¶n ta cán câ thº ¡p döng x²t sè d÷ tøng v¸ º ¡nh gi¡ (¥y công l  mët ph÷ìng ph¡p gi£i ph÷ìng tr¼nh nghi»m nguy¶n). B i tªp · nghà B i 1. Thû bi¸n êi c¡c b i to¡n gi£i ph÷ìng tr¼nh nghi»m nguy¶n ð ph÷ìng ph¡p Biºu thà mët ©n theo ©n cán l¤i b¬ng ph÷ìng ph¡p ÷a v· ÷îc sè. B i 2. T¼m ë d i c¤nh mët tam gi¡c vuæng sao cho t½ch hai c¤nh huy·n g§p ba l¦n chu vi tam gi¡c â. B i 3. Gi£i ph÷ìng tr¼nh nghi»m nguy¶n x y + 2xy = 6 B i 4. Gi£i ph÷ìng tr¼nh nghi»m nguy¶n 2x + 5y + 2xy = 8 B i 5. (Thi HSG lîp 9 t¿nh Qu£ng Ng¢i n«m 2011-2012) Gi£i ph÷ìng tr¼nh nghi»m nguy¶n 6x + 5y + 18 = 2xy B i 6. T¼m nghi»m nguy¶n (xy 7)2 = x2 + y2 B i 7. T¼m x; y 2 Z thäa m¢n 2x2 2xy = 5x y 19. B i 8. T¼m nghi»m nguy¶n cõa ph÷ìng tr¼nh x2+6xy+8y2+3x+6y = 2. B i 9. T¼m nghi»m nguy¶n d÷ìng cõa ph÷ìng tr¼nh x3 y3 = xy +61 B i 10. T¼m nghi»m nguy¶n cõa ph÷ìng tr¼nh 4x2y2 = 22+x(1+x)+ y(1 + y) B i 11. Gi£i ph÷ìng tr¼nh nghi»m nguy¶n x(x + 1)(x + 7)(x + 8) = y2. Chuy¶n · Sè håc Di¹n  n To¡n håc
  • 74. 66 4.1. X²t t½nh chia h¸t B i 12. T¼m nghi»m nguy¶n d÷ìng cõa ph÷ìng tr¼nh 6x3 xy(11x + 3y) + 2y3 = 6 (T¤p ch½ TTT2 sè 106). B i 13. T¼m nghi»m nguy¶n d÷ìng cõa ph÷ìng tr¼nh x(x+2y)3y(y+ 2x)3 = 27 (t¤p ch½ THTT sè 398). B i 14. T¼m nghi»m nguy¶n cõa ph÷ìng tr¼nh p 9x2 + 16x + 96 = 3x 16y 24. B i 15. T¼m nghi»m nguy¶n d÷ìng cõa ph÷ìng tr¼nh 2 + s x + 1 2 + r x + 1 4 = y . B i 16. T¼m sè nguy¶n x º x2 4x 52 l  sè ch½nh ph÷ìng. B i 17. Gi£i ph÷ìng tr¼nh nghi»m nguy¶n x2 +2y2 +3xy 2xy = 6. B i 18. Gi£i ph÷ìng tr¼nh nghi»m nguy¶n x2 +3xy y2 +2x3y = 5. B i 19. Gi£i ph÷ìng tr¼nh nghi»m nguy¶n 2x2 +3y2 +xy 3x3 = y. B i 20. (Tuyºn sinh v o lîp 10 THPT chuy¶n tr÷íng KHTN H  Nëi n«m håc 2012-2013) T¼m t§t c£ c¡c c°p sè nguy¶n x; y thäa m¢n ¯ng thùc (x + y + 1)(xy + x + y) = 5 + 2(x + y). B i 21. Gi£i ph÷ìng tr¼nh nghi»m nguy¶n x42y4x2y24x27y2 5 = 0. (Thi HSG lîp 9 t¿nh H÷ng Y¶n n«m 2011-2012) B i 22. (Romanian 1999) Chùng minh r¬ng ph÷ìng tr¼nh sau khæng câ nghi»m nguy¶n x5 x4y 13x3y2 + 13x2y3 + 36xy4 36y5 = 1937 Di¹n  n To¡n håc Chuy¶n · Sè håc
  • 75. 4.1. X²t t½nh chia h¸t 67 4.1.3 Biºu thà mët ©n theo ©n cán l¤i rçi sû döng t½nh chia h¸t V½ dö 4.10. T¼m nghi»m nguy¶n cõa ph÷ìng tr¼nh 2x xy + 3 = 0 (4.17) Líi gi£i. Nhªn x²t. Ð ph÷ìng tr¼nh n y ta khæng thº ¡p döng c¡c c¡ch ¢ bi¸t, vªy ta ph£i l m sao? Chó þ hìn mët x½u núa ta th§y câ thº biºu di¹n y theo x ÷ñc rçi vªn döng ki¸n thùc t¼m gi¡ trà nguy¶n ð lîp 8 t¼m nghi»m nguy¶n cõa ph÷ìng tr¼nh, thû l m theo þ t÷ðng â xem sao. (4.17) , xy = 2x + 3 N¸u x = 0 th¼ ph÷ìng tr¼nh (4.17) ¢ cho væ nghi»m nguy¶n y. N¸u x6= 0 th¼ (4.17) , y = 2x + 3 x = 2 + 3 x Nh÷ vªy muèn y nguy¶n th¼ ta c¦n 3 x nguy¶n hay nâi c¡ch kh¡c x l  ÷îc cõa 3. Vîi méi gi¡ trà nguy¶n x ta t¼m ÷ñc mët gi¡ trà y nguy¶n. Tø â, ta câ bë nghi»m cõa (4.17) l  (x; y) = (3; 1); (1;1); (1; 5); (3; 3) V½ dö 4.11 (Thi HSG lîp 9 Qu£ng Ng¢i n«m 2011-2012). T¼m c¡c sè nguy¶n d÷ìng x; y sao cho 6x + 5y + 18 = 2xy (4.18) Nhªn x²t. H÷îng ph¥n t½ch v  ành h÷îng líi gi£i. ¢ x¡c ành ÷ñc ph÷ìng ph¡p cõa d¤ng n y th¼ b¥y gií ta s³ biºu di¹n ©n x theo y. Khæng khâ º vi¸t th nh x = 5y 18 6 2y . Ta d÷íng nh÷ nh¥n th§y biºu thùc n y r§t khâ ph¥n t½ch nh÷ biºu thùc ð v½ dö ¦u. Tuy nhi¶n, n¸u º þ k¾ s³ th§y b¶n m¨u l  2y v  tû l  5y, do â ta m¤nh d¤n nh¥n 2 v o tû º xu§t hi»n 2y gièng nh÷ ð m¨u. Chuy¶n · Sè håc Di¹n  n To¡n håc
  • 76. 68 4.1. X²t t½nh chia h¸t Líi gi£i. Ta câ (4.18) , x = 5y 18 6 2y , 2x = 10y 36 6 2y , 2x = 66 + 5(6 2y) 6 2y = 66 6 2y + 5 , 2x = 33 3 y + 5 Nh÷ vªy muèn x l  sè nguy¶n d÷ìng th¼ 3 y l  ph£i l  ÷îc cõa 33. Hay 3y 2 f1;3;11;33g. L¤i º þ r¬ng v¼ y 1 n¶n 3 y 2. Do â ch¿ câ thº 3 y 2 f1;3;11;33g. Ta câ b£ng sau: 3 y 1 1 3 11 33 y 2 4 6 14 36 x 14 19 8 4 3 Thû l¤i th§y c¡c c°p (x; y) nguy¶n d÷ìng thäa m¢n (4.18) l  (x; y) = (19; 4); (8; 6); (4; 14); (3; 36). Nhªn x²t. B i n y ta công câ thº sû döng ph÷ìng ph¡p ÷a v· ph÷ìng tr¼nh ÷îc sè. Công xin chó þ vîi b¤n r¬ng ð líi gi£i tr¶n th¼ ta ¢ nh¥n 2 ð x º bi¸n êi, do â ph£i câ mët b÷îc thû l¤i coi gi¡ trà x; y t¼m ÷ñc câ thäa m¢n (4.18) hay khæng rçi mîi câ thº k¸t luªn. B i tªp · nghà B i 1. Gi£i ph÷ìng tr¼nh nghi»m nguy¶n x2 xy = 6x 5y 8. B i 2. Gi£i ph÷ìng tr¼nh nghi»m nguy¶n x2 + x + 1 = 2xy + y. B i 3. Gi£i ph÷ìng tr¼nh nghi»m nguy¶n x3 x2y + 3x 2y 5 = 0: B i 4. (V o 10 chuy¶n THPT HKHTN H  Nëi n«m 2001-2002) T¼m gi¡ trà x; y nguy¶n thäa m¢n ¯ng thùc (y 2)x2 + 1 = y2. Di¹n  n To¡n håc Chuy¶n · Sè håc
  • 77. 4.1. X²t t½nh chia h¸t 69 B i 5. (V o 10 chuy¶n THPT HKHTN H  Nëi n«m 2000-2001) T¼m c°p sè nguy¶n (x; y) thäa m¢n ¯ng thùc y(x 1) = x2 + 2. B i 6. T¼m sè nhä nh§t trong c¡c sè nguy¶n d÷ìng l  bëi cõa 2007 v  câ 4 chú sè cuèi còng l  2008. B i 7. T¼m nghi»m nguy¶n cõa ph÷ìng tr¼nh 5x 3y = 2xy 11. 4.1.4 X²t sè d÷ tøng v¸ Cì sð ph÷ìng ph¡p. åc ngay ti¶u · ph÷ìng ph¡p th¼ ch­c b¤n s³ hiºu ngay ph÷ìng ph¡p n y nâi ¸n vi»c x²t sè d÷ ð tøng v¸ cho còng mët sè. Vªy, t¤i sao l¤i ph£i x²t v  x²t nh÷ vªy câ lñi ½ch g¼ trong cæng cuëc gi£i to¡n? H¢y còng t¼m hiºu qua v½ dö ¦u sau: V½ dö 4.12. T¼m nghi»m nguy¶n cõa ph÷ìng tr¼nh x2 + y2 = 2011 (4.19) Líi gi£i. Ta câ x2; y2 chia 4 câ thº d÷ 0 ho°c 1 n¶n têng chóng chia 4 ch¿ câ thº d÷ 0; 1 ho°c 2. M°t kh¡c 2011 chia 4 d÷a 3 n¶n ph÷ìng tr¼nh (4.19) væ nghi»m nguy¶n. Nhªn x²t. Qua v½ dö ¦u n y th¼ ta ¢ th§y rã sè d÷ khi chia cho 4 cõa hai sè kh¡c nhau th¼ ph÷ìng tr¼nh væ nghi»m. Do â ta l¤i c ng hiºu th¶m möc ½ch cõa ph÷ìng ph¡p n y. Bªt m½ th¶m t½ núa th¼ ph÷ìng ph¡p n y chõ y¸u dòng cho c¡c ph÷ìng tr¼nh khæng câ nghi»m nguy¶n. Cho n¶n, n¸u b¤n b­t g°p mët ph÷ìng tr¼nh b§t k¼ m  b¤n khæng thº t¼m ra ÷ñc nghi»m cho ph÷ìng tr¼nh â, th¼ h¢y ngh¾ ¸n ph÷ìng ph¡p n y ¦u ti¶n. Cán b¥y gií ta ti¸p töc ¸n vîi v½ dö sau: V½ dö 4.13 (Balkan MO 1998). T¼m nghi»m nguy¶n cõa ph÷ìng tr¼nh x2 = y5 4 (4.20) Líi gi£i. Ta câ: x2 0; 1; 3; 4; 5; 9 (mod 11). Trong khi â y5 4 6; 7; 8 (mod 11): væ lþ. Vªy ph÷ìng tr¼nh (4.20) væ nghi»m nguy¶n. Chuy¶n · Sè håc Di¹n  n To¡n håc